Está en la página 1de 150

TEORÍA DE NÚMEROS

Enfoque Problem-solving

Gerard Romo Garrido


Toomates Coolección
Los documentos de Toomates son materiales digitales y gratuitos. Son digitales porque están pensados para ser consultados mediante un
ordenador, tablet o móvil. Son gratuitos porque se ofrecen a la comunidad educativa sin coste alguno. Los libros de texto pueden ser digitales o
en papel, gratuitos o en venta, y ninguna de estas opciones es necesariamente mejor o peor que las otras. Es más: Suele suceder que los mejores
docentes son los que piden a sus alumnos la compra de un libro de texto en papel, esto es un hecho.
Lo que no es aceptable, por inmoral y mezquino, es el modelo de las llamadas "licencias digitales" con las que las editoriales pretenden cobrar
a los estudiantes, una y otra vez, por acceder a los mismos contenidos (unos contenidos que, además, son de una bajísima calidad). Este modelo
de negocio es miserable, pues impide el compartir un mismo libro, incluso entre dos hermanos, pretende convertir a los estudiantes en un
mercado cautivo, exige a los estudiantes y a las escuelas costosísimas líneas de Internet, pretende pervertir el conocimiento, que es algo social,
público, convirtiéndolo en un producto de propiedad privada, accesible solo a aquellos que se lo puedan permitir, y solo de una manera
encapsulada, fragmentada, impidiendo el derecho del alumno de poseer todo el libro, de acceder a todo el libro, de moverse libremente por todo
el libro.
Nadie puede pretender ser neutral ante esto: Mirar para otro lado y aceptar el modelo de licencias digitales es admitir un mundo más injusto, es
participar en la denegación del acceso al conocimiento a aquellos que no disponen de medios económicos, en un mundo en el que las modernas
tecnologías actuales permiten, por primera vez en la historia de la Humanidad, poder compartir el conocimiento sin coste alguno, con algo tan
simple como es un archivo "pdf".
El conocimiento no es una mercancía.

El proyecto Toomates tiene como objetivo la promoción y difusión entre el profesorado y el colectivo de estudiantes de unos materiales
didácticos libres, gratuitos y de calidad, que fuerce a las editoriales a competir ofreciendo alternativas de pago atractivas aumentando la calidad
de unos libros de texto que actualmente son muy mediocres, y no mediante retorcidas técnicas comerciales.

Este documento se comparte bajo una licencia “Creative Commons”: Se permite, se promueve y se fomenta cualquier uso, reproducción y
edición de todos estos materiales siempre que sea sin ánimo de lucro y se cite su procedencia. Todos los documentos se ofrecen en dos
versiones: En formato “pdf” para una cómoda lectura y en el formato “doc” de MSWord para permitir y facilitar su edición y generar versiones
parcial o totalmente modificadas. Se agradecerá cualquier observación, comentario o colaboración a
toomates@gmail.com

Actualmente, Toomates Coolección consta de los siguientes libros:

Geometría axiomática:
GA Geometría Axiomática pdf 1 2 ... 23 portada
PG Problemas de Geometría pdf 1234567
Problem-solving:
AR Teoría de números pdf 12
PT Trigonometría pdf doc
DE Desigualdades pdf doc
PC Números complejos pdf doc
PA Álgebra (en preparación) pdf doc
PC Combinatoria (en preparación) pdf doc
PR Probabilidad (en preparación) pdf doc
Libros de texto (En catalán)
AG Àlgebra pdf 12
FU Funcions pdf doc
GN Geometria analítica pdf 12
TR Trigonometria pdf doc
CO Nombres complexos pdf doc
AL Àlgebra Lineal 2n batxillerat pdf doc
GL Geometria Lineal 2n batxillerat pdf doc
CI Càlcul Infinitesimal 2n batxillerat pdf 12
PL Programació Lineal 2n batxillerat pdf doc
Recopilaciones de problemas
SE Compendium OME 1965-2019 pdf
SA Compendium AIME 1983-2019 pdf
ST Compendium PAU TEC 1998-2019 pdf
SC Compendium PAU CCSS 1998-2019 pdf
PM Problemas de Matemáticas pdf doc

Versión de este documento: 24/04/2020

www.toomates.net
Índice
1 Principios. →
El principio de buena ordenación. El principio de inducción. Principio del casillero.

2 Divisibilidad. →
El algoritmo de la división.

3 Máximo común divisor y mínimo común múltiplo. →


Números coprimos. El Teorema de Bezout. El algoritmo de Euclides.

4 Divisibilidad con identidades algebraicas. →

5 Números primos. →
Lema de Euclides. El Teorema fundamental de la aritmética (TFA).

6 Equivalencia modular. →
Congruencias.

7 Introducción a las ecuaciones diofánticas. →

8 Ecuaciones diofánticas lineales. →

9 Congruencias lineales y sistemas de congruencias lineales. →


Congruencias lineales. Inversos modulares. El Teorema chino del residuo.

10 El pequeño teorema de Fermat. →

11 La función Phi de Euler. →


Teorema de Euler.

12 Orden de un entero. →

13 Números factoriales. →
La función suelo. Números factoriales. La fórmula de Polignac.

14 Números combinatorios. Binomio de Newton. →

15 Números y primos de Fermat y de Mersenne. →

16 Número de divisores de un entero. →


El producto de los divisores de un entero.

17 Suma de los divisores de un entero. →


Números perfectos.

18 Problemas olímpicos de Teoría de números. →

Soluciones. →

Fuentes. →

Apéndice. →
El "problem-solving", tal y como yo lo entiendo.
Las competiciones AMC, un excelente sendero hacia las IMO detrás de un mar de siglas.
1 Principios.

Problemas sin teoría.


La "Teoría de números" o "aritmética" estudia las propiedades de los números enteros.
Los conceptos teóricos de esta rama de las matemáticas pueden ser complicados, muy
complicados y terriblemente complicados. Sin embargo, muchos problemas se
resuelven con solo utilizar el sentido común, toda una serie de estrategias y conceptos
que, de tan obvios que son, los libros de teoría no dedican tiempo a explicarlos.

En este primer apartado se incluyen problemas cuya resolución no necesita ningún


concepto teórico previo, sólo el sentido común, la pura lógica, y algunas formulitas de
la matemática elemental. Sin embargo, no hay que despreciarlos. Es fundamental que el
estudiante dedique a cada problema tanto tiempo como sea necesario, y si no llega a
resolverlo, estudie detenidamente la solución que se presenta al final del libro.

Dedicar tiempo a pensar un problema y estudiar detenidamente la solución es la


base del aprendizaje "problem-solving".

Bases de numeración.
Diremos que n se escribe como an an 1 an  2 ... a1 a0 en base b  2 si
n  anbn  an 1bn 1  an  2bn  2  ...  a1b  a0

con 0  ai  b , ai  IN , an  0

Por ejemplo, 3562 en base 7 es el número 3  73  5  72  6  7  2  1318 , y se escribe


35627 .

1.1 MF
¿Cuál de los siguientes enteros se puede expresar como la suma de 100 enteros positivos
consecutivos?

(A) 1,627,384,950 (B) 2,345,678,910 (C) 3,579,111,300 (D) 4,692,581,470 (E) 5,815,937,260

ASHME 1997 #20

1.2 M
Consideremos el entero N  9  99  999  9999  ..  99
...
99
321cifras

Calcula la suma de todas las cifras de N.

AIME I 2019 #1

1.3 M
Para cada entero positivo n , sea d n la cifra de las unidades de 1  2  3  ..  n .
2017
Determina el residuo cuando d
n 1
n se divide entre 1000.
AIME I 2017 #3
1.4 MF
Sea a0  2 , a1  5 , a2  8 , y para cada n  2 , define an recursivamente como el
residuo cuando 4an 1  an  2  an 3  se divide entre 11. Determina a2018  a2020  a2022 .

AIME II 2018 #2

1.5 F
Multiplicamos todos los números pares del 2 al 98 inclusive, excepto aquellos acabados
en 0. ¿Cuál será la cifra de las unidades del resultado?

(A) 0 (B) 2 (C) 4 (D) 6 (E) 8

AMC10 1999 Sample #14

1.6 M
Demostrar que si entre los infinitos términos de una progresión aritmética de números
enteros hay un cuadrado perfecto, entonces infinitos términos de la progresión son
cuadrados perfectos.

OME 1993-94 (Primera sesión) #1

Principio de la buena ordenación.


Todo conjunto S de números enteros no negativos contiene un elemento mínimo, es
decir, existe un elemento a  S tal que a  b para todo b  S .

1.7 D
a 2  b2
Demostrar que, si a, b son enteros positivos tales que es un entero, entonces
1  ab
a 2  b2
es un cuadrado perfecto.
1  ab
IMO 1988

Teorema. Propiedad arquimediana de los números naturales.


Si a y b son números enteros positivos, entonces existe un entero positivo n tal que
na  b.

Demostración.
Supongamos que no es cierto, es decir, que existen dos números a, b  0 tales que
n a  b para todo n  0 . Consideremos el conjunto S  b  na , n  0 .
Está claro que es un subconjunto de números enteros positivos, pues
n a  b  0  b  na , y por tanto le podemos aplicar el Principio de la buena
ordenación, es decir, contendrá un elemento mínimo b  m a , para cierto m  0 .
Pero, por hipótesis, b  (m  1) a también pertenecerá a S, luego:
b  (m  1) a  b  ma  a  b  ma , pues a  1 , luego b  m a no puede ser el mínimo,
llegando a contradicción.
Así pues, la propiedad arquimediana de los números naturales debe ser cierta, pues su
negación nos lleva a contradicción.
Principio de Inducción.
Sea S un conjunto de números enteros positivos cumpliendo las dos condiciones
siguientes:
a) 1 pertenece a S.
b) Si n  S , entonces n 1 S
Entonces S es el conjunto de todos los enteros positivos: S  1, 2 , 3,... 

Demostración.
Sea T  1, 2 , 3, ...  S , y supongamos que T   , es decir, que no está vacío, o lo que
es lo mismo, que no se cumple el Principio de Inducción.
Aplicando el Principio de la buena ordenación, T contendrá elemento mínimo,
llamémosle a .
Puesto que 1 T , pues por hipótesis, 1 S , está claro que a  1 , luego 0  a  1  a .
El número a  1 tampoco pertenecerá a S, pues si a  1 S  a  1  1  a  S ,
contradiciendo la hipótesis. Pero a 1  a , llegando a contradicción, pues habíamos
supuesto que a era mínimo.

El principio de inducción es una herramienta muy poderosa para demostrar fórmulas


que nos serán muy útiles para solucionar una enorme variedad de problemas.

Ejemplo 1.
n(2n  1)(n  1)
12  22  32  ...  n 2  para todo n  1, 2 , 3, ...
6

Demostración.
Sea S el conjunto de números enteros positivos para los que la fórmula anterior es
cierta.
Está claro que 1 pertenece a S, pues
1(2  1  1)(1  1)
12  1 
6
Supongamos que la fórmula anterior se cumple para un cierto valor n , y veamos que,
entonces, se cumplirá también para n  1 :
n(2n  1)(n  1)  n(2n  1) 
12  22  32  ...  n 2  n  1   (n  1) 2  (n  1)  (n  1)  
2

6  6 
 n(2n  1)  6(n  1) 
 (n  1)   (*)
 6 
n(2n  1)  6(n  1)  2n  n  6n  6  2n2  7n  6  (n  2)(2n  3)
2

 (n  2)(2n  3)  (n  1)(n  2)(2n  3) (n  1)(2(n  1)  1)(n  1  1)


(*)  (n  1)   
 6  6 6
k (2k  1)(k  1)
 , tomando k  n  1 , luego la fórmula también es válida para n  1 .
6
Así pues, S  1, 2 , 3,...  , es decir, la fórmula es válida para todos los enteros positivos.
Ejercicios.
Demuestra por inducción las fórmulas siguientes (para todo n  1, 2 , 3, ... ) :
n(n  1)
a) 1  2  3  ...  n  "números triangulares"
2
b) 1  3  5  ...  (2n  1)  n2
n(n  1)(n  2)
c) 1  2  2  3  3  4  ...  n(n  1) 
3
n (2n  1) (2n  1)
d) 12  32  52  ...  (2n  1)2 
3
 n (n  1) 
2

e) 13  23  33  ...  n3   
 2 
f) 1  2  22  ...  2n  2n1  1
n (n  1) (2n  1)
g) 12  22  32  ...  n 2 
6

Principio del casillero.


La idea que subyace en este principio es muy sencilla: Si tenemos tres automóviles y
solo dos garajes, necesariamente en uno de los garajes habrá más de un automóvil. El
enunciado general es el siguiente:

Si debemos distribuir n  1 objetos en n celdas o casillas, entonces al menos una de


ellas contiene más de un objeto.

Este principio se conoce también como Principio del palomar o Principio de Dirichlet
(Dirichlet, P.G. Lejeune 1805-1859).

1.8 F
¿Cuántas personas hay que reunir para asegurar que hay al menos dos que tengan
nombres con la misma inicial?

1.9 F
¿Cuántas personas hay que reunir para asegurar que hay al menos seis que tengan
nombres con la misma inicial?

1.10 F
Demostrar que en un grupo de siete personas hay como mínimo 4 con el mismo sexo.

1.11 F
Sea A un conjunto de veinte enteros tomados de la progresión aritmética
1 , 4 , 8 , 12 , ... , 100 .
Demostrar que existen en A dos enteros diferentes cuya suma es 104.

PUTNAM 1978
1.12 F
Demuestra que, en todo conjunto de siete números positivos distintos no superiores a
126, se encuentran dos elementos a , b tales que a  b  2a .

1.13 F
Demostrar que en todo subconjunto de 55 elementos de 1, 2 , 3 , ... ,100  siempre
podemos encontrar dos elementos que cuya diferencia sea 10.
2 Divisibilidad.
Armados únicamente con los conceptos más básicos de la divisibilidad, los que se aprenden a
los doce años, ya podemos enfrentarnos a problemas muy interesantes, incluso de nivel AIME.

Algoritmo de la división.
Para todo a  Z y b  N , existen q  Z y 0  r  b únicos, llamados respectivamente
cociente y residuo de la división, tales que
a  q b  r

Ejemplo.
Aplicando el Algoritmo de la división, demuestra que todo cuadrado es siempre de la
forma 4k o 4k  1.

Aplicando el Algoritmo de la división, todo número n será de la forma


n  4b , n  4b  1, n  4b  2 , n  4b  3  4b  4  1  4(b  1)  1 .
Veamos su cuadrado, caso por caso:
 
n  4b  n 2  4b   4 4b 2
2

n  4b  1  n 2  4b  1  16b 2  8b  1  4(4b 2  2b)  1


2

n  4b  2  n 2  4b  2  16b 2  16b  4  4(4b 2  4b  1)


2

Sea cual sea el caso, siempre es de la forma 4k o 4k  1.

Divisibilidad.
Dados dos números enteros a, b , diremos que a divide a b, o que b es divisible entre a,
y escribiremos a | b , cuando exista un tercer número entero c tal que a c  b , es decir,
cuando al realizar la división entera b entre a el residuo sea cero.

Propiedades de la divisibilidad.
a) a | a para todo entero a (propiedad reflexiva)
b) a | b y b | c  a | c (propiedad transitiva)
c) a | b y a | c  a | x b  y c para cualquier par de enteros x, y
d) a | b y c | d  ac | bd . En particular, a | b  a | b c .
e) a | b y a | b  c  a | c

Observación:
a | a n , y por tanto a n | c  a | c , por la propiedad transitiva: a | a n | c  a | c

Criterios de divisibilidad.
Entre 2: Cuando acaba en cero o cifra par.
Entre 3: Cuando la suma de sus cifras es múltiplo de 3.
Entre 5: Cuando acaba en 0 o en 5.
Entre 11: Cuando la diferencia entre la suma de sus cifras pares y la suma de sus cifras
impares sea 0 o múltiplo de 11.
2.1 MF
El número de dígitos de 416525 (cuando está escrito en la base 10 usual) es

(A) 31 (B) 30 (C) 29 (D) 28 (E) 27

AHSME 1984 #9

2.2 F
Demuestra que el cuadrado de un número impar es siempre de la forma 8k  1

OPOS BALEARES 2018

2.3 F
El perímetro de un triángulo equilátero excede el perímetro de un cuadrado en 1989 cm.
La longitud de cada lado del triángulo excede la longitud de cada lado del cuadrado en d
cm. El cuadrado tiene perímetro mayor que 0. ¿Cuántos posibles enteros positivos no
son válidos para d?

(A) 0 (B) 9 (C) 221 (D) 663 (E) infinitos

ASHME 40 #17

2.4 F
¿Cuántos números en base 10, N  a b c d satisfacen todas las tres condiciones
siguientes?
(i) 4000  N  6000
(ii) N es múltiplo de 5
(iii) 3  b  c  6

(A) 10 (B) 18 (C) 24 (D) 36 (E) 48

AHSME 1995 #12

2.5 F
La profesora Walter corrige un examen de matemáticas de sus cinco alumnos. Entra en
orden aleatorio las puntuaciones en una hoja de cálculo, que va recalculando la media
de la clase después de cada puntuación (sobre el número de alumnos ya introducidos, no
sobre el total de 5). La profesora se da cuenta de que, después de cada puntuación, la
media es siempre un entero. Las puntuaciones (presentadas en orden ascendente) son
71, 76, 80, 82 y 91. ¿Cuál fue la última puntuación que introdujo la profesora Walter?

(A) 71 (B) 76 (C) 80 (D) 82 (E) 91

AMC12 2000 #9
2.6 F
Determina los valores enteros de x para los cuales la expresión x 2  6 x es el cuadrado
de un entero.
2.7 M
Determina la suma de todos los números enteros positivos b  1000 tal que el número
36b (escrito en base b) es un cuadrado perfecto y el número 27b (también escrito en
base b) es un cubo perfecto.

AIME II 2018 #3

2.8 F
Determina la suma de todos los enteros positivos n tales que n2  85n  2017 es un
entero.

AIME II 2017 #6

2.9 F
Demuestra que ningún número de la forma 11,111,1111,11111,... es un cuadrado
perfecto.

Indicación: Todo número de la forma 111...111 se puede escribir como


111...111  111...108  3  4k  3 .

2.10 MF
¿Para cuantos enteros n entre 1 y 100 el polinomio x 2  x  n factoriza en el producto
de dos factores lineales con coeficientes enteros?

(A) 0 (B) 1 (C) 2 (D) 9 (E) 10

ASHME 1989 #8

2.11 F
Sea S el número de pares ordenados de enteros (a, b) con 1  a  100 y b  0 tales que
el polinomio x2  ax  b pueda ser factorizado como producto de dos (no
necesariamente distintos) factores lineales con coeficientes enteros. Determina el
residuo cuando S se divide entre 1000.

AIME I 2018 #1

2.12 F
Una sucesión pucelana es una sucesión crecientes de dieciséis números impares
positivos consecutivos, cuya suma es un cubo perfecto. ¿Cuántas sucesiones pucelanas
tienen solamente números de tres cifras?

OME Fase Nacional 2010 #1

2.13 M
a) Demuestra que el producto de dos números consecutivos es par.
b) Demuestra que el producto de tres números consecutivos es divisible entre 6.
c) Demuestra que n5  n es divisible entre 30.
2.14 F
Determina la suma de todos los números primos entre 1 y 100 tal que sean
simultáneamente 1 mayor que un múltiplo de 4 y 1 menor que un múltiplo de 5

ASHME 1999 #4

2.15 MF
¿Cuántos enteros positivos b existen con la propiedad de que log b 729 sea un entero
positivo?

(A) 0 (B) 1 (C) 2 (D) 3 (E) 4

AMC12 2000 #7

2.16 F
Aplicando el Algoritmo de la división, demuestra que:
a) El cuadrado de todo entero es siempre de la forma 3k o 3k  1 .
b) Un número de la forma 3a 2  1 nunca puede ser el cuadrado de un entero.

2.17 F
N2  7
¿Para cuántos enteros N entre 1 y 1990 la fracción impropia no es irreducible?
N 4

ASHME 1990 #19

2.18 MF
Sea r el residuo cuando 1059, 1417 y 2312 se dividen entre d  1 . Determina el valor de
d r .

AHSME 1976

2.19 F
Demuestra que existen infinitos números enteros n tales que n 2  23 es divisible por 24.

2.20 M
Determina todos los enteros positivos d tales que d divide n 2  1 y (n  1) 2  1 para
algún entero n.
Divisibilidad y orden.

Divisibilidad implica desigualdad.

Si a y b son positivos, a | b  a  b .
En general, si a | b y b  0  a  b . En particular, a | 1  a  1

2.21 F
Sean 1  d1  d2  d3  ...  dk  n los divisores del entero positivo n.
Encuentra todos los números n tales que n  d 22  d33 .
México 2008

Distribución de los divisores de n 2 .


Un dato que puede ser útil es que n es el divisor central de n 2 , es decir, el número de
divisores de n 2 menores que n es igual al número de divisores de n 2 mayores que n.

2.22 M
Sea n  231 319 . Determina el número de enteros positivos de n 2 que sean menores que
n y que no sean divisores de n .

AIME 1995 #6
3 Máximo común divisor y mínimo común múltiplo.
Máximo común divisor. Números coprimos. Mínimo común múltiplo.
Definimos el máximo común divisor de dos números a, b positivos como el mayor
divisor positivo común de ambos, y escribiremos a, b  . La unidad siempre es divisor
común, por lo que siempre existe el máximo común divisor, y a, b  1 . Diremos que
dos números son coprimos cuando su único divisor común es el 1, es decir, cuando
a, b  1 .
Definimos el mínimo común múltiplo de dos números a, b positivos como el menor
múltiplo común de ambos, y escribiremos a, b . Está claro que el producto ab es
múltiplo común de ambos, luego el mcm está bien definido.

Algunas propiedades del mcd y del mcm.


a) a, b | a, b y a, b | a, b .
b) a, b  a, b  a, b .
   
c) n a, n b  n a, b , a n , b n  a , b  y a n , b n   a , b  .
n n

d) a, b  a, a  b , y a, a  b | b .


e)  a , b , c     a , b ,  a , c   y  a , b, c    a, b, a, c  .
f) a b  a, b a, b .

Interpretación geométrica del máximo común divisor.


El máximo común divisor de a y b indica el número de puntos ( x, y) con coordenadas
enteras en el segmento que une los puntos ( 0 , 0 ) y ( a , b ) , sin contar el inicial ( 0 , 0 ) .
Por ejemplo, (15 , 6 )  3 , y por tanto el segmento que une los puntos ( 0 , 0 ) y (15 , 6 )
pasa por tres puntos con coordenadas enteras, aparte del propio ( 0 , 0 ) :

15 
 5 ( 5 , 2 )
3  
  ( 5  5 , 2  2 )  (10 ,4 )
6
 2  (10  5 , 4  2 )  (15 ,6 )
3 

3.1 F
Diremos que un punto ( x, y) del plano es “punto entero” cuando sus coordenadas sean
enteras. ¿Cuántos de estos puntos de este tipo hay (incluyendo ambos extremos) en el
segmento que cuyos extremos son ( 3,17 ) y ( 48 , 281) ?

ASHME 1989 #16

3.2 MF
Demostrar que (n, n  k ) | k , independientemente del valor de n . En particular,
(n, n  1)  1
Lema.
(a, b) divide a cualquier combinación lineal de a y b .

Demostración. Sea ax  by una combinación lineal de a y b .


(a, b) | a  (a, b) | ax 
  (a, b) | ax  by
(a, b) | b  (a, b) | by 

Teorema de Bezout (TDB).


Dados dos números enteros a, b no ambos cero, el máximo común divisor de a, b se
caracteriza por ser el elemento mínimo del conjunto no vacío

A   ax  by , x, y  Z , ax  by  0 

Luego existe, es único y siempre se puede escribir como combinación lineal de a y b :


Existen enteros x, y tales que (a, b)  ax  by

Demostración.
Consideremos el conjunto anterior A   ax  by , x, y  Z , ax  by  0 .
Es un conjunto no vacío pues al menos aa  bb  a 2  b2  0 pertenece a A (estamos en
todo momento suponiendo que a, b no son ambos cero).

Por el Principio de buena ordenación, A tendrá un mínimo, al que llamaremos d .


Vamos a demostrar que d  (a, b) .

Supongamos que d  a x1  b y1  0 para ciertos enteros x1 , y1 .


Por el Algoritmo de la división, existirán enteros q, r tales que a  d q  r con
0r d.

Si r  0 , entonces
0  r  a  dq  a  (a x1  b y1 )q  a  a x1q  b y1q  a(1  x1q)  b y1q , y por tanto, r
pertenece al conjunto A, tomando x2  1  x1q , y2   y1q . Pero r  d , lo cual
contradice la hipótesis de d como elemento mínimo. Luego r  0 y por tanto a  d q , es
decir, d divide al número a.

Con el mismo razonamiento se demuestra que d divide al número b, y por tanto d es


común divisor de a y b. Veamos que es el máximo común divisor.
Sea m otro común divisor de a y b, entonces, aplicando el lema anterior,
m | a x1  b y1  d  m | d  m  d .
Corolario.
Si x1 ,..., xn son números enteros y a es cualquier número entero positivo,
a x1,..., a xn   a x1,..., xn 
Demostración.
Sean d  a x1,..., a xn  y e  x1 ,..., xn  . Luego e | xi  a e | a xi  a e | d  a e  d .
Por el TDB, e  k1x1  ...  kn xn  a e  k1 ax1   ...  kn axn 
Es decir, ae es combinación lineal de a x1 ,..., a xn , y por tanto es un múltiplo de d ,
luego ae  d . Y, puesto que anteriormente hemos demostrado que a e  d , llegamos a
ae  d .

Corolario.
(a, b)  1  Existe una combinación lineal ax  by  1

Demostración.
 Es el TDB.
 (a, b) | ax  by  1  (a, b) | 1  (a, b)  1

3.3 M
21n  4
Demuestra que la fracción es irreducible para todo número natural n.
14n  3
IMO 1959

3.4 M
Los números de la sucesión 101,104 ,109 ,116, ... son de la forma an  100  n2 ,
n  1, 2, ...
Para cada n , sea d n   an , an1  . Determina max n 1 d n .
AIME 1985

Teorema.
a a a 
Si d  a1 , a2 , ..., an  entonces  1 , 2 , ..., n   1
d d d 

Demostración.
a a a 
d  a1 , a2 , ..., an   d | ai  ai  ki d . Sea d '   1 , 2 , ..., n  y supongamos que
d d d 
d ' 1.
a a a  a a
d '   1 , 2 , ..., n   d '| i  i  k 'i d '  ai  k 'i d ' d  d ' d | ai  d ' d | d
d d d  d d
Lo cual es imposible suponiendo d ' 1. Luego d ' 1 .
Teorema.
Si (a, b)  1 , entonces:

d | ac  d | ac 
a) a | bc  a | c b) d |c c)   d | ce
d | bc  d | be 

Demostración.
a)
a | bc  bc  ak1
d | bc  bc  dk2
(a, b)  1  1  k3a  k4b  c  k3ac  k4bc  k3ac  k4 ak1  ak3c  k4 k1   a | c

Por el TDB, (a, b)  1  1  ax  by para ciertos enteros x, y . Luego c  acx  bcy , y


puesto que trivialmente a | acx y por hipótesis a | bcy , se deduce a | acx  bcy  c .
b)
d | ac  ac  dk1
d | bc  bc  dk2
(a, b)  1  1  k3a  k4b  c  k3ac  k4bc  k3dk1  k4 dk2  d (k3k1  k4 k2 )  d | c

c)
d | ac  d | a(ce)
  d | ce aplicando el apartado anterior.
d | be  d | b(ec ) 

3.5 D
Sean m, n dos enteros positivos diferentes. Demostrar que

(m, n)  (m  1, n  1)  (m  2, n  2)  m  n  1

Indian National Mathematical Olympiad 2019 #3 (parcial)


El algoritmo de Euclides.
El algoritmo de Euclides es un método efectivo para calcular el máximo común divisor
de dos números a y b que se basa en el algoritmo de la división y el siguiente
principio:

Si b  a k  c , entonces (a, b)  (a, a k  c)  (a, c)

Suponiendo a  b , podemos dividir a entre b para expresar a  kb  r , con r  b y


así

(a, b)  (bk  r, b)  (r, b)

Este mismo proceso repetiremos una y otra vez, con números más y más pequeños,
hasta que el máximo común divisor se haga evidente.

Ejemplo 1.
Calcula  29 , 23  mediante el Algoritmo de Euclides.

Solución.
29  1  23  6  (29,23)  (1  23  6 , 23)  ( 6 , 23)
23  3  6  5  (23,6)  ( 3  6  5, 6 )  ( 5, 6 )
6  1 5  1  (6,5)  (1  5  1, 5 )  (1, 5 )  1
Así pues,  29 , 23   1

Ejemplo 2.
Calcula  3456 , 246  mediante el Algoritmo de Euclides.

Solución.
( 3456 , 246 )  (13  246  158, 246 )  (158, 246 )
(158 , 246 )  (158 ,1158  88 )  (158 , 88 )
Y de la misma manera: (158, 88 )  ( 70 , 88 )  (18, 70 )  (16 ,18 )  ( 2 ,16 )  2
Luego  3456 , 246   2
4 Divisibilidad con identidades algebraicas.
La primera identidad algebraica fundamental.
La identidad algebraica


xn  y n  x  y  x n 1  x n  2 y  x n 3 y 2  ...  x y n  2  y n 1 
es fundamental en la resolución de todo tipo de problemas de Aritmética.
Su demostración es fácil: Basta con desarrollar el producto de la derecha:

x  y  x n 1  x n  2 y  x n 3 y 2  ...  x y n  2  y n 1  
n 1 n2 n2 n 1 n 1
x x
n
yx y  ...  x y
2 2
 xy x y  x n  2 y 2  x n  3 y 3  ...  x y n 1  y n
 xn  yn
En particular, tomando y  1 obtenemos una identidad muy útil:
x n1  1
x n  x n1  x n2  ...  x  1 
x 1

De la primera identidad se desprende directamente el siguiente resultado:

a) a  b | a n  bn para cualquier n  1.

Por ejemplo, sin necesidad de ningún cálculo, 87672345  81012345 es divisible entre 666.

Que se puede generalizar para obtener el siguiente resultado:

b) Si d | n con d y n positivos, entonces a d  b d | a n  b n para todo entero a, b .

En efecto, supongamos que n  k d para cierto entero k . Entonces


a n  bn  a k d  bk d  a d    b   a
k d k d

 bd a d 1  a d  2b  a d 3b2  ... 
La segunda identidad algebraica fundamental.
Otra identidad algebraica muy útil es


Si n es impar, xn  y n  x  y  x n 1  xn  2 y  xn 3 y 2  ...  x y n  2  y n 1 
En efecto, Si n es impar, x n  y n  x n   y  , y basta aplicar la identidad anterior.
n

De esta segunda identidad se desprende de forma directa el siguiente resultado:

c) Si n es impar, a  b | a n  b n

Que se puede generalizar para obtener el siguiente resultado:

e) Si d | n con d y n positivos, y n / d es impar, entonces a d  b d | a n  b n para todo


entero a, b .
Proposición.
Si 2n  1 es primo, entonces n es una potencia de 2.

Demostración.
Supongamos, por el contrario, que n no es una potencia de 2, es decir, que podemos
escribir n  k  h , con k  1 impar.

Entonces
  k
 
2n  1  2k h  1  2h  1  2h  1k  2h  1 2h
k
   k 1
 2h k 2
 
 2h
k 3

 ...  2h  1

Y por tanto nuestro número es divisible entre 2h  1 .  


Observación 1.
Este resultado será fundamental en el Tema 15, como base para definir los “primos de
Fermat”.

Observación 2.
El recíproco no es cierto. Por ejemplo: 641 | 232  1 .

En efecto, utilizando que 641  24  54  5  27  1 ,


  
232  1  2 28 2 4  1  2 28 2 4  54  54  1  2 28 2 4  54  2 28 54  1    
   
 2 28 2 4  54  27  5  1  2 28  641  641  1  1  2 28  641  641  639 640 2  1 
4 4
 
 6412 28
 639640  1
2

En donde hemos utilizado que:



6404  1  6404  14  640  1 6403  6402  640  1  639 6403  6402  641    
  
 639 640 640  1  641  639 640  641  641  639  641 640  1
2 2
  2

(En el Tema 6 volveremos a demostrar este resultado aplicando la aritmética modular)

4.1 F
Determina todos los números primos de la forma n3  1 , para enteros n  1 .

4.2 M
Demostrar que 2903n  803n  464n  261n es divisible entre 1897 para todo natural n .

EOTVOS 1899

4.3 M
Demuestra que n4  4 con n  IN es primo si y solo si n  1 .

4.4 M
Determina todos los enteros n  1 para los cuales n4  4n es un número primo.
4.5 F
Demuestra que, para todo n  IN , n 2 divide a n  1  1 .
n

4.6 F
Demostrar que 1001 divide 11993  21993  31993  ...  10001993

4.7 M
Demuestra que 7 divide al número 147  247  347  447  547  647

4.8 F
Si al cuadrado de un número de dos dígitos se le resta el cuadrado del número formado
invirtiendo el orden de sus dígitos, entonces el resultado no siempre será divisible por:

(A) 9 (B) El producto de los dígitos. (C) La suma de los dígitos.


(D) La diferencia de los dígitos. (E) 11

ASHME 1957 #24


5 Números primos.
Números primos.
Todo número es divisible por sí mismo y por la unidad. Diremos que un número natural
p  1 es primo cuando solo sea divisible por sí mismo y por la unidad. Por ejemplo, son
primos los números 5, 13, 59 o 397. Llamamos compuestos a los números que no son
primos. El número 1 no se considera ni primo ni compuesto.

Algunas propiedades de los números primos.


a) Si p es primo y a | p , entonces a  1 ,  p .
b) Si p y q son primos, entonces p | q  p  q .
c) Todos los primos son impares excepto el 2.
d) El 2 y el 3 son los únicos primos cuya diferencia es 1.
e) Si p es primo y a, b  N , entonces p | a b  p | a .

Ejemplo.
Demuestra que si p  3 es primo, entonces 24 | p 2  1.

Solución.
Por el algoritmo de la división, todo número se puede representar como 6n , 6n  1 o
6n  2 .
Si es primo, la única opción aceptable es 6n  1 , pues las otras son divisibles entre 2 o
3.
Luego
p  6n  1  p 2  6n  1  36n2  12n  1  p 2  1  36n2  12n  12n(3n  1)
2

Está claro que o bien n es par o bien 3n  1 es par, luego 24 | p 2  1.

Ejemplo.
Determina la suma de todos los números primos entre 1 y 100 que son simultáneamente
1 más que un múltiplo de 4 y 1 menos que un múltiplo de 5.

AHSME 1999 #3
Solución.
p  4a  1
  4a  1  5b  1  5b  4a  2  2a  1
p  5b  1 
 2 | 5b  2 | b  b  2c  p  5(2c)  1  10c  1
Con p  10c  1 ya tenemos un conjunto de candidatos suficientemente pequeño como
para proceder a testearlos, uno por uno:
c  1  p  10  1  1  9 (no es primo) c  2  p  10  2  1  19  4  4  3
c  3  p  10  3  1  29  4  7  1 c  4  p  10  4  1  39  4  9  3
c  5  p  10  5  1  49 c  6  p  10  6  1  59  4  14  3
c  7  p  10  7  1  69 c  8  p  10  8  1  79  4  19  3
c  9  p  10  9  1  89  4  22  1 c  10  p  10  10  1  99

Luego la suma es 29+89=118.


Lema de Euclides.
Si p es primo y p | ab entonces p | a o p | b .

Demostración.
Sea p un número primo. Si p | a entonces (a, p)  1 y por el TDB existirán enteros
x, y tales que 1  ax  py , y por tanto b  bax  bpy .
Pero p | ab  p | bax , y claramente p | bpy , luego p | bax  bpy  b

5.1 M
Sean x, y enteros. Demostrar que 2 x  3 y es divisible entre 17 si y solo si 9 x  5 y es
divisible entre 17.

Corolario.
a) a es par si y solo si a n es par
b) a es impar si y solo si a n es impar.

Demostración.
 
a)  a  2k  a n  (2k )n  2 2n 1 k n
 Basta aplicar el Lema de Euclides con p  2 .
n
 n
b)  a  2k  1  a n  (2k  1)n  1    2k  impar.
k

k 1  k 

 Por el apartado a, si a es par entonces a n es par, luego si a n es impar,


necesariamente a debe ser par.

Corolario.
Hay infinitos números primos.

Demostración.
Entre otras muchas demostraciones de este resultado, la de Euclides es un ejemplo de
elegancia:
Supongamos, por el contrario, que existe una cantidad finita de números primos. Sean
estos
1  p1  p2  ...  pn
Consideremos el número n  p1  p2  pn  1 .
No puede ser primo, pues n  pn , luego existirá al menos un k tal que pk | n , pero
también se cumple trivialmente pk | n  1  p1  p2  pn , y por tanto
pk | n  (n  1)  1  pk | 1  pk  1 , lo cual es imposible. Así pues, no es posible que
exista un número finito de primos.
Teorema fundamental de la aritmética (TFA).
Todo entero positivo n se descompone de forma única como producto de potencias de
números primos (el orden es irrelevante).
n  p1a1 p2a2 ... pkak

La expresión anterior se denomina descomposición canónica de n .

Por ejemplo: 18  2  32 , 84  22  3  7 , 4576  2  11  13 , 32716  2  8179

Este teorema es la clave para resolver muchísimos problemas de aritmética, pues reduce
el problema a un recuento de casos.

Proposición.
Dado un número en descomposición canónica n  p1a1 p2a2 ... pkak ,
a) Todo divisor de n es de la forma d  p1e1 p2e2 ... pkek , con 0  ei  ai .
b) El número de divisores de n es a1  1a2  1...ak  1 .

Demostración.
a) Todo número de la forma d  p1e1 p2e2 ... pkek , con 0  ei  ai es un divisor de n, y son
todos diferentes por el TFA.
b) Basta aplicar el principio fundamental del conteo.

5.2 F
Existen enteros positivos A, B y C, sin factores comunes mayores que 1, tales que
A log 200 5  B log 200 2  C
Determina A  B  C .
ASHME 1995 #24

5.3 M
Demostrar que existe un único número natural n tal que 28  211  2n es un cuadrado
perfecto.

5.4 F
Si 1998 se escribe como producto de dos enteros positivos cuya diferencia es lo más
pequeña posible, entonces la diferencia es:

(A) 8 (B) 15 (C) 17 (D) 47 (E) 93

AHSME 1998 #6

5.5 M
Determina los tres números naturales consecutivos más pequeños cuya suma es un
cuadrado perfecto y un cubo perfecto de números naturales.
5.6 M
Halla todas las sucesiones finitas de n números naturales consecutivos a1 , a2 ,..., an , con
n  3 , tales que a1  a2 ,...  an  2009
OME Fase Nacional 2009 #1

5.7 MF
En el año 2001, los Estados Unidos acogieron las Olimpiadas Matemáticas. Sean
I , M , O enteros positivos tales que I  M  O  2001 . ¿Cuál será el valor más grande
posible de la suma I  M  O ?

(A) 23 (B) 55 (C) 99 (D) 111 (E) 671

AMC12 2000 #1

5.8 F
Existen enteros positivos A, B, C , sin factores comunes mayores que 1, tales que
A log 200 5  B log 200 2  C .

¿Cuál es el valor de A  B  C ?

(A) 6 (B) 7 (C) 8 (D) 9 (E) 10

AHSME 1995 #24

5.9 M
¿Cuántos conjuntos de tres elementos  a, b, c  de enteros positivos verifican
a  b  c  2310 ?

(A) 32 (B) 36 (C) 40 (D) 43 (E) 45

AHSME 1995 #29

5.10 F
Existe un número primo p tal que 16 p  1 es el cubo de un entero positivo. Determina p.

AIME I 2015 #3

5.11 F
Cuando los números 702, 787 y 855 son divididos entre el entero positivo m, el resto es
siempre el mismo entero positivo r. Cuando los números 412, 722 y 815 son divididos
entre el entero positivo n, el resto es siempre el mismo entero positivo s  r . Determina
mnr  s.

AIME I 2017 #2
5.12 M
Determina el número de polinomios de segundo grado f (x) con coeficientes enteros y
ceros enteros tales que f (0)  2010 .

AIME II 2010 #10

5.13 M
Determina todos los primos p para los cuales la ecuación x 2  px  444 p  0 tiene
soluciones enteras.

ASHME 1987 #23

5.14 M
Demostrar que en cualquier conjunto de n  1 números entre 1 y 2n siempre podemos
encontrar dos elementos tales que el menor divide al mayor.

5.15 F
Determina el número de valores de k sabiendo que 1212 es el mínimo común múltiplo de
66, 88 y k.

AIME 1998

5.16 F
Denotamos por [r,s] el mínimo común múltiplo de los enteros positivos r y s. Determina
el número de triplas ordenadas a, b, c tales que [a,b]=1000, [b,c]=2000 y [c,a]=2000.

5.17 F
Determina 3x 2 y 2 si x, y son enteros tales que y 2  3x 2 y 2  30 x 2  517

AIME 1987 #5

5.18 M
Determina p sabiendo que es un número primo tal que 16 p  1 es el cubo de un
número entero.

AIME I 2015 #3

5.19 M
Determina todos los enteros a, b  0 tales que
(a, b)  a, b  a  b  6
6 Equivalencia modular.
El lenguaje de las congruencias nos permite abordar con éxito problemas aparentemente
muy difíciles. Las congruencias es un lenguaje, una técnica, y por tanto solo se aprende
practicando, jugando con ella durante mucho tiempo.

Definición de congruencias.
Diremos que a  b (mod n) , y diremos que "a es congruente con b módulo n" cuando
sucede alguna de estas condiciones equivalentes:
a) n | (a  b)
b) a  k n  b para cierto entero k .
c) a y b dejan el mismo residuo cuando son divididos entre n.

Por ejemplo: 24  3 (mod 7) , pues 24  3  7  3 , 34  4 (mod 5) , pues 34  6  5  4


En particular: a  0 (mod n)  n | a

Ejemplo.
Determina x tal que 5x  6 (mod 8) .

Solución.
Vamos probando, uno por uno:
5 1  5  0  8  5  5 (mod 8) 5  2  10  1 8  2  2 (mod 8)
5  3  15  1 8  7  7 (mod 8) 5  4  20  2  8  4  4 (mod 8)
5  5  25  3  8  1  1 (mod 8)
5  6  30  3  8  6  6 (mod 8)  La solución es x  6 (mod 8)
5  7  35  4  8  3  3 (mod 8)

Propiedades de las congruencias.


Sea n  0 fijo, y a, b, c, d enteros arbitrarios. Entonces se cumple:

a) a  a (mod n) (Propiedad reflexiva).


b) Si a  b (mod n)  b  a (mod n) (Propiedad simétrica).
c) Si a  b (mod n) y b  c (mod n)  a  c (mod n) (Propiedad transitiva).
d) Si a  b (mod n) y c  d (mod n)  a  c  b  d (mod n) y a c  b d (mod n) .
e) Si a  b (mod n)  k a  k b (mod n) para cualquier entero k

f) Si a  b (mod n)  a k  b k (mod n) para cualquier entero positivo k .


Qué funciona y qué no funciona con congruencias.
Las propiedades anteriores nos permiten trabajar con congruencias prácticamente igual
a como trabajamos con números, pero no todo lo que hacíamos con números funciona
ahora con congruencias:

a) Cancelación de términos:
No funciona en general la cancelación de términos, el "tachar" de toda la vida.
Por ejemplo: 2  4  2  1 (mod 6) , pero 4  1 (mod 6)
217  21 (mod 7) , pero 216  1 (mod 7)

Aunque existe una Regla de cancelación:


Si (c, n)  1 , entonces ca  cb (mod n)  a  b (mod n)

b) Principio del producto nulo:


No existe tampoco el principio del producto nulo en general. Por ejemplo:
4  3  0 (mod 12) , pero 4  0 (mod 12) y 3  0 (mod 12)

Pero sí se verifica cuando el módulo es un número primo:


Si p es primo, a  b  0 (mod p)  a  0 (mod p) o b  0 (mod p)

Modificaciones en el módulo.
 m
a) Si (a, m)  d  1 , entonces ax  ay (mod m)  ax  ay  mod  .
 d
b) Si a  b (mod n) y d | n  a  b (mod d )
c) a  b (mod n) y a  b (mod m)  a  b  mod n, m  .
d) Si n1 , n2 ,... , nk   1 , a  b (mod ni ) i  1,..., k  a  b (mod n1n2 ...nk )

Demostración.
a) ax  ay (mod m)  m | ax  ay   m | ax  y   | x  y 
m a
d d
b) a  b (mod n m)  n m | (a - b)  n | (a - b)  a  b (mod n)
a  b (mod n)  n | (a  b) 
c)   n, m | (a  b)  a  b  modn, m  )
a  b (mod m)  m | (a  b)
d) Es un caso particular de c.
Trabajar con congruencias es una técnica muy potente para resolver problemas de
Teoría de Números, como se puede ver en los siguientes ejemplos:

Ejemplo 1.
Demostrar que 41 divide 220  1

Solución.
En primer lugar vemos que 25  32  9 (mod 41)
 
Luego 220  25   9 (mod 41)   9  9 (mod 41)  81  81 (mod 41)
4 4 2 2

Pero, por otro lado, 81 (mod 41)  1 (mod 41)


Luego 81  81 (mod 41)  (1)(1) (mod 41)  1 (mod 41)
Finalmente, 220  1  1  1 (mod 41)  0 (mod 41)  41 | 220  1 , tal y como queríamos
ver.

Ejemplo 2.
Determina el residuo al dividir 1! 2! 3! 4! ...  99!100! entre 12.

Solución.
Observamos que 4! 24  0 (mod 12) , luego, para todo k  4 ,
k! 4!5  6...  k  0  5  6...  k  0 (mod 12)
luego
1!2!3!4!...  99!100! 1!2!3!4!0  ...  0  0  1!2!3!4! (mod 12)

Y nuestro problema se reduce a encontrar el residuo al dividir 1!2!3!4! 33 entre 12,


que es 9.

Un método efectivo para calcular ab (mod n): “Método de las potencias de dos”
m
Calculamos las potencias a 2 (mod n) sucesivamente y en orden ascendiente. Después
descomponemos nuestra potencia ab (mod n) como producto de las anteriores. Como
tantas cosas en Teoría de Números y en general en matemáticas, lo mejor es observar un
ejemplo práctico:

Ejemplo.
Calcular 3172 (mod 191) .

Solución.
32  9 (mod 191)   2
34  32  81 (mod 191)
 2
38  34  6561  67 (mod 191) 316  3   4489  96 (mod 191)
8 2

332  3   9216  48 (mod 191)


16 2
364  3   2304  12 (mod 191)
32 2

 3   144 (mod 191)


2
3128 64

Y ahora, puesto que 172  128  32  8  4 ,


3172 (mod 191)  31283323834 (mod 191)  144  48  67  81 (mod 191)  170 (mod 191)
Podemos encontrar otros ejemplos de aplicación de este método en las soluciones de los
problemas #6.13 , #11.1 y #6.15.

6.1 MF
Si n ! denota el producto de todos los números del 1 al n, ¿Cuál es el residuo de
1! 2! 3! ...  n!
al dividirlo entre 9?

6.2 MF
Encuentra un ejemplo que demuestre que a 2  b 2 (mod n) no implica a  b (mod n) .

6.3 F
Determina los residuos cuando 2 50 y 4165 son divididos entre 7.

6.4 F
Utilizando la teoría de congruencias, demuestra que 89 | 244  1 y 97 | 2 48  1 .

6.5 F
Determina el último dígito de 91003  7902  3801 .

6.6 MF
Demuestra que para todo n  IN , el número an  11n  2  122 n 1 es divisible entre 133.

6.7 MF
La cifra de las unidades de 2137 753 es:

(A) 1 (B) 3 (C) 5 (D) 7 (E) 9

AHSME 1961 #28

6.8 F
El dígito de las unidades de 3100171002131003 es:

(A) 1 (B) 3 (C) 5 (D) 7 (E) 9

AHSME 1983 #14

6.9 F
Demuestra que 186  1 (mod 7k ) para k  1,2,3.
6.10 MF
Demuestra que si n es impar, entonces n 2  1 (mod 8)

6.11 F
Determina el número de enteros n , 1  n  25 , tales que n2  3n  2 es divisible entre
6.

6.12 F
Demuestra que 2n  6  9n siempre es divisible entre 7, para todo entero positivo n.

6.13 M
Sea an  6  8 . Determina el residuo cuando a83 se divide entre 49.
n n

AIME 6.13 #6

6.14 F
Determina el residuo al dividir 9  99  999  ...  99
 9 entre 1000.
...
999 nueves

AIME I 2010 #2

6.15 F
Consideremos el esquema triangular de números 0 ,1, 2 , 3... a lo largo de los lados y con
números interiores obtenidos sumando los dos números superiores de la fila anterior.
Las filas 1 a 6 se muestran en el siguiente esquema:

Sea f (n) la suma de los números de la fila n. ¿Cuál es el residuo cuando dividimos
f (100) entre 100?

AHSME 1995 #27

6.16 M
Sea k  20082  22008. ¿Cuál es el dígito de las unidades de k 2  2k ?

(A) 0 (B) 2 (C) 4 (D) 6 (E) 8

AMC12A 2008 #15, AMC 10A 2008 #24


6.17 F
Determina el menor entero positivo n tal que n y 107n tienen las dos últimas cifras
iguales.

HMMT 2008 #2

6.18 M
Demuestra que si x, y, z son enteros cumpliendo x 2  y 2  3z 2 , entonces x  y  z  0 .

6.19 F
Determina los enteros n tales que n  2 divide n  18 .
2

PUMaC 2007/NT #B2

Congruencias con potencias y polinomios.


Congruencias con potencias.
Las propiedades estudiadas en el Tema 4 tienen unas aplicaciones muy importantes en
el estudio de las congruencias:

Dados dos enteros positivos d , k , con d | k , entonces:


a) a  b (mod n)  a k  b k (mod n)
b) a d  b d (mod n)  a k  b k (mod n)
c) a d  b d (mod n) y k / d es impar,  a k  b k (mod n)

Demostración.
a) Basta aplicar a  b | a k  b k para todo k.
b) Basta aplicar d | k  a d  b d | a k  b k .
c) a d  bd (mod n)  a d  bd  0 (mod n)  n | a d  bd (*)
Pero si k / d es impar tenemos a d  b d | a k  b k , luego
(*)  n | a k  bk  a k  bk  0 (mod n)  a k  bk (mod n)

Ejemplo.
Aprovechando que 641  24  54  5  27  1 , demostrar que 641 | 232  1 .

641  2 4  54  2 4  54  0 (mod 641)  54  2 4 (mod 641)


 54 2 28  2 4 2 28 (mod 641)  54 2 28  232 (mod 641)

 5  27 
4
 232 (mod 641)  641  1  232 (mod 641)
4

  1  232 (mod 641)  1  232 (mod 641)  232  1  0 (mod 641)


4

 641 | 232  1
Teorema.
Dado un polinomio con coeficientes enteros p( x)  cm x m  cm1 x  ...  c1 x  c0 ,
entonces:
a  b (mod n)  p(a)  p(b) (mod n) .

Demostración.
a  b (mod n)  a k  b k (mod n)  ck a k  ck b k (mod n)
m m
 p(a)   ck a k  p(b)   ck b k (mod n)
k 0 k 0

6.20 F
Dado cualquier número positivo n, y sea S la suma de sus cifras, demuestra:
a) n  S es divisible entre 9.
b) n es divisible entre 9 si y solo si S es divisible entre 9.

En este problema justificamos el “criterio de divisibilidad del nueve”: Un número es


divisible entre nueve si y solo si las suma de sus cifras es divisible entre 9.

6.21 F
Demuestra el “criterio de divisibilidad del once”: Un número es divisible entre 11 si y
solo si la suma alternada de sus cifras es múltiplo de 11.

Proposición.
Dado un polinomio con coeficientes enteros p( x)  cm x m  cm1 x  ...  c1 x  c0 , diremos
que a es una solución de la congruencia p( x)  0 (mod n) si p(a)  0 (mod n) .
Si a es una solución de la congruencia p( x)  0 (mod n) y b  a (mod n) entonces b
también es una solución de la congruencia p( x)  0 (mod n) .

Demostración.
a  b (mod n)  0  p(a)  p(b) (mod n)  0  p(b) (mod n)
Teorema del inverso modular. Lema de Gauss.

Teorema. Teorema del inverso modular.


(a, b)  1 si y solo si existe un entero x tal que a x  1 (mod b) , y este valor es único
módulo b. A x le llamaremos inverso de a módulo b.

Demostración.
Aplicando el TDB, si (a, b)  1 existirán x, y tales que a x  b y  1 , y por tanto
a x  1  by  ax  1 (mod b) .
Supongamos que existen dos valores x, x' tales que a x  1 (mod b) y a x'  1 (mod b) .
Entonces
x'  1x'  a xx'  a x' x  1x  x (mod b)

Recíprocamente, supongamos que a x  1 (mod b) , es decir, a x 1  yb para cierto


entero y ,
y por tanto a x  yb  1  a x  ( y)b  1 . Aplicando el TDB llegamos a (a, b)  1 .

Corolario. Lema de Gauss.


Si a, b, c son enteros tales que a | bc y (a, b)  1 , entonces a | c .

Demostración.
(a, b)  1 , luego existirá un x tal que bx  1 (mod a) . Pero por otro lado,
a | bc  0  bc  xbc  cbx  c1  c (mod a)  a | c .

Corolario.
a) (a, b1 )  (a, b2 )  ...  (a, bn )  1   a , b1  b2  ...  bn   1
 
b) En particular, (a, b)  1  a , bk  1 para todo k  1 .

Demostración.
a) (a, b1 )  (a, b2 )  ...  (a, bn )  1, luego existirán x1 , ..., xn tales que xibi  1 (mod a) .
Multiplicando tenemos que x1x2 ...xn b1b2 ...bn   1  1  ...  1  1 (mod a) , y por tanto
 a , b1  b2  ...  bn   1 por el recíproco del Teorema del inverso modular.
b) Basta tomar en el apartado anterior b1  b2  ...  bk  b

Corolario.
a n | bn  a | b

Demostración.
Si a  0 o b  0 está claro que a | b . Supongamos que a, b  0 .
Sea d  (a, b)  a  du , b  dv con (u, v)  1 .
Luego a n | bn  du  | dv   d nu n | d nv n  u n | v n  u | v n
n n

Por el corolario anterior, (u, v)  1  (u, v n )  1 , y por tanto u  1 , y en consecuencia


a  d , y por lo tanto divide a b  dv .
Corolario.
Si a | c y b | c con (a, b)  1 , entonces ab | c

Demostración.
a | c  c  a d para cierto d .
b | ad 
  b | d por el Lema de Gauss, y por tanto a b | a d  c , como queríamos ver.
(a, b)  1

Teorema.
Sean a, b enteros positivos coprimos, tales que su producto es una potencia de grado n ,
es decir, ab  c n para cierto entero positivo c . Entonces a y b también son ambos
potencias de grado n .

Demostración.
Sea d  (a, c) , y escribimos a  du y c  dv para ciertos u, v coprimos.
a  du 
n
 dub  (dv) n  d nv n  ub  d n 1v n (*)
ab  c 

De la igualdad anterior se desprende que u | d n 1v n , pero por hipótesis,


u
(u, v)  1  (u, v n )  1 , y por tanto u | d n 1 . Así pues, v n  n 1 b , es decir, b | v n .
d
Pero de la igualdad (*) también se deduce que v | ub , y puesto que (u, v n )  1 ,
n

llegamos a v n | b , y finalmente:
b | vn 

b v
n

v | b
n

Sustituyendo en (*) se deduce u  d n 1  a  d n , con lo que se concluye la
demostración.

Problema resuelto.
Demuestra que el producto de tres números consecutivos nunca puede ser una potencia
perfecta.

Solución.
Supongamos que (n  1)n(n  1)  a k , para ciertos enteros a, k  1.
Entonces podemos escribir n(n2  1)  a k . Puesto que (n, n2  1)  1 podemos aplicar el
teorema anterior: n  bk , n2  1  c k para ciertos enteros b, c  1 .
  2
  k
 
n2  1  c k  1  n2  c k  bk  c k  b2  c k  b2  c b2( k 1)  ...  c k 1 
Lo cual es imposible pues b2  c  1 y b2( k 1)  ...  ck 1  k  1.
7 Introducción a las ecuaciones diofánticas.
Definición. Ecuación diofántica.
Una ecuación diofántica es aquella en la que únicamente son aceptables soluciones
enteras. En general, no existen métodos "mecánicos" para resolver ecuaciones
diofánticas y precisamente esto las hace muy interesantes para el planteamiento de
problemas.

Antes de empezar a trabajar con los métodos para resolver ecuaciones diofánticas
lineales, vamos a presentar una lista de problemas de ecuaciones diofánticas sin
armadura teórica previa, solo la aplicación del sentido común y los conceptos básicos
del álgebra y la divisibilidad.

7.1 M
El número de pares de enteros ( x, y) con 0  x  y tales que 1984  x  y es:

(A) 0 (B) 1 (C) 3 (D) 4 (E) 7

ASHME 1984 #28

7.2 M
El número n se escribe en base 14 como a b c , se escribe en base 15 como a c b y se
escribe en base 6 como a c a c , con a  0 . Determina el número n en base 10.

AIME I 2018 #2

7.3 F
Determina justificadamente todos los pares de números enteros ( x, y) que verifican la
ecuación x 2  y 4  2009 .

OME Fase Nacional 2009 #4

7.4 F
El número de triples a, b, c  de números enteros positivos que satisfacen el sistema
ab  bc  44

ac  bc  23

es
(A) 0 (B) 1 (C) 2 (D) 3 (E) 4

AHSME 1984 #20


7.5 F
Determina el número de 7-tuplas de números positivos (a, b, c, d , e, f , g ) que satisfacen
el siguiente sistema de ecuaciones:
abc  70

cde  71
efg  72

AIME II 2019 #3

7.6 M
Determina 3x 2 y 2 si x, y son enteros que satisfacen y 2  3x 2 y 2  30 x 2  517 .

AIME 1987 #5

7.7 M
4 2
Resuelve la siguiente ecuación diofántica  1
m n

ASHME 1993 #19

7.8 F
Encuentra todos los primos p y q que satisfacen la ecuación p  q  ( p  q)3

Rusia, 2001

7.9 M
Resuelve el siguiente sistema de ecuaciones, con a, b, c enteros.
 a  b  c  19

 ab  c  97

ASHME 1997 #28

7.10 F

Resuelve la ecuación con solución entera x 2  x  1  x2
1

ASHME 1985 #21

7.11 M
Determina todas las tercias de enteros positivos a , b , c tales que abc  a  b  c  1 .

México, 2010
7.12 D
Determina todas las soluciones enteras de la ecuación 2 x  1  3 y

7.13 F
Describir todas las soluciones enteras positivas m, n  de la ecuación
8m  7  n2

y dar el primer valor de m (si existe) mayor que 1959.

OME 2017 Fase Local #1

7.14 D
 
Calcular todos los pares de enteros x, y  tales que 3423 x 2  y 2  x3 y 3 .

OME Fase nacional 2019 #4


Ternas pitagóricas.

Definición. Terna Pitagórica.


Definimos por Terna Pitagórica las ternas ( x, y, z ) de números enteros solución de la
ecuación diofántica
x2  y 2  z 2

Teorema. Caracterización de las ternas pitagóricas.


Las ternas pitagóricas son todas las ternas de enteros que se pueden escribir de la forma


 x  d m2  n2   x  2dmn

 y  2dmn o bien


y  d m  n
2 2


 z  d m2  n2
  
 z  d m2  n2
 
Donde m  n  0 son números coprimos con paridad diferente y d  0 .

Demostración.
Que las ternas de enteros anteriores son pitagóricas es fácil de comprobar:
 
x 2  y 2  z 2  d 2 m2  n2  2dmn  d 2 m2  n2
2 2
 
2

Simplificando:
m     
 n2  2mn  m2  n2  m2  n2  4m2n2  m2  n2 , que es una
2 2 2 2 2
 
2

aplicación directa de la igualdad (a  b)2  4ab  (a  b)2 .

Veamos que si una terna es pitagórica, necesariamente debe ser como las anteriores.
Antes de nada vemos que podemos reducir nuestro problema a ternas de valores
coprimos dos a dos:
Sea ( x, y, z ) una solución entera de la ecuación x 2  y 2  z 2 .
Sea d  ( x, y) , y por tanto x  da , y  db para ciertos enteros a, b cumpliendo
(a, b)  1 .
Luego
 
z 2  x 2  y 2  da   db  d 2 a 2  b2  d 2 | z  d | z  z  dc ,
2 2

con lo que llegamos a


 
d 2 a 2  b2  (dc)2  d 2c 2  a 2  b2  c 2

Y, puesto que a y b son coprimos, a, b, c deben ser coprimos dos a dos.

Así pues, queremos encontrar las soluciones (a, b, c) de la ecuación a 2  b2  c 2 , con


a, b, c coprimos dos a dos.
En primer lugar, vemos que c debe ser impar. En efecto, puesto que a y b son
coprimos, no pueden ser ambos pares, pero tampoco ambos impares, pues en ese caso
c 2  a 2  b2  2 (mod 4)

Y todo cuadrado perfecto debe ser 0 o 1 módulo 4.


Así pues, a y b deben tener paridad diferente. Supongamos que a impar y b par.

Entonces la ecuación a 2  b2  c 2 se puede escribir como


ca ca
2
b
  
2 2 2

ca ca ca ca


Teniendo en cuenta que  c,   a , (a, c)  1 , llegamos a
2 2 2 2
 ca ca 
 ,  1
 2 2 

ca ca
Y por tanto y son cuadrados perfectos, es decir:
2 2

ca ca
 n2 y  m 2 para ciertos enteros n y m.
2 2

Se cumple claramente que m  n y son enteros coprimos.


También se cumple que m y n tienen diferente paridad, puesto que c  m2  n2 es
impar.
Y se cumple b  2mn , x  d (m2  n2 ) , y  2dmn , z  d (m2  n2 )

Tal y como queríamos ver.

Observación.
Las ternas pitagóricas son un caso particular de la llamada Ecuación de Fermat:

xn  y n  z n

Que fue resuelta por Wiles en 1994, más de 350 años después de que este problema
fuera propuesto, demostrando que para n  2 no existen soluciones que no sean la
trivial x  y  z  0 .

7.15 D
Calcula todos los números enteros a, b y c tales que a 2  2b2  3c 2

OME 2011 Fase local, Segunda sesión #4


8 Ecuaciones diofánticas lineales.
Ecuaciones diofánticas lineales.
Son las que tienen la forma a x  b y  c , con a, b, c enteros, y a, b no ambos cero.

Por ejemplo, la ecuación 3x  6 y  18 tiene infinitas soluciones:


x  4 , y  1  3  4  6  1  18
x  6 , y  6  3  (6)  6  6  18
x  10 , y  2  3  10  6  (2)  18
En general, cualquier valor x  2k , y  3  k  3(2k )  6(3  k )  6k  18  3k  18

Sin embargo, la ecuación 2 x  10 y  17 no tiene solución: Para cualquier x, y , la parte


izquierda de la ecuación será par, mientras que la parte derecha es un impar.

Teorema fundamental de las ecuaciones diofánticas lineales.


Una ecuación diofántica lineal a x  b y  c tendrá solución si y solo si (a, b) | c .

Si  x0 , y0  es una solución particular de esta ecuación, entonces todas las soluciones


son de la forma
b a
x  x0    k , y  y0    k para cualquier k entero.
d  d 

Ejemplo.
Resuelve la ecuación diofántica 172 x  20 y  1000

Solución.
d  (172,20)  4 , y 4 | 1000 , luego esta ecuación tiene solución.

Probando números vemos que 172  5  20  7  1000 , luego x0  5 , y0  7 es una


solución particular de la ecuación.
Por el Teorema anterior, las soluciones de esta ecuación son todas las parejas de la
forma
 20   172 
x  5    k  5  5 k  5(1  k ) , y  7    k  7  43 k , con k  Z
 4  4 
o equivalentemente, tomando q  1  k ,
x  5q , y  7  43(q  1)  7  43q  43  50  43q , con q  Z .

8.1 F
Resuelve la ecuación diofántica 7 x  9 y  3
8.2 F
Un cliente compra una docena de piezas de fruta, manzanas y naranjas, por 1.32€. Si
una manzana cuesta 3 céntimos más que una naranja, y se compraron más manzanas
que naranjas, cuantas piezas de cada fueron compradas?
Resolución de ecuaciones diofánticas lineales mediante el Algoritmo de Euclides.

El Algoritmo de Euclides para el cálculo del máximo común divisor de dos números
mediante sucesivas divisiones nos permite resolver ecuaciones diofánticas lineales. Lo
veremos con varios ejemplos:

Ejemplo 1.
Resuelve la ecuación diofántica 2173x  2491y  53

Solución.
Aplicamos el algoritmo de Euclides:
2491  1  2173  318
2173  6  318  265 
  (2173,2491)  53 , y claramente 53 | 53 , luego existe
318  1  265  53 
265  5  53  0 
solución.

Deshaciendo los pasos del algoritmo de Euclides:


53  318  1  265 

265  2173  6  318   53  318  1  265  2491  1  2173  1  2173  6  318 
318  2491  1  2173
 2491  1  2173  2173  6  2491  1  2173 
 2491  1  2173  2173  6  2491  6  2173 
 7  2491  8  2173

Luego x  8 , y  7 es una solución de la ecuación diofántica del enunciado.

El conjunto de soluciones de la ecuación serán las parejas de la forma:

2491 2173
x  8  k  8  47k , y  7  k  7  41k
53 53

8.3 F
Resuelve la ecuación diofántica 858x  253 y  33 mediante el algoritmo de Euclides.

8.4 F
Resuelve la ecuación diofántica 258x  147 y  369

8.5 F
Resuelve la ecuación diofántica 60 x  33 y  9
8.6 F
Las medidas (en grados) de los ángulos interiores de un hexágono convexo forman una
sucesión aritmética de enteros positivos. Sea mº la medida del mayor de los ángulos
interiores de este hexágono. El mayor valor posible de mº es

(A) 165º (B) 167º (C) 170º (D) 175º (E) 179º

AHSME 1991 #12

8.7 F
Determina un número que, cuando se divide entre 10, deja un residuo de 9, cuando se
divide entre 9 deja un residuo de 8, entre 8 el residuo es 7, y así sucesivamente, hasta
que, finalmente, cuando se divide entre 2, deja un residuo de 1.

ASHME 1951 #37


9 Congruencias lineales y sistemas de congruencias lineales.
Congruencia lineal.
Llamamos congruencia lineal a toda ecuación de la forma a x  b (mod n)
Diremos que el entero x0 satisface la congruencia lineal a x  b (mod n) cuando
a x0  b (mod n)

O equivalentemente: a x0  b (mod n)  a x0  ny0  b  a x0  n y0  b

Es decir, buscamos soluciones x0 , y0  de la ecuación lineal diofántica


a x0  n y0  b

Teorema.
La congruencia lineal a x  b (mod n) tiene solución si y solo si d | b , donde
d  (a, n) , en cuyo caso existen d soluciones diferentes (aquí se entiende diferentes
como mutuamente incongruentes), todas ellas de la forma

n n n n
xo , x1  x0  , x2  x0   2 , x3  x0   3 , ... , xd 1  x0   (d  1)
d d d d

Donde x0 es una solución particular de la ecuación.

Ejemplo.
Resuelve la congruencia 18x  30 (mod 42)

Solución.
d  (18,42)  6 , y 6 | 30 , luego la ecuación anterior tiene seis soluciones.
Por tanteo, vemos que 18  4  72  42  1  30  4 es una solución de la ecuación.
Luego las soluciones serán:
x0  4
x1  4  (42 / 6)  1  4  7  11 , efectivamente: 18  11  198  4  42  30
x2  4  (46 / 6)  2  4  14  18 , efectivamente: 18  18  324  7  42  30
x3  4  (46 / 6)  3  4  21  25 , efectivamente: 18  25  450  10  42  30
x4  4  (46 / 6)  4  4  28  32 , efectivamente: 18  32  576  13  42  30
x5  4  (46 / 6)  5  4  35  39 , efectivamente: 18  39  630  16  42  30

Las soluciones son: 4 ,11,18 , 25 , 32 , 39 

9.1 F
Resuelve la congruencia 9 x  21 (mod 30)

9.2 F
Resuelve la congruencia lineal 3x  7 (mod 10)
Inversos modulares.
Diremos que a y b son inversos módulo n si a b  1 (mod n) , o equivalentemente,
diremos que b es el inverso de a módulo n.
La congruencia lineal a x  1 (mod n) tiene solución si y solo si (a, n) | 1 , es decir,
cuando
(a, n)  1 , así pues, existirá el inverso multiplicativo de a si y solo si (a, n)  1 , y será
único modulo n.

Ejemplos.
El inverso de 3 módulo 4 es 3 porque 3  3  9  1 (mod 4) .
El inverso de 3 módulo 5 es 2 porque 3  2  6  1 (mod 5) .

9.3 F
Determina el inverso de 9 módulo 82.
Sistemas de congruencias lineales (caso particular).

Queremos resolver ahora un sistema de congruencias lineales:


a1 x  b1 mod m1 
a x  b mod m 
 2 2 2

...
ar x  br mod mr 

En donde vamos a suponer que los módulos mi son todos coprimos entre ellos.
Evidentemente, el sistema tendrá solución cuando cada ecuación la tenga
individualmente, y por tanto
d k | bk , donde d k  ak , mk  para todo 1  k  r

Resolución de sistemas de congruencias lineales mediante el método interactivo.

Ejemplo.
 x  3 (mod 4)

Resuelve el sistema  x  1 (mod 5)
 x  2 (mod 3)

Solución.
(Más adelante, mediante el Teorema chino del residuo, se verá que la solución existe y
es única mod 60, pues (4,5)  (5,3)  (4,3) )

x  3 (mod 4)  x  4a  3 
x  1 (mod 5)  4a  3  1 (mod 5)  4a  2 (mod 5)  2  5 (mod 5)  3 (mod 5)
4a  3 (mod 5)  4  4a  4  3 (mod 5)  16a  12 (mod 5)  a  2 (mod 5) 
a  5b  2  x  4a  3  45b  2  3  20b  11

Luego:
x  2 (mod 3)  20b  11  2 (mod 3)  20b  9 (mod 3)  0 (mod 3)
 b  0 (mod 3)  b  3c

Finalmente:
x  20b  11  20  3c  11  11  60c (mod 60) . En efecto:
11  4  2  3 , 11  5  2  1 , 11  3  3  2
Teorema chino del residuo.
Sean m1 , m2 , ..., mr enteros positivos tales que mi , m j   1 si i  j . Entonces el sistema
de congruencias lineales
a1 x  b1 mod m1 
a x  b mod m 
 2 2 2

...
ar x  br mod mr 

Tiene una única solución (módulo el entero m1  m2  ...  mr ).

Y se obtiene siguiendo los siguientes pasos:


Paso 1: Sea N  m1  m2  ...  mr ,
Paso 2: Sean N1  N / m1 , N2  N / m2 , ... , Nr  N / mr .
Paso 3: Resolver las congruencias lineales:
N1 y1  1 mod m1  , N 2 y2  1 mod m2  , ... , N r yr  1 mod mr 

Paso 4: x  N1 y1 b1  N 2 y2 b2  ...  N r yr br (mod N ) es la única solución del


sistema.

Ejemplo: El problema de Sun-Tsu.


El Teorema chino del residuo debe su nombre en honor al siguiente problema del siglo I
DC: Determina un número cuyos residuos son 2, 3 y 2 al dividirlo entre 3, 5 y 7,
respectivamente.

Nota: Este mismo problema aparece en las Introductio Arithmeticae del matemático
griego Nicómano de Gerasa, alrededor del 100 DC.

Solución.
Se trata de resolver el sistema de congruencias lineales
 x  2 (mod 3)

 x  3 (mod 5)
 x  2 (mod 7)

Paso 1: N  3  5  7  105
105 105 105
Paso 2: N1   35 , N 2   21 , N3   15
3 5 7
Paso 3: Las congruencias lineales 35 y1  1 (mod 3) , 21y2  1 (mod 5) y 15 y3  1 (mod 7)
tienen soluciones y1  2 , y2  1 y y3  1 .

Paso 4: x  35  2  2  21  1 3  15  1 2  233 será solución del sistema módulo 105, y


233  23 (mod105)

Y por tanto 23 es la única solución del sistema (módulo 105).


En efecto: 23  7  3  2 , 23  4  5  3 , 23  3  7  2
Ejemplo.
 x  2 (mod 4)
Resolver el sistema de congruencias 
 x  7 (mod 9)

Solución.
Claramente (4,9)  1 y por tanto el sistema tiene solución.
Paso 1: N  4  9  36
36 36
Paso 2: N1   9 , N2   4.
4 9
Paso 3: Resolvemos las ecuaciones 9 y1  1 (mod 4) y 4 y2  1 (mod 9)
Puesto que 9  1  2  4  1 , y 4  7  28  3  9  1 , tenemos que y1  1 , y2  7 son
soluciones.

Paso 4: La solución es x  9  1  2  4  7  7  214 (mod 36) , es decir, 34.

Efectivamente, 34  4  8  2 , y 34  9  3  7 ,
Sistemas de congruencias lineales (caso general).

Se pueden resolver sistemas de congruencias incluso cuando sus módulos no son


necesariamente coprimos. El criterio es similar al de las ecuaciones diofánticas lineales.

Teorema.
Dado el sistema
 x  a1 (mod m1 )

 x  a2 (mod m2 )

Si m1,m2  no es divisor de a1  a2 , el sistema no tiene solución.


En caso contrario, existe una única solución mod a1 , a2 

Observamos que el Teorema chino del residuo sería un caso particular de este teorema
cuando m1, m2   1 , pues entonces garantizamos que el sistema tenga solución, y
a1, a2   m1 m2 .
Ejemplo.
Resolver el sistema
 x  5 (mod12)

 x  11 (mod18)

Solución.
Puesto que (12,18)  6 , y 6 | (11 - 5) , existirá una única solución. La vamos a obtener
con el método interactivo.

x  5 (mod12)  x  12a  5
x  11 (mod18)  12a  5  11 (mod18)  12a  11  5 (mod18)  12a  6 (mod18)

Esta última congruencia se puede simplificar: 6 divide a 12 y a 6, y además


mcd (6,18)  6 , luego podemos simplificarla:

12a  6 (mod18)  2a  1 (mod 3)  a  2 (mod 3)  a  3k  2


x  12a  5  123k  2  5  36k  29 mod(36)

Donde hemos tenido en cuenta que 12,18  36

Efectivamente, 29  12  2  5 , y 29  18  1  11
Como calcular congruencias cuando el módulo no es primo.

Ejemplo.
Calcular 20182018 (mod 26) .

Solución.
Puesto que 26 no es primo, no podemos aplicar directamente el PTF. Puesto que
26  2  13 ,
vamos a calcular por separado 20182018 (mod 2) y 20182018 (mod 13) , y después
aplicaremos el Teorema Chino del Residuo para determinar el resultado del enunciado.

Está claro que 2018  0 (mod 2) y por tanto 20182018  02018  0 (mod 2) .

2018  13  155  3 , luego 13 | 2018 , y por tanto podemos aplicar el PTF:


201812  1 (mod13) .

Por otro lado, 2018  12  168  2 , luego:


 
20182018  201812168 2  201812 20182  1 20182  20182 (mod 13)
168 168

2018  13  155  3  2018  3 (mod13)  20182  32  9 (mod13)

De todo lo anterior tenemos:



2018  0 (mod 2)
2018



2018  9 (mod 13)
2018

Y aplicamos el Teorema Chino del Residuo:


N  2  13  26
N1  13
N2  2
No hace falta resolver la congruencia 13 y1  1(mod 2) pues b1  0 .
Resolvemos la congruencia 2 y2  1(mod13)  y2  7 ,
Luego 20182018  13  y1  0  2  7  9  126 (mod 26)  22 (mod 26)

Observación.
Para calcular potencias elevadas con módulos no primos disponemos de dos técnicas:
El “método de las potencias de dos” y el método que acabamos de ver: descomponer el
módulo y aplicar el Teorema Chino del Residuo. Es importante dominar estas dos
técnicas, pues son la clave para resolver muchísimos problemas de Aritmética. Se
propone resolver el siguiente problema mediante las dos técnicas anteriores:

9.4 F
Determina los dos últimos dígitos de 10321032.

HMMT 2009
Congruencias lineales que se resuelven mediante sistemas de congruencias lineales.

El siguiente resultado nos puede ser útil para resolver congruencias lineales:

a x  b (mod n m)  a x  b (mod n) y a x  b (mod m)

En efecto: a x  b (mod n m)  a x  k nm  b  km n  b  a x  b (mod n)

Existe un recíproco: a x  b (mod n) y a x  b (mod m)  a x  b mod n, m 

Ejemplo.
Resolver la congruencia lineal 17 x  9 (mod 276) .

Solución.
Puesto que 276  4  3  23 , la ecuación anterior es equivalente a resolver el sistema de
congruencias
17 x  9 (mod 3)  x  0 (mod 3)
 
17 x  9 (mod 4) o equivalentemente:  x  1 (mod 4)
17 x  9 (mod 23) 17 x  9 (mod 23)
 

x  0 (mod 3) equivale a decir que x  3a , luego sustituyendo en la segunda ecuación y


multiplicando por 3 ambos lados:
3a  1 (mod 4)  9a  3 (mod 4)  a  3 (mod 4)  a  4b  3  x  3a  12b  9

Sustituyendo en la tercera ecuación:


17 x  9 (mod 23)  1712b  9  9 (mod 23)  204b  153  9 (mod 23)
 204b  144 (mod 23)  20b  17 (mod 23)  3b  6 (mod 23)
 3b  6 (mod 23)  b  2 (mod 23)  b  23k  2
 x  12(23k  2)  9  276k  24  9  276k  33  x  33 (mod 276)

Efectivamente, 17  33  561  2  276  9


Congruencias no lineales.

Ejemplo.
Resuelve la ecuación x 2  1 (mod 144)

Solución.
Puesto que 144  16  9 , y (16,9)  1 , podemos descomponer la ecuación anterior en el
sistema no lineal
 x 2  1 (mod 16)

 2

 x  1 (mod 9)
x  1 (mod 16) tiene 4 soluciones: x  1 o  7 (mod 16)
2

x 2  1 (mod 9) tiene 2 soluciones: x  1 (mod 9)


Luego tenemos ocho alternativas:
i) x  1 (mod16) y x  1 (mod 9)
ii) x  1 (mod16) y x  1 (mod 9)
iii) x  1 (mod16) y x  1 (mod 9)
iv) x  1 (mod16) y x  1 (mod 9)
v) x  7 (mod16) y x  1 (mod 9)
vi) x  7 (mod16) y x  1 (mod 9)
vii) x  7 (mod16) y x  1 (mod 9)
viii) x  7 (mod16) y x  1 (mod 9)

Podemos ir resolviendo cada caso mediante el Teorema chino del residuo.


Independientemente del caso, (16,9)  1 , luego todos los ocho sistemas tienen solución.
Además: N1  9 , N2  16 , 9 y1  1 (mod16)  y1  9 , 16 y2  1 (mod 9)  y2  4
i) x  9  9  1  16  4  1  145 (mod144)  1 (mod144)
ii) x  9  9  1  16  4  (1)  17 (mod144)
iii) x  9  9  (1)  16  4  1  17 (mod144)
iv) x  9  9  (1)  16  4  (1)  145 (mod144)  1 (mod144)
v) x  9  9  7  16  4  1  631 (mod144)  55 (mod144)
vi) x  9  9  7  16  4  (1)  503 (mod144)  71 (mod144)
vii) x  9  9  (7)  16  4  1  503 (mod144)  73 (mod144)  71 (mod144)
viii) x  9  9  (7)  16  4  (1)  631 (mod144)  55 (mod144)

9.5 F
Calcula los tres últimos dígitos de 200511  200512  ...  20052006

Senior Hanoi Open MO 2006

9.6 F
Demostrar que si x es un número impar no divisible entre tres, entonces x 2  1 mod 24 .
10 El pequeño Teorema de Fermat.
Teorema. Pequeño Teorema de Fermat (PTF).
Si p es primo,
a) a p  a (mod p) para cualquier entero a .
b) Si p | a , entonces a p1  1 (mod p)

Nota: El recíproco no es cierto: a n1  1 (mod n) para cierto entero a 


 n primo. Esto
se estudiará detenidamente en una observación más adelante.

Demostración.
a) Aunque en el próximo tema veremos que este teorema es un caso particular de
aplicación de la función Phi de Euler, vamos a presentar aquí una demostración directa.

Caso 1: Si a  0 . a  0  a p  0  a y está claro que entonces a p  a (mod p) .

Caso 2: Si a  0 . Vamos a demostrarlo por inducción en a :


Si a  1 , a p  1p  1 y está claro que entonces a p  a (mod p) .
Supongamos cierto a p  a (mod p) , queremos ver que entonces es cierto para a  1 .
Aplicando el binomio de Newton:
a  1p  a p    a p 1  ...  
p p 
 a  1
1  p  1
 p  p p!
Pero p |   para todo 1  k  p (por la definición de    al ser p primo,
k  k  k!( p  k )!
estará en el numerador, pero no en el denominador) , y por tanto
a  1p  a p  1 (mod p) .
Finalmente, aplicando la hipótesis de inducción:
a  1p  a p  1 (mod p)  a  1 (mod p) ,
tal y como queríamos ver.

Caso 3: Si a  0 . Si p  2 , entonces
a2  (a)2  a (mod 2)  2 | a 2  a  2 | a 2  a  2a  a 2  a  a 2  a (mod 2)
Si p  2 , entonces p es impar, y por tanto: a p  (a) p  (a) (mod p)  a (mod p)
En donde hemos aplicado el “Caso 1” pues  a es positivo.

 
b) Aplicando el apartado anterior, a p  a (mod p)  p | a p  a  a a p 1  1 .
Aplicando el Lema de Euclides, puesto que, por hipótesis, p | a , deducimos que
p | a p 1  1 , o equivalentemente, a p 1  1  0 mod p   a p 1  1 mod p 
El PTF se puede aplicar al cálculo de congruencias con potencias de números, como en
el siguiente ejemplo:

Ejemplo 1.
Demostrar que 538  4 (mod 11) .

Solución.
Aplicamos el PTF para garantizar que 510  1 (mod 11) , luego:
  3
538  5310  8  510 58  1358 (mod11)  58 (mod11)  (*)

Por otro lado 52  25  3 (mod11) , y por tanto:


  4
(*)  52 (mod11)  34 (mod11)  81 (mod11)  4 (mod11)

Donde hemos aplicado que 81  7  11  4

Ejemplo 2.
Calcular 7121 (mod 13)

Solución.
Puesto que 13 | 7 podemos aplicar el PTF para garantizar que 712  1 (mod13) .
 
Puesto que 121  12  10  1 7121  712101  712
10
 7  110  7  7 (mod13)

El PTF se puede utilizar también para testear si un número n es primo o no. Si


encontramos un entero a tal que a n  a (mod n) , entonces seguro que n no es primo
(pues en caso contrario contradeciría el PFT). Veamos esto en el siguiente ejemplo:

Ejemplo.
Demostrar que 117 no es primo.

Solución.
Tomamos a  2 . Sabemos que 27  128  11 (mod117) , y que 121  4 (mod117) ,
luego:
2 
7 16
  8
 
8
 1116  112  1218  48  22  216 (mod117)
2117  2716 5 2  2  2 2 (mod117)  2 (mod117)
7 16 5 16 5 21

 2   11  121  11  4  11  44 (mod117)
3
Pero 221 7 3

Con lo que, finalmente llegamos a 2117  44  2 (mod117) , y por tanto 117 debe ser un
número compuesto (de hecho: 117  13  9 ).
Proposición. Descomposición de módulos.
Si p y q son primos diferentes tales que a p  a (mod q) y a q  a (mod p) , entonces:
a p q  a (mod p q)

a 
q p
 a q (mod p) por el Corolario al PTF, y a q  a (mod p) por hipótesis, luego:
Luego a p q  a q p
 a q (mod p)  a (mod p) , es decir: p | a p q  a

Con un razonamiento similar llegamos a q | a p q  a , y por tanto:


p q | a p q  a , o equivalentemente: a p q  a  0 (mod p q)

Ejemplo.
Demostrar que 2340  1 (mod 341) .

Solución.
Aquí 341  11  31 , y por otra parte, 210  1024  31  33  1, luego 210  1 mod(31) , y por
tanto:
211  2  210  2  1  2 (mod 31)

Por otro lado, 210  1024  11  93  1  210  1 (mod11) , y por tanto:


  3
  3
231  2 210  2 210  2  13  2 (mod11)

Y aplicando la proposición anterior: 2341  21131  2 (mod 341) , y cancelando un factor 2


llegamos al resultado deseado: 2340  1 (mod 341)

Observación. Los números pseudoprimos y el recíproco del PTF.


El PTF dice que n primo  2n  2 (mod n) , y acabamos de ver que 2341  2 (mod 341)
y sin embargo 341  11  31 no es primo, es decir un contraejemplo para el recíproco del
PTF.

Este dato es curioso porque n  341 es el primer compuesto tal que 2n  2 (mod n) , es
decir, sirve como contraejemplo del recíproco del PTF, es decir:

n primo  2n  2 (mod n) para 2  n  340

Esta es la razón por la que los matemáticos chinos antiguos creyeron, equivocadamente,
que 2n  2 (mod n) caracterizaba los números primos.

Los números n tales que 2n  2 (mod n) , es decir n | 2n  2 se denominan


pseudoprimos. Hay infinitos pseudoprimos y los más pequeños son 341, 561, 645 y
1105.
Aplicación del PTF a la resolución de congruencias no lineales.

Ejemplo 1.
Determina una solución de la congruencia x103  4 (mod 11)

Solución.
Puesto que, aplicando el PTF, x10  1 (mod11)
  3
x103  x1010 3  x10 x3  x3 (mod11)
Luego hemos reducido nuestro problema a resolver la congruencia x3  x (mod11)

Probando valores x  1 , x  2 , x  3 ,... llegamos a 53  4 (mod11) , y por tanto:


x  5 (mod11)

Ejemplo 2.
Determina una solución de la congruencia x86  6 (mod 29) .

Solución.
Aplicando el PTF, sabemos que x 28  1 (mod 29) para todo x .
 3
Luego x86  x328 2  x 28 x 2  x 2 (mod 29) , luego hemos reducido nuestro problema a
resolver la congruencia x 2  6 (mod 29) .
Vamos probando valores x  1 , x  2 , x  3 ,... hasta llegar a
x  8  x  64  29  2  6  6 (mod 29)
Luego una solución es x  8 (mod 29)

Nota: Existe otra solución: x  21  x 2  441  29  15  6  6 (mod 29)


Para encontrar estas soluciones se puede hacer el siguiente planteamiento:
6  64 (mod 29) , y por tanto, la ecuación x 2  6 (mod 29) es equivalente a
x 2  64 (mod 29) ,
y ahora:
x  8
x 2  64 (mod 29)  x 2  64  0 (mod 29)  x  8x  8  0 (mod 29)  
 x  8
y finalmente:  8  21 (mod 29)

10.1 F
Aplicando el PTF, determina:
a) 331 (mod 7) b) 235 (mod 7) c) 128129 (mod 17)

10.2 F
Dividimos el número 21000 entre 13. ¿Cuál es el residuo?

AHSME 1972 #31


10.3 F
Utilizando el PTF, demuestra que 17 divide a 11104  1

10.4 F
Demuestra que si (a,35)  1, entonces a12  1 (mod 35)

10.5 F
Sea a1  4 , an  4an1 , n  1 . Determina el residuo cuando a100 se divide entre 7.

10.6 F
Demuestra que, si (a,42)  1, entonces 168  3  7  8 | a6  1 .

10.7 F
Determina 220  330  440  550  660 (mod 7)

10.8 D
En los años 60, tres matemáticos americanos demostraron que una de las conjeturas de
Euler era falsa al encontrar un entero positivo n tal que

1335  1105  845  275  n5

Determina n.

AIME 1989 #9

Nota: Se presentan dos soluciones, pero ninguna de las dos es completa: Son argumentos que justifican
que un cierto n es el mejor candidato posible.

10.9 F
Determina los números primos p para los cuales 29 p  1 es múltiplo de p
11 La función Phi de Euler.
Definición. La función Phi de Euler.
Dado un número natural n  1 ,  (n) indica el número de números naturales menores
que n y coprimos con n.

Por ejemplo,  (30)  8 porque el número de naturales coprimos con 30 son 1, 7, 11, 13,
17, 19, 23 y 29. De la misma manera vemos que
 (1)  1,  (2)  1,  (3)  2,  (4)  2,  (5)  4,
 (6)  2,  (7)  6...
Observamos que  (1)  1 porque mcd (1,1)  1 , sin embargo, si n  1 , mcd (n, n)  n  1 .
Observamos p es primo si y solo si  ( p)  n  1 .

Con Mathematica:

Cálculo explícito de la función Phi de Euler.


Si la descomposición en factores primos de n es n  p1k1 p2k 2 ... prk r , entonces:
 1
 (n)   p1k  p1k 1  p2k  p2k ... p 
r
1 1 2 2 1 kr
r  prk r 1  n   1  
i 1  pi 

Ejemplo 1.
Calcular  (360)

Solución.
 1  1  1 
360  23325   (360)  3601  1  1    96
 2  3  5 

Ejemplo 2.
Calcular  (1001)

Solución.
 1  1  1
1001  7  11  13   (1001)  10011  1  1    720
 7  11  13 
Propiedad de la función Phi de Euler.
Fijemos un número n y un divisor suyo: Por ejemplo, n  30 y d  5 .
Estudiemos el conjunto Sd   m  n , (m, n)  d . En la siguiente tabla vemos que
S5   5 , 20 

1 m  n (m,30) (m,30)  5  (6)  2


1 1
2 2
3 3
4 2
5 5 5  5 1 1
6 6
7 1
8 2
9 3
10 10 10  5  2 2
11 1
12 6
13 1
14 2
15 15 15  5  3 3
16 2
17 1
18 6
19 1
20 10 20  5  4 4
21 3
22 2
23 1
24 6
25 5 5  55 5
26 2
27 3
28 2
29 1
30 30 30  5  6 6

Consideremos ahora n / d  6 y  (6) . El conjunto de números menores que 6 y


coprimos con 6 es  1 , 5 , luego  (6)  2 .
Vemos que existe una biyección perfecta entre S5 y los coprimos de 6, y por tanto
Sd   (n / d )
Por otro lado, todo número m  n pertenece a algún S d con d | n , y por tanto:
n   S d   ( n / d )
d |n d |n

d
Existe también una biyección d | n  | n y por tanto llegamos a la siguiente
n
propiedad:
n    (d )
d |n
Teorema de Euler.
a ( n)  1 (mod n) si (a, n)  1

Nota histórica.
La primera demostración del PTF , a p 1  1 (mod p) si p | a , fue dada por Euler en
1736. El propio Euler presentó en 1760 este teorema, que es una generalización del PTF
porque si n es primo entonces  (n)  p  1 , y (a, p)  1  p | a .

Observación.
La condición (a, n)  1 es necesaria, pues si (a, n)  1 , ya vimos en el capítulo 8 que la
congruencia a x  1 (mod n) no tiene solución, y por tanto no puede existir ningún k tal
que a k  1 (mod n) , pues en ese caso x  a k 1 sería solución de la congruencia
a x  1 (mod n) .

Ejemplo.
Tomando n  30 y a  11 ,  (30)  8 y 118  1 (mod 30)
En efecto, 112  121  1 (mod 30)  118  112   4
 14  1 (mod 30)

Orden de un entero.
El Teorema de Euler indica que si (a, n)  1 , la secuencia
 
a , a 2 , a3 , a 4 ,...
siempre alcanza el 1 (y por lo tanto se vuelve periódica), y lo alcanza en a (n ) .
Naturalmente,  (n) no es necesariamente el primer número k para el cual a k  1 . El
menor exponente k para el cual a k  1 se denomina orden de a (módulo n) y se
estudiará en el apartado siguiente.

El Teorema de Euler y el PTF nos permiten reducir potencias muy grandes módulo n.

Ejemplo.
Determina los dos últimos dígitos de la expresión decimal de 3256 .

Solución.
Está claro que este problema implica estudiar 3256 (mod100) , y aquí nos puede ayudar
el Teorema de Euler:
   1  1 
 (100)   2252  1001  1    40 , y puesto que (3,100)  1 , podemos aplicar el
 2  5 
Teorema de Euler: 3  1 (mod100)
40

  6
Luego 3256  340616 (mod100)  340 316 (mod100)  316 (mod100) .
38  6561  61 mod(100)  316  3838  61  61  3721  21 mod(100)
Por lo tanto, el número 3256 acaba en "21".
11.1 F
¿Para cuántos enteros i , cumpliendo 1  i  1000 , existe un entero j , cumpliendo
1  j  1000 , tal que i es un divisor de 2 j  1 ?

11.2 M
Determina los ocho últimos dígitos de la expansión binaria de 271986 .

11.3 D
1
... 2
20072006
Determina los tres últimos dígitos de 2008
1
... 2
20072006
Nota: 2008 se define recursivamente:
a1  1 , a2  2a1 , a3  3a2 , ... , a2008  2008a2007

PuMAC (Princeton University Mathematics Competition)

11.4 D
xx
Definimos f ( x)  x x . Determina los dos últimos dígitos de
f (17)  f (18)  f (19)  f (20)

PuMAC 2008

Nota: f (x) se define recursivamente: f1 ( x)  x , f 2 ( x)  x f1 ( x ) ,..., f ( x)  f 4 ( x)  x f3 ( x ) .

Indicación: Si (a, n)  1 , entonces ab  ab mod ( n) (mod n) y podemos reducir el cálculo


a determinar b mod  (n) .

11.5 F
Encontrar las tres últimas cifras del número 7 2014

OME 2014 Fase local, primera sesión #2


12 Orden de un entero.
Definición. Orden de un entero.
Si a es cualquier número entero y n es un número entero n  1 , (a, n)  1 , definimos el
orden de a módulo n, ordn (a) , como el entero positivo más pequeño d tal que
a d  1 (mod n)

Ejemplos:
31  3, 32  2 , 33  6 , 34  4 , 35  5 , 36  1 (mod 7)  ord7 (3)  6 .
ord5 (2)  4 pues 21  2 , 22  4 , 23  3 , 24  16  1 (mod 5) .
ord12 (5)  2 pues 51  5 , 52  25  2  12  1  1 (mod12) .

Con Mathematica:

Observaciones:
No todos los números tienen un definido un orden. Por ejemplo, si n | a , entonces
a k  0 (mod n) para todo k .

La siguiente proposición nos indica cuando un entero tendrá un orden asociado.

Proposición.
Si (a, n)  1 , entonces existe un k  n tal que a k  1 (mod n) .

Demostración.
 
Consideremos el conjunto a, a 2 , a3 , ..., a n1 mod n . Consta de n  1 números y solo
existen n residuos mod n , luego por el Principio de las Casillas, dos de estas potencias
deben ser iguales módulo n. Es decir, existirán 1  s  t  n  1 tales que
a s  a t (mod n) .
Ahora, 1  t  s  n , y por tanto:
a s  at (mod n)  at s a s  at s at (mod n)  at  at s at (mod n)
Puesto que (a, n)  1  (at , n)  1 , podemos cancelar el factor a t , con lo que llegamos a
1  a ts (mod n) , tal y como queríamos ver.
Corolario.
Dado un entero n  1 , a  Z tendrá orden módulo n si y solo si (a, n)  1 .

Demostración.
Si (a, n)  1 , por la proposición anterior existirá un 1  k  n tal que a k  1 (mod n) ,
luego por el Principio de la buena ordenación, existirá un mínimo k cumpliendo tal
condición, es decir, el número a tendrá un orden asociado.

Supongamos ahora que existe un entero positivo m tal que a m  1 (mod n) . Entonces
existirá un entero s tal que
a m  sn  1  a  a m 1  sn  1

Esta última expresión es una combinación lineal de a y n , luego


(a, n) | a  a m 1  sn  1  (a, n)  1
Teorema.
Si (a, n)  1 , entonces, para cualquier entero t, at  1 (mod n)  ordn (a) | t .

Ejemplo.
Determina todos los enteros positivos n tales que 2n  1 sea divisible entre 7.

IMO 1964
Solución:
7 | 2n  1  2n  1  0 (mod 7)  2n  1 (mod 7)
Puesto que (2,7)  1 , podemos aplicar el Teorema anterior:
2n  1 (mod 7)  ord7 (2) | n
21  2 , 22  4 , 23  8  1 (mod 7)  ord7 (2)  3
Luego, finalmente, 7 | 2n  1  3 | n , es decir, para todos los múltiplos de 3.

Corolario 1.
Dados p primo y a entero con p | a , entonces ordn (a) | p  1

Demostración. Puesto que p es primo y p | a , podemos aplicar el PTF para garantizar


que a p 1  1 (mod p) , y aplicar ahora el Teorema anterior.

Ejemplo.
Determinar ord11(8) .

Solución.
Puesto que 11 | 8 , aplicar el corolario anterior para asegurar que ord11(8) | 10 .
ord11(8)  2 no puede ser pues 82  64  9  1 (mod11) .
ord11(8)  5 tampoco puede ser:
82  9 (mod11) y 83  512  6 (mod11) , luego
85  8283  9  6  54 (mod11)  10 (mod11)
Luego solo nos queda ord11(5)  10 .
Propiedad de la secuencia de potencias de un número.
Dados a y n cumpliendo (a, n)  1 , sea k  ordn (a) .
En la secuencia

a , a 2 , a3 , a 4 ,..., a k  1 
no hay números repetidos (módulo n), pues si ai  a j con 1  i  j  k , entonces
podemos cancelar términos pues (a, n)  1 , y deducir que a j i  1 (mod n) , con
j  i  k contradiciendo la definición de orden.

En general:
a j  1 (mod n)  ordn (a) | j

En efecto, por el Algoritmo de la división, j  k  q  r con 0  r  k , pero entonces:


  q
a j  a kq  r  a kq a r  a k a r  1q a r  a r (mod n) . Si 0  r  k se contradeciría la
definición de k como índice de a, por lo tanto r  0 y j es un múltiplo del orden.

En particular, podemos aplicar este hecho a la función Phi de Euler:

ordn (a) |  (n)

Ejemplo de aplicación.
Determinar ord13 (2)

Solución.
 (13)  12  ord13(2)  1, 2 , 3, 4 , 6 ,12 
21  2 , 22  4 , 23  8 , 24  16  3 (mod 13) , 26  32  12 (mod 13) , luego solo puede
ser ord13 (2)  12 .
Problema modelo.
Determina el menor factor primo impar de 20198  1

AIME I 2019 #14

Solución.
Buscamos el menor número primo p tal que p | 20198  1
p | 20198  1 
20198  1  0 (mod p) 
20198  1 (mod p)

Entonces, elevando al cuadrado ambos lados, 201916  1 (mod p)

Pero 201916  1 (mod p)  ord p (2019)  1, 2 , 4 , 8 ,16 

Sin embargo, ord p (2019)  1, 2 , 4 , 8   20198  1 (mod p) y no -1, como queríamos,
luego deducimos que ord p (2019)  16 .

Puesto que ord p (2019) |  (p) ,  (p) será un múltiplo de 16.


 1
Puesto que por hipótesis p es primo,  ( p)  p  1    p  1
 p
Y por tanto p  1 (mod16) . Los dos primeros primos que cumplen p  1 (mod16) son
17 y 97.
Sin embargo, 20198  1 (mod17) , pero 20198  1 (mod 97) , luego la solución es 97.

Fuente de esta solución: artofproblemsolving.com


13 Números factoriales.
Función suelo, o parte entera.
Definimos la parte entera de un número real x , y la denotaremos por x  , como el
mayor entero n menor o igual que x . Por ejemplo, 2.4  2 , 1.99  1 ,  3.15  4

El tema PA/7 está dedicado íntegramente a esta función. Se recomienda practicar con
ella realizando algunos de los problemas que allí se presentan

13.1 M
Calcula la suma
 20   21   22   21000 

3 3 3   ...   3 
       

All Russian MO 2000

13.2 F
¿Cuál es la probabilidad de que un entero del conjunto  1 , 2 , 3 , ... , 100  pueda ser
divisible entre 2 y no divisible entre 3?

AMC 10A 2003 #15

La fórmula de Polignac.
Antes de presentar la fórmula de Polignac, propondremos un par de ejemplos para
entender por qué aparecen los elementos que la constituyen.

Problema.
¿Cuál será el exponente del factor 5 en la factorización en números primos de 1005! ?

Solución.
1005! 1  2  3...  1004  1005

De todos los factores de la derecha, uno de cada cinco es múltiplo de 5, luego habrán
1005 5
0 201
201 múltiplos de 5, que contribuirán en una unidad al exponente de 5.

De los 201 anteriores, uno de cada cinco será múltiplo de 25, luego habrán
201 5
1 40
40 múltiplos de 25, que contribuirán con una unidad más al exponente de 5.

De los 40 anteriores, uno de cada cinco será múltiplo de 53 , luego habrán


40 5
0 8
3
8 múltiplos de 5 , que contribuirán con una unidad más al exponente de 5.
De los 8 anteriores, uno de cada cinco será múltiplo de 54 , luego habrá
8 5
3 1
4
un único múltiplo de 5 , que contribuirá con una unidad más al exponente de 5.

Está claro que no existirán múltiplos de 55 ,56 ,...

Luego el exponente de 5 será 201  40  8  1  250

Fórmula de Polignac.
Estos mismos cálculos los podríamos haber realizado con cualquier número primo y con
cualquier factorial. Para expresarlos en una única fórmula con el lenguaje algebraico
observamos lo siguiente:

1005 / 5  201 , 1005 / 5   40 , 1005 / 5   8 , 1005 / 5   1


2 3 4

Luego la suma anterior se puede expresar mediante la función “parte entera”,


justificando la denominada fórmula de Polignac:

La mayor potencia de un número primo p que divide a n! viene dado por



n 
 p k 
k 1  

Observación.
Esta serie es en realidad una suma finita, pues para k suficientemente altos se cumple
n
n  p k y por tanto  i   0 para todo i  k .
p 

Segundo ejemplo.
¿Cuál será el exponente del factor 2 en la factorización en números primos de 1005! ?

Solución.
En 1005! uno de cada dos factores es par, luego
1005 2
1 502
hay 502 números pares, de los cuales, uno de cada dos será múltiplo de 2 2 :
502 2
0 251
De estos 251, la mitad serán múltiplos de 23 :
251 2
1 125
De estos 125, la mitad serán múltiplos de 24 :
125 2
1 62
De estos 62, la mitad serán múltiplos de 25 :
62 2
0 31
6
De estos 31, la mitad serán múltiplos de 2 :
31 2
1 15
7
De estos 15, la mitad serán múltiplos de 2 :
15 2
1 7
8
De estos 7, la mitad serán múltiplos de 2 :
7 2
1 3
9
De estos 3, la mitad serán múltiplos de 2 :
3 2
1 1
10 11 12
Y ya no hay múltiplos de 2 , 2 , 2 ...

El exponente de 2 en la factorización será 502+251+125+62+31+15+7+3+1=997

Problema ejemplo.
Determina el mayor n tal que 10n divide a 1005!

AHSME 1977

Solución.
Puesto que 10  2  5  10n  2n 5n , el número n quedará determinado por el número de
parejas 2  5 que podemos hacer en la descomposición factorial de 1005! , es decir, el
mínimo entre a y c
donde 2a  3b  5c...  1005! es la descomposición factorial de 1005!

Acabamos de ver que a  997 y c  250 , luego el resultado será min250,997  250 .

O, escrito en la forma de la fórmula de Polignac:


1005 
El exponente de 2 será  
k 1 2k 
 502  251  125  62  31  15  7  3  1  997


1005 
El exponente de 5 será  
k 1 5k 
 201  40  8  1  250

Ejemplo.
¿Cuántos ceros hay a la derecha de la expresión decimal de 300! ?

Solución.
El número de ceros vendrá dado por el número de veces que 10 divide a 300! .
Puesto que hay más factores de 2 que de 5 en 300! , el número de ceros vendrá dado la
mayor potencia de 5 en 300!, y aquí aplicamos la fórmula de Polignac:
 300   300   300   300 
 51   60 ,  52   12 ,  53   2 , y si k  4  5k   0 .


 300 
Por lo tanto,   5
k 1
k 

 60  12  2  74 .

13.3 M
1000 
Demostrar que 7 no divide a   .
 500 

13.4 F
 200 
¿Cuál es el mayor factor primo de dos dígitos del entero n    ?
 100 

AIME 1983 #8
14 Números combinatorios. Binomio de Newton.
14.1
Demuestra que la expresión
(m, n)  n 
 
n  m 
es un entero para todo par de números enteros n  m  1 .

Putnam 2000

Proposición.
 p
a) Si p es primo, p divide   para todo 0  k  p .
k
b) Si p es primo, p | 2  2 .
p

Demostración.
 p p!  p p!
a)     k!    p( p  1)( p  2)...( p  k  1)
 k  k!( p  k )!  k  ( p  k )!

 p
Está claro que p | p( p  1)( p  2)...( p  k  1) , luego p | k! 
k
 p
Puesto que k  p , es imposible que p | k! , luego p |  
k

 p  p  p
b) 2 p        ...   
0 1  p
 p  p  p  p 
Y puesto que       1 , tenemos 2 p  2     ...   
 0   p 1  p  1
 p
Aplicando el apartado anterior, p |   para todo 0  k  p , luego p | 2 p  2 .
k
Números de Catalan.

Definición. Número de Catalan de orden n.


Definimos el número de Catalan de orden n como
1  2n 
Cn   
n  1  n 

Los primeros diez números de Catalan son 1, 2, 5, 14, 42, 132, 429, 1430, 4862, 16796.

Proposición.
Todos los números de Catalan son enteros.

Demostración.
2n  1  2n   2n  1
Vamos a aplicar la identidad    
n  1  n   n  1 
Puesto que 2n  1 y n  1 son coprimos, y la parte de la derecha de la identidad anterior
 2n 
es un entero, está claro que n  1 divide a   .
n
15 Números y primos de Fermat y de Mersenne.
Números y primos de Fermat.
Definición. Números de Fermat. Primos de Fermat.
Queremos estudiar los números primos de la forma 2n  1 . Ya vimos en el Tema 4 que
si es primo entonces n debe ser una potencia de 2.

Definimos el n-ésimo número de Fermat como Fn  22  1 , n  0 .


n

F0 = 3, F1 = 5, F2 = 17, F3 = 257, F4 = 65537, F5 = 4294967297

El propio Fermat conjeturó que Fn era primo para todo n, sin embargo Euler demostró
que 641 | F5 . El razonamiento se encuentra en el Tema 4.

Así pues, no todo número de Fermat es primo. Llamaremos “primos de Fermat” a


aquellos números de Fermat que sean primos. Es un problema abierto la existencia o no
de infinitos primos de Fermat. Se conjetura que Fn es compuesto para n  5 .

Proposición.
Los números de Fermat son todos coprimos entre ellos:  Fn , Fm   1 si n  m .

Demostración.
Supongamos que m  n , y sea d un divisor común de Fn y Fm . En primer lugar vemos
que d debe ser impar, pues lo son todos los números de Fermat.
d | Fn  22  1  22  1  0 (mod d )  22  1 (mod d )
n n n

Pero entonces
22  22
m n mn
2
 22 n 2 m n
  1
2 m n
 1 (mod d )

Pero, por otro lado, d | Fm  22  1  22  1  0 (mod d )  22  1 (mod d ) , con lo


m m m

cual llegamos a 1  1 (mod d )  2  0 (mod d )  d | 2 , y puesto que d es impar, solo


puede ser d  1 .

Nota: Un argumento alternativo podría ser aprovechar la identidad


Fn  2  F0 F2 ...Fn 1

Que se deduce de
22  1  2  12  1 22  1 ... 22
m
  n 1

1

Y por tanto, si d divide Fn y Fm , con m  n , también dividirá a 2  Fn  F0 ...Fn 1 , y


puesto que d es impar, llegamos a d  1
Números y primos de Mersenne.

Números de Mersenne. Primos de Mersenne.


Definimos los números de Mersenne como aquellos de la forma M n  2n  1 , n  1.

Está claro que si n es compuesto, también lo será M n :


  b
  
Si n  ab  M n  2ab  1  2a  1b  2a  1 2a1  2a2  ...  2  1  2a  1 | M n

Por lo tanto, si M n es primo, también lo será n.

Sin embargo, existen primos p para los cuales M p es compuesto. Por ejemplo:
47 | M23, 167 | M83, 263 | M13

Llamaremos “primos de Mersenne” a los números de Mersenne que sean primos. Es


un problema abierto la existencia o no de infinitos primos de Mersenne.

Los mayores primos de Mersenne descubiertos.


Hasta octubre de 1999 solo se han descubierto los siguientes 38 primos de Mersenne:

22−1 , 23−1 , 25−1 , 27−1 , 213 – 1 , 217−1 , 219−1 , 231−1 , 261−1 , 289 −1 , 2107−1 ,
2127−1 , 2521−1 , 2607−1 , 21279−1 , 22203−1 , 22281−1 , 23217−1 , 24253−1 , 24423−1 ,
29689−1 , 29941−1 , 211213−1 , 219937−1 , 221701−1 , 223209−1 , 244497−1 , 286243−1 ,
2110503−1 , 2132049−1 , 2216091−1 , 2756839−1 , 2859433−1 , 21257787−1 , 21398269−1 ,
22976221−1 , 23021377−1 , 26972593 −1.

El 4 de noviembre del 2001 se propuso M13466917 = 213466917 − 1, como el primo de


Mersenne número 39. Su representación decimal tiene 4053946 dígitos. Fue propuesto
por el equipo de Michael Cameron, George Woltman, Scott Kurowski et al, como parte
del “Great Internet Mersenne Prime Search” (GIMPS).
16 Número de divisores de un entero.
Número de divisores de un entero.
Dado un número entero n, definimos  (n) como el número de enteros positivos
divisores de n.

Ejemplos:
Los divisores positivos de 6 son  1 , 2 , 3 , 6 , luego  (6)  4 .
Los divisores positivos de 20 son  1 , 2 , 4 , 5 , 10 , 20 , luego  (20)  6 .
 (1)  1,  (2) = 2,  (3) = 2,  (4) = 3,  (5) = 2,  (6) = 4,  (7) = 2,  (8) = 4,  (9) = 3,
 (10) = 4
Con Mathematica:

Proposición.
Para cualquier entero positivo n,
a)  (n)  1 y  (n)  1  n  1 .
b) Para todo n  2 ,  (n)  2 y  (n)  2  n es primo.
c)  (n)  2 n

Demostración.
a) y b) se deducen de la propia definición de  (n) .
c) Cada divisor positivo a de n tiene asociado su divisor complementario b  n / a .
n
Puesto que a  b  a  n , se tiene que cumplir a  n o b  n . Así pues, existirán
a
como mucho 2 n divisores de n.

Teorema.
La función d (n) es multiplicativa, es decir, si n  a b , con (a, b)  1 , entonces
 (n)   (a) (b)

Corolario.
a) Si n  p a para cierto p primo, entonces los divisores de n son

1 , p , p 2 , p 3 , ... , p a 
y por tanto  (n)  a  1

b) Si n  p1a1 p2a2 ... prar es la descomposición en factores primos de n, entonces los


divisores d de n son todos los números que se pueden escribir como
d  p1b1 p2b2 ... prbr con 0  bi  ai ,
y
 (n)  a1  1a2  1...ar  1

Por ejemplo:  (2904)  d (23  3 112 )  (3  1)(1  1)(2  1)  24


16.1 F
Demostrar que  (n) es impar si y solo si n es un cuadrado perfecto.

16.2 F
Demostrar que  (n) es primo si y solo si n  p q1 con p, q primos.

16.3 MF
Si tomamos aleatoriamente un divisor de 1099 , ¿Cuál es la probabilidad de que sea
múltiplo de 1088 ?

AIME 1998 #5

16.4 F
Determina todos los enteros positivos n tales que  (n)  6 .

16.5 F

Determina el número de enteros positivos que son divisores de al menos uno de los
siguientes números: 1010 , 157 , 1811 .

AIME II 2005 #4

16.6 D
Sea  (n) el número de enteros positivos de n. Determina la suma de los seis enteros
positivos n más pequeños tales que
 (n)   (n  1)  7

AIME 1 2019 #9

16.7 F
¿Cuántos divisores positivos de 20042004 son divisibles por exactamente 2004 enteros
positivos?

AIME II 2004 #8
Producto de los divisores de un entero.
Una propiedad interesante de  (n) es que el producto de todos los divisores de n es
igual a la raíz cuadrada de n (n ) :
d 
d |n
n ( n )

Demostración.
La clave está en observar que los divisores de un número van por parejas: Dado un
divisor d , tenemos su divisor complementario d ' n / d , de forma que d  d '  n .
Sean d1 , d 2 , ..., d ( n) todos los divisores de n. Si multiplicamos dos veces dichos
divisores, los factores se pueden reorganizar agrupando  (n) parejas cuyo resultado sea
n, luego:
2 2
   
  d    d   d  d1.d 2 ...d ( n )  d1.d 2 ...d ( n )  n ( n )    d   n ( n )
   
 d |n  d |n d |n  d |n 

Observación.
De aquí se deduce que n (n ) es un cuadrado perfecto. Esto es obvio si  (n) es par, pero
no estaba tan claro si  (n) es impar. Pero en el problema 16.1 anterior ya demostramos
que si  (n) es impar, n es un cuadrado perfecto, y por tanto también lo es n (n ) .

Ejemplo.
El producto de todos los divisores de 16 es
 d | 16 
d |n
16 ( n )  165  45  1024

En efecto, 1 2  4  8 16  1024 .


17 Suma de los divisores de un entero.
Suma de los divisores de un entero.
Dado un número entero n, definimos  (n) como la suma de todos los enteros positivos
divisores de n, incluyendo 1 y el propio n.
 ( n)   d
d |n

Está claro que  (n)  n  1, y  (n)  n  1 si y solo si n es primo.

Ejemplos.
Los divisores positivos de 6 son  1 , 2 , 3 , 6 , luego  (6)  1  2  3  6  12 .
Los divisores positivos de 20 son  1 , 2 , 4 , 5 , 10 , 20 , luego
 (20)  1  2  4  5  10  20  42 .

Teorema.
La función  (n) es multiplicativa.

Proposición.
a) Si n  p a para cierto primo p , entonces  (n)  1  p  p 2  ...  p a .
b) Si n  p1a1 p2a2 ... prar es la descomposición en factores primos de n, entonces

 (n)  1  p1  p12  ...  p1a 1  p2  p22  ...  p2a ...1  pr  pr2  ...  pra  
1 2 r

p1a11  1 p2a2 1  1 prar 1  1


  ...
p1  1 p2  1 pr  1

Demostración.
a) Aplicando la propia definición de  (n) .
b) Basta tener en cuenta que  (n) es multiplicativa y aplicar la serie geométrica:
x n1  1
x n  x n1  x n2  ...  x  1 
x 1

Por ejemplo:
23  1 33  1 52  1 7 26 24
180  22  32  5   (180)        7 13  6  546
2 1 3 1 5 1 1 2 4
Problema.
Sea n un entero positivo tal que 24 | n  1 . Demuestra que entonces se cumple
24 |  (n) .
PUTNAM 1969

Solución.
n  1  0 (mod 3)  n  -1 (mod 3)
24 | n  1  
n  1  0 (mod 8)  n  -1 (mod 8)

Observamos que los divisores de un número van en parejas: Si d1 es divisor entonces su


n
complementario d 2  también es divisor.
d1
Por un lado, haciendo módulo 3:
n
d1  d 2  d1  n  1 (mod 3) , y las únicas opciones son:
d1
d1  1 (mod 3) y d 2  2 (mod 3) o viceversa.
En todo caso se cumple d1  d 2  0 (mod 3) .

Y por otro lado, haciendo módulo 8:


n
d1  d 2  d1  n  1 (mod 8) , y las únicas opciones son:
d1
d1  1 (mod 8) y d 2  7 (mod 8) o viceversa.
d1  3 (mod 8) y d 2  5 (mod 8) o viceversa.
En todo caso se cumple d1  d 2  0 (mod 8) .

Así pues, d1  d 2  0 (mod 3) y d1  d 2  0 (mod 8) , luego d1  d 2  0 (mod 24) , y por


tanto el sumatorio  (n) se puede reordenar por parejas divisibles entre 24, y por tanto:
 (n)   d será divisible entre 24.
d |n

17.1 F
Los únicos factores primos de un número n son 2 y 3. Si la suma de todos los divisores
de n (incluyendo el propio n) es 1815, determina n.

PUMaC 2011/NT #A1


Números perfectos.

Definición de número perfecto.


Los antiguos griegos clasificaban los enteros positivos en tres clases, en función de la
suma de sus divisores:
“Números perfectos” si  (n)  2n
“Números abundantes” si  (n)  2n
“Números deficientes” si  (n)  2n

Los primeros números perfectos son: 6 , 28 , 496 , 8128 .

Si utilizamos la notación clásica en la que se cuentan solo los divisores propios, es


decir, todos menos el propio n, tiene más sentido la idea de “número perfecto” como
aquel que es igual a la suma de todos sus divisores (propios).

Teorema. Caracterización de los números pares perfectos.


Un número par n es perfecto si y solo si existen números primos p, q tales que
n  2 p1 q con q  2 p  1

Así pues, cada número par perfecto está asociado a un primo de Mersenne, que fueron
introducidos en el Tema 15.

Demostración.

Supongamos que n se puede escribir de esta forma.
Está claro que q es impar , 2n  2 p q y que q  1  2 p .
2 p11  1
 2 p1    2 p  1 ,  q   q  1
2 1
   
Luego  (n)   2 p1  q   2 p  1 q  1  2 p q  2 p  (q  1)  2n
Y por tanto n es perfecto.


Supongamos ahora que n es perfecto, es decir, que  (n)  2n . Puesto que además
estamos suponiendo que n es par, podemos escribir n  2k 1 m con m impar y k  2 .

    
Luego 2k m  2n   (n)   2k 1 m   2k 1  m  2k  1  m 
2k 11  1
Donde hemos utilizado que  2k 1   2 1
 2k  1 .

 
De la igualdad 2k m  2k  1  m se deduce que 2k  1 | 2k m , y puesto que
 
2k  1 , 2k  1 , el Lema de Euclides implica que 2k  1 | m , y por tanto

 
m  2k  1 l para cierto entero l impar.
     
2k 2k  1 l  2k m  2n   (n)   2k 1 (2k  1)l   2k 1  (2k  1)l  
 2 k
 1 (2 k

 1)l  2 l   (2  1)l
k
 k
 (*)

Si l  1 entonces 1 , l , (2k  1)l son factores diferentes de (2k  1)l , y por tanto, por la
definición de  , tenemos que 1  l  (2k  1)l   (2k  1)l  
 
Luego 2k l   (2k  1)l  1  l  (2k  1)l  1  l  2k l  l  1  2k l absurdo.
 
Así pues, l  1 , y n  2k 1 2k  1 . Ahora solo falta demostrar que k es primo.

De (*) tenemos que 2k   2k  1  1  2k  1   d  d  0


d |(2 k 1) d |(2 k 1)
1d 2k 1 1d 2k 1

Es decir, 2k  1 no tiene divisores propios, es decir, es un número primo. Ya vimos en el


Tema 15 que si un número de Mersenne es primo entonces k es primo, y con esto
acabamos la demostración.

Observación.
Acabamos de ver que todo número perfecto par está asociado a un primo de Mersenne.
Es un problema abierto la existencia o no de infinitos números perfectos pares, y apenas
sabemos nada de los números perfectos impares, ni tan solo si existe alguno.
18 Problemas olímpicos de Teoría de números.

18.1 MF
Sea n un número natural. Probar que si la última cifra de 7 n es 3, la penúltima es 4.

OME 2018 Primera Fase, segunda sesión #5

18.2 MF
Demuestra que 22225555  55552222 es múltiplo de 7.

OME Fase local, segunda sesión #4


Soluciones.
1.1
Supongamos que N  n  0  n  1  n  2  n  3  ...  n  99  100n  (1  2  3  ...  99)
99(99  1)
Sabemos que 1  2  3  ..  99   4950
2
Luego N  100n  4950 , luego será un número que acabe en "50", y el único candidato
aceptable es (A).

1.2
N  9  99  999  9999  ..  99
 99 
...
321cifras

      
 10  1  10  1  10  1  ...  10321  1
1 2 3

 
 10  10  10  ...  10   1
1 2 3
  ...
1 
321
1  10  10
1 2

 103  ...  10321  321  (*)
  321

10  10  10  ...  10
1 2 3 321
 10  100  1000  ..  100
...   111
000
 ...111
0
321 321

(*)  111
 0  321  111
...111 ....
111
0789
321 321 3 318
Las cifras de este número suman 1 318  0  7  8  9  342

1.3
Vamos a ir observando la pauta sn  1  2  3  ...  n
s1  1  d1  1
s2  1  2  3  d 2  3
s3  1  2  3  6  d 3  3
s4  1  2  3  4  10  d 4  0
s5  1  2  3  4  5  15  d5  5
s6  1  2  3  4  5  6  21  d 6  1
s7  1  2  3  4  5  6  7  28  d 7  8
Las sucesivas sn siguen de la siguiente manera:36, 45, 55, 66, 78, 91, 105, 120, 136, 153,171,
190,210,231,253,276,300,325,351,378,406,435,465,496,528,561,595,630,666,703,741,
780,820,861,903...
Vemos que los primeros d1  d2  ...  d19  70 .
y vemos que, a partir de la posición d19 sigue una pauta repetitiva de 20 números:
0, 0, 1, 3, 6, 0, 5, 1, 8, 6, 5, 5, 6, 8, 1, 5, 0, 6, 3, 1
que suman 70.

Entre d19 y d 2017 hay 20 bloques completos y 19 elementos "sueltos" finales:


2017  18  1999  99  20  19
Los 19 primeros elementos de la pauta anterior suman 69, luego
2017

d
n 1
n  70  99  70  69  7069 . El residuo de 7069 entre 1000 es 69.
1.4
Vamos calculando pacientemente la sucesión:
a3  4(8  5  2)  60  5 , a4  4(5  8  5)  76  6 ,
a5  10 , a6  7 , a7  4 , a8  7 , a9  6 , a10  2 , a11  5 , a12  8

Vemos que los tres últimos elementos coinciden con los primeros, por lo que podemos asegurar
que los elementos se van repitiendo en grupos de diez.

2018  201  10  8  a2018  a8  7 



2020  202  10  0  a2020  a0  2  a2018  a2020  a2022  7  2  8  112
2022  202  10  2  a2022  a2  8

1.5
2  4  6  8  384 , luego la multiplicación de cuatro números, acabados en 2, 4, 6 y 8 acabará en
4.
La multiplicación de números acabados en 4 va alternando: Acabados en 6 y acabados en 4:
42  4  4  16, 43  16  4  64, 44  64  4  256,... luego 410 acabará en 6.
2  4  6  8 12 14 16 18  22  24  26  28  ...  92  94  96  98 la podemos entender como la
multiplicación de diez grupos, cada uno de ellos acabado en 4, luego el resultado será el mismo
que 410 , es decir, 6.

1.6
Supongamos que nuestra sucesión es de la forma xn  a n  b para ciertos enteros a, b .
Supongamos que para cierto n se cumple xn1  a n1  b  m2 .
Entonces
m  a2  m2  a 2  2am  an1  b  a 2  2am  an1  a  2m  b
Y por tanto el término an2 con n2  n1  a  2m  n1 es también un cuadrado perfecto. Esta
sucesión de cuadrados perfectos la podemos hacer tan grande como queramos.

1.7
a 2  b2
Supongamos, por el contrario, que existe un k  , con a, b enteros positivos que no es
1  ab
un cuadrado perfecto.
Por ser a, b simétricos en la expresión, podemos suponer que a  b .
Podemos suponer, además, que b es el entero positivo más pequeño posible.

En primer lugar, vemos que a  b , pues


a 2  b2 2a 2
abk    2  k  1 , pero 1 es un cuadrado perfecto: 1  12 , llegando a
1  ab 1  a 2

contradicción.

Así pues, a  b .
a 2  b2
k  1  ab k  a 2  b 2  k  abk  a 2  b 2
1  ab
 a 2  b 2  k  abk  0  b 2  akb  a 2  k  0
Esta última expresión la podemos considerar como una ecuación de segundo grado en b, con lo
que existirán dos soluciones, b,b1 , tales que:
ak  b  b1
a 2  k  b b1

Puesto que a, b  0 , la hipótesis b1  0 contradice la condición


1  ab1 k  a 2  b12
Puesto que k no es un cuadrado perfecto, la hipótesis b1  0 también contradice la condición
anterior:
1  a  0k  a 2  02 . Así pues, b1  0 .
Por otro lado:
b1b  b1b  b1 ak  b1   a 2  k  akb1  b12  a 2  k  akb1  k  a 2  b12
a 2  b12
 k ab1  1  a  b  k 
2 2

ab1  1
1

Y también:
a 2  k b2  k
b1   b
b b

a 2  b12
Luego hemos encontrado un nuevo número b1  b que también satisface k  , cuando
ab1  1
habíamos supuesto que b era el menor número posible, llegando a contradicción. Por lo tanto,
k tiene que ser un cuadrado perfecto.

Fuente de la solución: Elementary Number Theory Notes (David A. Santos, 2004) , pág. 3.

1.8
Aquí el conjunto de casillas son las 27 letras del abecedario. Por lo tanto, si tenemos 28
personas y las colocamos en las casillas, seguro que habrá al menos dos ocupando una misma
casilla, es decir, tendrán nombres con la misma inicial. Si solo hubiera 27 podría darse el caso
de que todas tuvieran nombres con iniciales diferentes.

1.9
Si tenemos 27  5  135 personas, podría darse el caso de que hubieran 5 personas en cada una
de las 27 letras (“casillas”). Luego llamando a una más, es decir, con 136 personas,
garantizamos que al menos en una de las casillas hay un mínimo 6 personas, es decir, que hay
al menos 6 personas con la misma inicial.

1.10
En este caso tenemos 2 “casillas”: “varón” y “mujer”, y si colocamos 6 personas, podría darse
el caso de tener tres y tres. Pero si añadimos la séptima, seguro que al menos en una de las dos
hay 4 personas.
1.11
Los elementos de A son de la forma xi  4ki , con 1  ki  25 , y
xi  x j  104  4ki  4k j  104  ki  k j  104 / 4  26 , luego nuestro problema se reduce a un
conjunto A de 20 elementos entre 1 y 25 en el que debemos encontrar al menos una pareja cuya
suma sea 26.
Consideremos los siguientes 13 conjuntos, que serán nuestras “casillas”:
1, 25  ,  2 , 24  , 3, 23  ,  4 , 24  , … , 11,15  , 12 ,14  , 13 
Cada elemento de A irá a parar a una de estas casillas, y puesto que hay 20 números y solo 13
casillas, aplicando el “Principio del casillero”, obligatoriamente dos elementos de A irán a
parar a la misma casilla. Dicho de otra manera, existirán dos números de A cuya suma sea 26.

1.12
Consideremos los conjuntos siguientes:
{ 1 , 2 } , { 3 , 6 } , { 7 , 14 } , { 15 , 30 } , { 31 , 62 } , { 63 , 126 }

Los siete números del enunciado deberán ir a alguno de estos seis conjuntos, y por el Principio
del Casillero, al menos existirán dos elementos a  b en el mismo conjunto. Los conjuntos
están ya diseñados de forma que la distancia máxima sea
n  a  b  2n  2a  a  b  2a .

1.13
Observamos que en un conjunto de 10 elementos, por ejemplo en
1,11, 21, 31, 41, 51, 61, 71, 81, 91 
Solo pueden haber un máximo de cinco elementos separados. Si añadimos un sexto, estará
tocando, por la derecha o por la izquierda, a otro.

Consideremos los siguientes 10 conjuntos:

1,11, 21, ..., 91  ,  2 ,12, 22, ..., 92  , … , 9 ,19, 29,..., 99  , 10 , 20 , 30 , ...,100 
Al repartir los 55 elementos en estos 10 conjuntos, al menos habrá uno con 6 elementos o más,
y no podrán estar separados entre ellos, luego existirán dos elementos juntos, es decir, cuya
diferencia será 10.

2.1
  16
416525  22 525  2216525  232525  27 225525  128  (2  5) 25  128  1025 
 1.28  1027

Un número que tiene 28 cifras decimales.

2.2
Si n es impar, n  2k  1 para cierto k entero, luego
n2  2k  1  4k 2  4k  1  4k (k  1)  1
2

Ahora bien, el producto de dos números consecutivos es siempre par, pues o bien k es par o
bien k  1 es par, y por tanto 4k (k  1) es múltiplo de 8, tal y como queríamos ver.
2.3
Sea x la longitud del lado del cuadrado.
Entonces x  d es la longitud del lado del triángulo equilátero y
3( x  d )  4 x  1989 
3x  3d  4 x  1989 
3d  x  1989 
3d  1989  x

Puesto que suponemos x  0 , se tiene que cumplir 3d  1989  0 y esto solo pasa si d  663 .
Luego existen 663 casos (contando d  0 ) que no son válidos para d.

2.4
La condición (i) equivale a a  4,5
La condición (ii) equivale a d  0,5
La condición (iii) equivale a a b 34,35,36,45,46,56
Luego el número posible de enteros es 2  6  2  24

2.5
Una vez introducida la cuarta puntuación, el resultado tiene que ser múltiplo de 4.
El 80 es solución al problema. En efecto, 71  76  82  91  320 , que es múltiplo de 4.
Cualquier otra posibilidad no es aceptable:

71  400  71  329 , 76  400  76  374 , 82  400  82  378 , 91  400  91  389

Ninguno de estos números es múltiplo de 4.

2.6
Tenemos que resolver la ecuación x 2  6 x  n2 con x, n enteros.
x2  6 x  n2  x2  6 x  n2  0 

x  

 6  62  4  1  (n 2 )  6  36  4n 2  6  4 n 2  9 

2 2 2
 6  2 n2  9
  3  n 2  9
2

Luego n2  9 debe ser un cuadrado:


n 2  9  m2  9  m2  n 2   m  n  m  n 
Por otro lado, m  n  m  n , luego la posibilidad 9  3  3 queda descartada, y la única
m  n 1
posibilidad es  m  n  m  n   1  9  
m  n  9

Resolvemos el sistema anterior:



m  n 1 m 1 n
 1 n  n  9  1 2n  9  n  4  m 1 4  5

 m  n  9

Así pues, las soluciones son:


n  4, m  5  (m  n)(m  n)  (1)  (9)  9
n  4, m  5  (m  n)(m  n)   9 1  9
n  4, m  5  (m  n)(m  n)  (9)  (1)  9
2
En todo caso, n 2  m2  9  25  9  16  x  3  16  9  3  25  3  5  
 8
2.7
Primera versión.
27b  2b  7 es un cubo perfecto, es decir, 2b  7  m3 para cierto entero m.

b  1000  m3  2b  7  2007

Puesto que hay menos cubos perfectos que cuadrados perfectos, listamos todos los cubos:

23  8, 33  27, 43  64, ..., 123  1728

Puesto que 2b  7 es siempre impar, podemos eliminar de la lista todos los cubos pares, y
quedarnos solo con
33 , 53 , 73 , 93 ,113

36b  3b  6 es un cuadrado perfecto, es decir, 3b  6  n2 para cierto entero n.

n2  3b  6  3b  2  3 | n2  3 | n  n  3k  n2  (3k )2  9k 2

Luego nos podemos quedar solo con aquellos cubos divisibles entre 9: 33 , 93

Ya solo nos queda comprobar si estos candidatos se adaptan a nuestras condiciones:


33  27  2b  7  b  10  3b  6  36  62 , luego la base 10 es aceptable.
93  729  2b  7  b  361  3  361  6  1089  332

Luego el resultado es 361  10  371.

Segunda versión.
36b  3b  6  3(b  2)  n2 es un cuadrado perfecto, luego n  3k  n2  9k 2 .
3(b  2)  9k 2  b  2  3k 2  b  3k 2  2

El valor de k está limitado superiormente: b  1000  3k 2  2  1000  k  18

Sustituyendo en la segunda ecuación:


   
m3  2b  7  2 3k 2  2  7  6k 2  4  7  6k 2  3  3 2k 2  1
y vemos que 3 es divisor de m3 , y por tanto 3 es divisor de m .
También vemos que es impar:
 
2k 2 par  2k 2  1 impar  3 2k 2  1 impar

Y que está acotado superiormente:


b  1000  m3  2b  7  2  1000  7  2007

y 132  2197 , luego m  13

Los cubos que cumplen las condiciones anteriores son dos: 33 , 93 , y como en la versión
anterior, solo nos queda comprobar que, efectivamente, satisfacen las condiciones del
enunciado.

Fuente de estas versiones: www.artofproblemsolving.com

2.8
n2  85n  2017 es un cuadrado perfecto 
n2  85n  2017  m2 para cierto m  Z 

n 2  85n  2017  m2  0  n 

 85  852  4  1  2017  m2

 85  4m2  843

2 1 2 1
4m2  843 es un cuadrado perfecto 
4m2  843  p 2  4m2  p 2  843   2m  p  2m  p   843  3  281

Las dos posibilidades son:


a)

 2m  p  3 
2 m  p  3
   m  71, p  139
 2m  p  281 
 2 m  p  281

p  139  4m 2  843  p 2  1392 


 85  4m 2  843  85  1392  85  139 27
n   
2 1 2 1 2 1  112
b)
 2m  p  1
 
2 m  p  1
   m  211, p  421

 2 m  p  843 
2 m  p  843

p  421  4m 2  843  p 2  4212 


 85  4m 2  843  85  4212  85  421 168
n   
2 1 2 1 2 1  253

Comprobamos estas cuatro soluciones:


n  27  n2  85n  2017  5041  712
n  112  n2  85n  2017  5041  712
n  168  n2  85n  2017  44521  2112
n  253  n2  85n  2017  44521  2112

Finalmente, nos piden la suma de las soluciones positivas: 27  168  195 .

2.9
Seguimos la indicación propuesta.
Todo número n acabado en "08" es múltiplo de 4. En efecto, se podrá escribir como
n  m  100  8  425m  2
Luego todo número de la forma 111...111 se puede escribir como
111...111  111...108  3  4k  3 para cierto k natural.
Supongamos que 111...111  111...108  3  4k  3  a 2 para cierto a  IN
Está claro que a 2 es impar, luego a será impar, y por tanto a  2b  1 , para cierto b  IN .

Luego
a 2  2b  1  4b 2  4b  1  4k  3  4b 2  4b  1 
2


4k  4b 2  4b  2  2 2b 2  2b  1  
2k  2b(b  1)  1

Lo cual es contradictorio, pues 2k es par, y 2b(b  1)  1 es impar, llegando a contradicción.

2.10
Sabemos que x 2  x  n  ( x  a)( x  b)  x 2  (a  b) x  ab

Puesto que el término independiente es negativo, a o b deben ser negativos. Por lo tanto,
buscamos a, b positivos tales que:
ab  n
x 2  x  n  ( x  a)( x  b)  x 2  (a  b) x  ab    n  b(b  1)
a  b  1  a  b  1
Los números entre 1 y 100 que se pueden escribir como el producto de un número y el
siguiente son los siguientes nueve:

1 2  2 , 2  3  6 , 3  4  12 , 4  5  20 , 5  6  30 , 6  7  42 , 7  8  56 , 8  9  72 , 9 10  90

2.11
Nos basaremos en la igualdad
1  a  n  m  100
x 2  ax  b  x  n x  m  x 2  (n  m) x  n m  
b  n m  0
La segunda condición implica que n, m tienen el mismo signo, pero entonces la primera
condición implica que los dos no pueden ser negativos, luego n, m serán positivos o cero.

Si b  0 , el polinomio x 2  ax  b  x 2  ax  xx  a  es factorizable siempre, luego hay cien


casos: 1  a  100 .

Supongamos que 0  b  n m  n, m  0

Al no encontrar más restricción, procedemos a contar casos.


Para cada valor de 1  a  100 , las parejas no ordenadas n, m, n, m  0 , tales que n  m  a
(no importa el orden pues (n, m) y (m, n) dan lugar a la misma ecuación pues la ecuación es
simétrica en n, m)

Por ejemplo:
a 1 
a  2  1,1
a  3  1,2
a  4  1,3, 2,2
a  5  1,4, 2,3
a  6  1,5, 2,4, 3,3
a  7  1,6, 2,5, 3,4
a  7  1,6, 2,5, 3,4
a  8  1,7, 2,6, 3,5, 4,4
a  9  1,8, 2,7, 3,6, 4,5
...
a  98  1,97, 2,96, 3,95, 4,94...
a  99  1,97, 2,96, 3,95, 4,94...
a  100  1,99, 2,98, 3,97, 4,96...

Vemos que van por parejas, y que la suma de todos los elementos será
1  1  2  2  3  3  4  4  5  5  ...  49  49  50 
49
49  50
2 n  50  2  50  49  50  50  2500
n 1 2

Y sumando los 100 casos cuando b  0 dan un total de 2600 casos.


Finalmente, 2600 mod1000  600

2.12
Primera versión.
Las secuencias a tratar son de la forma

 2a  1 , 2a  3 , ... , 2a  31    2a  2b  1  1  b  16

La suma será
16 16 16
s   2a  2b  1  162a  1   2a  162a  1  2 a 
b 1 b 1 b 1

16  17
 162a  1  2  32a  16  16  17  32a  16  16  25 a  8
2

Buscamos valores de a de forma que s  25 (a  8) sea un cubo perfecto.


25 (a  8)  23 22 (a  8) es un cubo perfecto  22 (a  8) es un cubo perfecto
22 (a  8)  d 3  2 | d3  2 | d  d  2e  e3  2e  23 e3 
3


22 (a  8)  23 e3  a  8  2e3  a  2e3  8  2 e3  4 
Luego los valores de a buscados son los de la forma a  2e3  4

Ahora solo queda acotar el rango de valores, pues nos imponen


100  2a  1  999  99  2a  998  99 / 2  a  998 / 2  49.5  a  498 
50  a  498
Y por tanto
 
50  2 e3  4  498  50 / 2  e3  4  498 / 2  25  e3  4  249 
25  4  e3  249  4  29  e3  253

33  27 , 53  125 , 63  216 , 73  343 , luego 4  e  6 , y hay tres secuencias.

Segunda versión. La solución oficial de este problema es mucho más expeditiva.


Queremos determinar secuencias n, n  2, n  4, n  6,..., n  30 con n impar y cuya suma sea
un cubo perfecto.
La suma es 82n  30  23 (2n  30) será un cubo perfecto si y solo si 2n  30 es un cubo
perfecto.
Luego buscamos valores n impares y 101  n  999 tales que 2n  30 sea un cubo perfecto (y
además es par).
Los cubos pares entre 232 y 1968 son 512, 1000 y 1728, que se corresponden con los valores
de n: 241, 485 y 849. Luego hay exactamente tres secuencias válidas.

Fuente de la segunda versión: Solución oficial de la OME.

2.13
a) Los números pares e impares van alternados, luego en la secuencia n(n  1) al menos uno de
los factores es par.
b) De la misma manera, para una sucesión de tres números consecutivos, al menos uno de ellos
ha de ser múltiplo de tres. Y también se puede aplicar el apartado a, luego la sucesión será
múltiplo de 2  3  6 .
      
c) n5  n  n n4  1  n n2  1 n2  1  nn  1n  1 n2  1 , luego podemos aplicar los
apartados anteriores para garantizar que es divisible entre 6. Veamos que también es divisible
entre 5.
Por el Teorema de la división, el número n se puede escribir como n  5k  q , 0  q  4 .
Si n  5k  0  n  5k y sería un factor de n5  n .
Si n  5k  1  (n  1)  5k y sería un factor de n5  n
Si n  5k  4  (n  1)  5k y sería un factor de n5  n .
 
Si n  5k  3  (n2  1)  (5k  3)2  1  52 k 2  2  5  3k  9  1  5 5k 2  6k  2 y sería un
factor de n5  n .
 
Si n  5k  2  (n2  1)  (5k  2)2  1  52 k 2  2  5  2k  22  1  5 5k 2  4k  1 y sería un
factor de n5  n .
Así pues, 5 | n 5  n .
Y puesto que también son divisores 2 y 3, lo será su producto: 2  3  5  30 | n 5  n

2.14
Los múltiplos de 5 acaban todos en 0 o en 5, luego si es uno menor, acabará en 4 o en 9.
Los múltiplos de 4 son todos pares, luego si sumamos uno, serán impares, y por tanto no
pueden acabar en 4.
Por lo tanto, solo nos queda buscar entre los primos acabados en 9 que sean de la forma 4k  1 .
Solo hay dos: 29  4  7  1 y 89  4  22  1 .
Y su suma es 29  89  118 .

2.15
log b 729  a  b a  729  36  323 . Las posibilidades son las cuatro siguientes:
323  b  3, a  6
3   b  9, a  3
2 3

3   b  27, a  2
3 2

3   b  729, a  1
2 3 1

2.16
a) Aplicando el algoritmo de la división, todo entero n siempre será de la forma
3a  0 , 3a  1 , 3a  2
 
n  3a  0  n 2  3a   9a 2  3  3a 2  3k
2

 
n  3a  1  n 2  3a  1  9a 2  6a  1  3 3a 2  2a  1  3k  1
2

n  3a  2  3(a  1)  1  3b  1, tomando b  a  1
 
y por tanto n 2  3b  1  9b 2  3b  1  3 3b 2  b  1  3k  1
2

b) Es una aplicación directa del apartado anterior, pues


 
n 2  3a 2  1  3k  3a 2  3k  1  3 a 2  k  1  3 | 1 absurdo.
 
n 2  3a 2  1  3k  1  3a 2  3k  2  3 a 2  k  2  3 | 2 absurdo.

2.17
En primer lugar dividimos numerador entre denominador:

N2 +7 N+4
-4N +7 N-4
23
N2  7 23
Luego  N 4
N 4 N 4
23
Y por tanto nuestro problema se reduce a determinar las fracciones no irreducibles,
N 4
cuando 23 y N  4 tengan algún divisor común. Puesto que 23 es primo, esto solo pasará
cuando N  4 sea múltiplo de 23.

23 | N  4  N  4  23k  N  23k - 4
5 1994
1  23k - 4  1990  1  4  23k  1990  4  5  23k  1994  k
23 23
 1  k  86

La solución es 86.

2.18
El enunciado nos está diciendo que
1059  k1 d  r

1417  k 2 d  r
2312  k d  r
 3

Restando la segunda ecuación a la primera obtenemos:


358  k2  k1 d  d | 358  2 179

Restando la tercera a la segunda obtenemos:


895  k3  k2 d  d | 895  5 179

Y restando la tercera a la primera obtenemos:


1253  k3  k1 d  d | 1253  7 179

Puesto que d  1 , la única posibilidad es d  179 , y entonces, realizando por ejemplo la


primera división tenemos
1059  5 179  164  r  164

Finalmente, d  r  179  164  15

2.19
n2  23  n2  1  24  (n  1)(n  1)  24

Luego basta demostrar que existen infinitos n tales que 24 | (n  1)(n  1) .

Tomando n  12k  1, por un lado tenemos n  1  12k  12 | n  1

Pero también vemos que n es impar, y por tanto n  1 es par, luego 2 | n  1

12 | n  1
  24 | (n  1)(n  1)  24 | (n  1)(n  1)  24  n  23
2
Finalmente,
2 | n 1 

Nota: Más fácilmente, bastaba tomar elementos de la forma n  24k  1 .

2.20
d | n2  1 

 
  d | n  1  1  n  1  2n  1
2 2

d | n  1  1
2

d | 2n  1  d | 2n  1  4n 2  4n  1
2
d | 4n 2  4n  1

   
  d | 4 n  1  1  4n  4n  1  4n  7
2 2

d | n  1  1 
2

d | 2n  1 
  d | 4n  7  22n  1  5
d | 4n  7 

d | 5  d 1, 5

 1 , 5 | 2 1
2

Las dos soluciones son válidas, basta tomar n  2  



 1 , 5 | (2  1)  1
2

2.21
En primer lugar, vemos que d 2  2 , pues si d 2  2 entonces n es impar y el resto de divisores
también son números impares, pero entonces d 22 , d33 también son impares, y n  d 22  d33 es la
suma de dos impares, luego par, llegando a contradicción.

Así pues, 1  2  d3  ...  dk  n , y n  22  d33  4  d33

En segundo lugar, d 3  3 , pues si d 3  3  n  22  33  31 que no es aceptable.

En tercer lugar, vemos que d 3 tiene que ser 4, pues


d3 | d33 
  d3 | 4  n  d3  d3  4 , y como d3  1,2,3 , necesariamente d 3  4 .
3

d3 | n 

Y, finalmente, n  d22  d33  22  43  68 , y efectivamente, los divisores de 68 son:


1  2  4  17  34  68 , y 68  d22  d33  22  43

2.22
Sabemos que el número de divisores de n 2  262 338 es 62  138  1  2457 .
Todos estos divisores se pueden agrupar por parejas a , b de forma que a  b  n 2
De forma que, o bien a  b  n o bien a  n o b  n .
Así pues, hay
62  138  1  1  1228 factores de n 2 menores que n.
2
De entre estos 1228 factores, hay 31  119  1  1  639 que son divisores de n, luego el
número buscado es 1228  639  589 .

3.1
Serán los mismos puntos que en el extremo entre ( 0 , 0 ) y ( 48  3, 281  17 )  ( 45 , 264 ) y ya
sabemos que son en total: mcd ( 45, 274 )  1  3  1  4 .
Concretamente:
45  ( 3 ,17 )
 15  
3  ( 3  15 ,17  88 )  (18 ,105 )

264 (18  15 ,105  88 )  ( 33 ,193 )
 88 
3  ( 33  15 ,193  88 )  ( 48 ,281)

3.2
( n  k , n) | n  k 
  ( n  k , n) | n  k  n  k
( n  k , n) | n 

3.3
El problema equivale a demostrar que 21n  4 , 14n  3  1, y esto, por el TDB, es una
consecuencia directa de encontrar una combinación lineal de ambas expresiones igual a 1:

 2(21n  4)  3(14n  3)  42n  8  42n  9  1

3.4
an  100  n2 , an 1  100  (n  1)2  100  n2  2n  1  an  2n  1
Luego dn  an , an 1   an , an  2n  1   an , 2n  1 
Y también dn | (100  n2 )  n(2n  1)  200  n
y por tanto dn | 2200  n  (2n  1)  401 .
Así pues, d n | 401 . Y este valor se alcanza en algún n. Por ejemplo, para n  200 ,
a200  100  n2  100  2002  100  4  1002  1001  4  100  100  401
a201  a200  2  200  1  100  401  401  401  101
y por tanto d 200  401 .

3.5
En primer lugar, podemos suponer m  n  k con k  1 . Debemos demostrar que

(n  k , n)  (n  1  k , n  1)  (n  2  k , n  2)  2k  1

Vamos a utilizar la propiedad (n  k , n) | k , independientemente del valor de n , demostrada en


el problema 3.2.

Luego
( n  k , n) | k  k  a  ( n  k , n)
(n  1  k , n  1) | k  k  b  (n  1  k , n  1)
(n  2  k , n  2) | k  k  c  (n  2  k , n  2)

k  a  ( n  k , n) 
  k | an
( n  k , n) | n 

k  b  (n  1  k , n  1)
  k | b(n  1)
(n  1  k , n  1) | n  1 
k
Puesto que (n, n  1)  1 , de aquí deducimos que k | ab , y por tanto k  ab  b
a

k
De la misma forma deducimos que k | bc , y por tanto k  bc  b
c

Así pues,
k k k k k
(n  k , n)  (n  1  k , n  1)  (n  2  k , n  2)     b   b  2b 
a b c b b

k
Queda por demostrar que 2b   2k  1
b
k
2b   2k  1  2b 2  k  2kb  b  2b2  k  2kb  b  0  (b  k )(2b  1)  0
b

Pero b | k  b  k  0 , y por otro lado b  1  2b  1  0 , con lo que acabamos la


demostración.

Fuente de la solución: Solución oficial.

4.1
 
Puesto que n3  1  n  1 n2  n  1 , y teniendo en cuenta que n2  n  1  1 ,
n3  1 primo implica que n  1  1  n  2  n2  1  7 . El único primo de esta forma es el 7.

4.2
N  2903n  803n  464n  261n  2903n  803n  464n  261n  
Aplicando la proposición anterior:
7  300  2100  2903  803 divide a 2903n  803n
7  29  203  464  261 divide a 464n  261n
Y por tanto 7 divide a N.


Por otro lado, 2903n  803n  464n  261n  2903n  464n  803n  261n 
271 9  2439  2903  464 divide a 2903n  464n
271  2  542  803  261 divide a 803n  261n
Y por tanto 271 divide a N.

Finalmente, puesto que (7,271)  1 , tendremos que 1897  7  271 dividirá a N.

4.3
Primera versión.
Está claro que para n  1 , n4  4  5 que es primo.
n 4  4  n 4  4n 2  4  4n 2  (n 2  2)2  (2n)2  (n 2  2  2n)(n 2  2  2n) 
 
 (n  1) 2  1 (n  1)2  1 
Y si n  1 cada factor es mayor que 1, luego n4  4 no puede ser primo.
Segunda versión.
Está claro que para n  1 , n4  4  5 que es primo.

Aplicamos la identidad algebraica x 4  4 y 4  x 2  2 xy  2 y 2 x 2  2 xy  2 y 2 .  
En nuestro caso:

n4  4  n4  4  14  n2  2n  2 n2  2n  2  
Puesto que n2  2n  2  1 , debemos suponer que
n2  2n  2  1  0  n2  2n  1  (n  1)2  n  1

4.4
Si n  1 , n4  4n  1  4  5 que es primo. Supongamos que n  1 .
Está claro que si n es par no será primo:
n  2k , k  1  n4  4n  (2k )4  42k  16k 4  16k es un múltiplo de 16.

Supongamos que n es impar.


    2 2

n 4  4 n  n 2  2 n  2 n 2 2 n  2 n 2 2 n  n 2  2 n  2n 2 2 n  
2

 n  2   n 2  n  2   2
n 2
n n 2
n 1 ( n 1) / 2 2
2 2 2

 n  2  2
2 n
n n  2  2
( n 1) / 2
n 2 n ( n 1) / 2

n 1
Esta última igualdad está bien construida pues estamos suponiendo que n es par, luego es
2
entero.

Está claro que n2  2n  2( n 1) / 2 n  1.


(Faltaría demostrar que n2  2n  2( n 1) / 2 n  1 si n  3 ).

4.5
Aplicamos el Binomio de Newton:
n  1n  n n    n n1    n n2  ...    n1  1  n n    n n1    n n2  ...  n 2  1
n n n n n
1  2 1 1  2
 n  n   n n 
Luego n  1  1  n n    n n 1    n n  2  ...  n 2  n 2  n n  2    n n 3    n n  4  ...  1 ,
n

1  2  1  2 

4.6
Teniendo en cuenta que 1993 es impar, basta agrupar por parejas:
1001  1  1000 | 11993  10001993
1001  2  999 | 21993  9991993
1001  3  998 | 31993  9981993

Por lo tanto, 1001 dividirá al total.

4.7

De la igualdad x2n 1  y 2n 1  ( x  y) x2n  x2n 1 y  x2n  2 y 2  ...  y 2n 
deducimos que si n es impar, a  b | a n  bn . En nuestro caso n  47 impar, luego:
Luego 7  1  6 | 147  647 , 7  2  5 | 247  547 y 7  3  4 | 347  447 , y por tanto
7 dividirá a la suma de todos, tal y como queríamos ver.

4.8
Supongamos que n  ab , es decir, n  10a  b . Entonces
10a  b2  10b  a 2  100a 2  20ab  b2  100b2  20ab  a 2  
 
 100(a 2  b 2 )  b 2  a 2  100(a 2  b 2 )  a 2  b 2  99(a 2  b 2 ) 
99(a  b)(a`b)
Que es un número divisible entre 9, entre 11, entre la suma de los dígitos y entre la diferencia
de los dígitos, luego la respuesta es (B).

5.1

17 | 2 x  3 y 
  17 | 15 x  3 y  35 x  y  , puesto que 17 | 3 , necesariamente 17 | 5x  y
17 | 17 x 
17 | 2 x  3 y 
  17 | 2 x  14 y  2x  7 y  , y puesto que 17 | 2 , necesariamente 17 | x  7 y
17 | 17y 

17 | 5 x  y  17 | 25 x  y   10 x  2 y 
  17 | 10 x  2 y  ( x  7 y)  9 x  5 y
17 | x  7 y 

tal y como queríamos ver.



17 | 9 x  5 y 
  17 | 8 x  5 y
17 | 17 x 
17 | 9 x  5 y 
  17 | 12 y - 9 x  3(4 y  3x)  17 | 4 y - 3x  17 | 24 y - 3x   8 y  6 x
17 | 17 y 

17 | 8 x  5 y 
  17 | 8 y  6 x  8 x  5 y   8 y  6 x  8 x  5 y  3 y  2 x
17 | 8 y  6 x
tal y como queríamos ver.

5.2
Aplicamos las propiedades básicas de los logaritmos.
C  A log 200 5  B log 200 2  log 200 5 A  log 200 2 B  log 200 5 A  2 B 
200C  5 A  2 B
Y ahora aplicamos el TFA:
2C  A

200C  52  23 
C
 52C  23C  5 A  2 B  
3C  B
Por hipótesis, 1  ( A, B, C)  ( 2C , 3C , C )  C ( 2 , 3,1)  C  C  1
Y por tanto: A  2C  2 , B  3C  3 , y C  1 , y finalmente: A  B  C  2  3  1  6 .

5.3
28  211  2n  2304  2n  482  2n  k 2  2n  k 2  482  k  48k  48 .
Luego, aplicando el TFA, k  48  2a y k  48  2b , con a  b  k .
Pero entonces, restando las dos igualdades anteriores:

25  3  96  2a  2b  2b 2a b  1 
Y , de nuevo por el TFA, b  5 , y 3  2a b  1  4  22  2a b  a  b  2
Con lo que llegamos a  2  b  2  5  7 , es decir, k  7  5  12 .

5.4
1998  2  3  3  3  37
Tenemos 1998  2  3  3  3 37  54  37 y 54  37  17
y es fácil ir comprobando, una a una, que cualquier otra combinación posible da como
resultado una diferencia mayor. Por ejemplo:
1998  2  3  3 3  37  27 104 y 104  27  77

5.5
Buscamos el menor número n tal que (n  1)  n  (n  1)  3n sea un cuadrado y sea un cubo
perfecto, es decir:

3n  a 2 , 3n  b3 para cierto ciertos a, b  IN


3n  a 2  3 | a 2  3 | a . Así pues, la descomposición factorial de a y a 2 :
a  p1a1 3a2 p3a3 ... pkak  a 2  p12a1 32a2 p32a3 ... pk2ak

También 3n  b3  3 | b3  3 | b . Así pues, la descomposición factorial de b y b3 :


b  p1b1 3b2 p3b3 ... pkbk  b3  p13b1 33b2 p33b3 ... pk3bk

Y, puesto que a 2  b3 ,
p12a1 32a2 p32a3 ... pk2ak  p13b1 33b2 p33b3 ... pk3bk

Y por tanto 32a2  33b2  2a2  3b2 . El número 6  mcm(2,3) es el exponente más pequeño
que sea múltiplo de 2 y de 3, luego, el candidato mínimo que cumpla las condiciones
anteriores será cuando tomemos solamente el factor 3 y con su exponente más pequeño:
3n  36  n  35  243 .
Así pues, los números buscados son 242, 243 y 244.

242  243  244  729  36  32  33    


3 2

5.6
Primera versión.
Queremos determinar números a  0 y n  3 tales que

2009  a1  a2 ,...  an  a  1  a  2  a  3  ...  a  n  n  a  (1  2  3  ...  n) 


n(n  1)
 na 
2
n(n  1)  n  1   2a  n  1  n
Es decir: 2009  7 2  41  n  a   n a    n   2a  n  1
2  2   2  2
Estudiemos todas las posibilidades:
n 1 7 1
n7,a  7  41  a  7  41   283
2 2
n 1 49  1
n  7 2  49 , a   41  a  41   16
2 2
n 1 41  1
n  41 , a   7 2  49  a  49   28
2 2
n 1 287  1
n  41  7  287 , a  7a 7  137 (*)
2 2
n 1 11
n 1, a   2009  a  2009   2008 (*)
2 2
n 1 2009  1
n  2009 , a  1 a  1  1004 (*)
2 2
n 7  41  1  14
 7  n  14 , 2a  n  1  2a  14  1  7  41  a   136
2 2
n 49  1  82
 41  n  82 , 2a  n  1  2a  82  1  49  a   17 (*)
2 2
n 41  1  98
 7 2  49  n  98 , 2a  n  1  2a  98  1  41  a   29 (*)
2 2
n 2009  1  2
 1  n  2 , 2a  n  1  2a  2  1  2009  a   1003 (*)
2 2
n 1  1  4018
 2009  n  4018 , 2a  n  1  2a  4018  1  1  a   2009 (*)
2 2

Las soluciones marcadas con (*) quedan descartadas por no cumplir las condiciones del
enunciado, luego las soluciones del problema son cuatro:

a  283 , n  7 ; a  16 , n  49 ; a  28 , n  41 ; a  136 , n  14

Segunda versión.
En primer lugar, acotamos los posibles valores de n:
 n 1  n 1
2009  n a    n 
 2   2 
 n 1
La ecuación 2009  n  tiene soluciones n  62.8 , luego n  62 .
 2 

 n  1   2a  n  1  n
2009  7  7  41  n a    n   2a  n  1
 2   2  2

Si n es impar, n  7 , n  41 o n  7 2  49 , pues cualquier otro divisor impar de 2009 es mayor


que 7  41  287 , y se obtienen las sucesiones con 283, 28 y 16.
Si n es par, entonces n / 2 es un divisor de 2009, y por tanto n  14 y a  136 , porque
cualquier otra combinación daría un n  62 .

Fuente de la segunda versión: Solución oficial.

5.7
2001  3  23  29  1 3  23  29 , luego el mayor valor posible aparecerá multiplicando los dos
factores más grandes: 23  29  667
Luego será tomando I  1, M  3, N  667  I  M  O  667  3  1  671

5.8
En primer lugar, aplicamos las propiedades fundamentales de los logaritmos:
A log 200 5  B log 200 2  C 
log 200 5 A  log 200 2 B  C 
 
log 200 5 A 2 B  C 
5 A  52C  A  2C

5 A 2 B  200C  2352 
C
 2 3C 5 2 C   B
2  23C  B  3C

En el último paso hemos aplicado el TFA, es decir, la unicidad de la factorización de todo


entero.
Está claro que C | A y C | B , luego para que A y B sean coprimos es necesario que C  1 , y
por tanto A  2, B  3 , y A  B  C  2  3  1  6

5.9
Por el TFA, puesto que 2310  2  3  5  7  11 , cinco números primos, el número de formas de
escribir 2310  2  3  5  7  11 como producto de tres números será el número de formas que
tenemos de repartir los números  2 , 3, 5 , 7 ,11 en tres grupos, sin repeticiones y sin importar el
orden.
Puesto que uno de los tres elementos puede ser 1 (como mucho uno de ellos, pues suponemos
que a, b, c son distintos), también debemos considerar el caso que uno de los tres grupos esté
vacío, por ejemplo a  1 , b  2  3 , c  5  7  11 .

Organizamos los casos en función del número de elementos de cada grupo:


5!
i) (3,1,1)  C53   10 posibilidades.
3!(5  3)!
5!
ii) (0,1,4)  C54   5 posibilidades.
4!(5  4)!
5!
iii) (0,2,3)  C53   10 posibilidades.
3!(5  3)!
iv) (1,2,2)  5 posibilidades para el primer grupo, y el número de posibilidades de repartir 4
1  4
elementos en dos grupos: 5     5  3  15 posibilidades.
2  2

Luego, en total, hay 10  5  10  15  40 posibilidades.


5.10
Primera versión.
  
16 p  1  n3  16 p  n3  1  n  1 n2  n  1 luego 24 p  n  1 n2  n  1 
De todas las posibilidades que podemos plantear, si n  1 es 1 , 2 , 4 , 8 entonces
n2  n  1  n(n  1)  1 será múltiplo de 2, lo cual es absurdo pues es impar (el producto de dos
números consecutivos es par, y si le sumamos 1 será impar).

Veamos la opción n  1  24 , p  n2  n  1
n  1  24  n  17  p  172  17  1  307 , que es primo, y 16 p  1  4913  173

Segunda versión.
De 16 p  1  n3 se deduce que n tiene que ser impar. Luego n  2a  1
 
16 p  1  (2a  1)3  8a3  12a 2  6a  1  16 p  2a 4a 2  6a  3 

8 p  a 4 a  6a  3
2

4a  6a  3 es impar, luego en su factorización no puede tener ninguna potencia de 2.
2

La única opción válida es que a  8 , y por tanto p  4a 2  6a  3  4  82  6  8  3  307 .

Tercera versión.(Utilizando congruencias, que se estudiarán en el Tema 6)


De 16 p  1  n3 se deduce que n  1 (mod 4) , y por tanto n  4a  1 , luego
16 p  1  64a 3  48a 2  12a  1 
16 p  64a 3  48a 2  12a 
 
4 p  a 16a 2  12a  3  4 | a  a  4b 

4 p  4b 16  16b  12  4b  3 
2


p  b 256b  48b  3 2

Y, puesto que p es primo, b  1  p  256  12  48  1  3  307

5.11
702  am  r 
 787  bm  702  am 85  5  17  m(b  a) 
787  bm  r r  702  am   
 855  cm  702  am  153  32  17  m(c  a)
855  cm  r 
De donde deducimos que m  17 , y por tanto:
702  41  17  5 

787  46  17  5  r  5
855  50  17  5 
412  dn  s 
 722  en  412  dn 310  2  5  31  n(e  d )
722  en  s   s  412  dn   
 815  fn  412  dn 403  13  31  n( f  d ) 
815  fn  s 
De donde deducimos que n  31, y por tanto:
412  13  31  9 

722  23  31  9  s  9
815  26  31  9

Y, finalmente, m  n  r  s  17  31  5  9  62

5.12
El polinomio se podrá expresar de la forma f ( x)  a( x  b)( x  c) , con a, b, c enteros.
Sabemos que 2  3  5  67  2010  f (0)  a(0  b)(0  c)  a b c

Primera parte: a, b, c son positivos.


Nuestro problema se reduce a contar el número de combinaciones posibles de 4 elementos
agrupados en tres cajas a, b, c , teniendo en cuenta que alguna (pero no las tres a la vez) puede
estar vacía (es decir, conteniendo el factor 1). Las cajas b y c son indistinguibles.
a) 4,0,0 a  2010 , b  1, c  1 1 caso
b) 3,1,0 a  p1 p2 p3 , b  p4 , c  1 : 4 casos.
 4
c) 2,1,1 a  p1 p2 , b  p3 , c  p4  : C42    
4!
 6 casos
 2  2!(4  2)!
 4
d) 2,2,0 a  p1 p2 , b  p3 p4 , c  1 : C42    
4!
 6 casos
 2  2!(4  2)!
e) 1,0,3 a  p4 , b  1 , c  p1 p2 p3  : 4 casos.

 4
f) 1,2,1 a  p1 , b  p2 p3 , c  p4  : C42    
4!
 6  6  2  12
 2  2!(4  2)!

g) 0,4,0 a  1 , b  2010 , c  1 : 1 caso


h) 0,3,1 a  1 , b  p1 p2 p3 , c  p4  : 4 casos.
 4
i) 0,2,2 a  1 , b  p1 p2 , c  p3 p4  : C42    
4!
 6 casos/2 =3
 2  2!(4  2)!

Total: 1+4+6+6+4+12+1+4+3 = 41 casos.

Segunda parte: a, b, c con signo.


Dos de estos números pueden ser negativos, no los tres, luego el signo multiplica por cuatro las
posibilidades:
a  0, b  0, c  0 2( x  3)( x  5  67)  2 x 2  676 x  2010
a  0, b  0, c  0 2( x  3)( x  5  67)  2 x 2  676 x  2010
por ejemplo:
a  0, b  0, c  0 (2)( x  3)( x  5  67)  2 x 2  664 x  2010
a  0, b  0, c  0 (2)( x  3)( x  5  67)  2 x 2  664 x  2010

Excepto en el caso a) que solo tiene tres:


2010( x  1)( x  1)  2010 x 2  4020 x  2010
2010( x  1)( x  1)  2010 x 2  4020 x  2010
(2010)( x  1)( x  1)  2010 x 2  2010
(2010)( x  1)( x  1)  2010 x 2  2010

luego el total es 4  40  3  163 polinomios diferentes.

5.13
 p p 2  4  444 p
x 2  px  444 p  0  x 
2
Las soluciones serán enteras si y solo si p 2  4  444 p es entero, es decir, si p 2  4  444 p es
un cuadrado perfecto.

 
Si p 2  4  444 p  p p  24  3  37 es un cuadrado perfecto entonces
 
p | p  24  3  37  p | 24  3  37  p  2 , 3, 37 .
Comprobemos estos posibles valores:
p  2  p 2  4  444 p  3556 no es un cuadrado.
p  3  p 2  4  444 p  5335 no es un cuadrado.
p  37  p 2  4  444 p  67081  2592 es un cuadrado, y las soluciones de la ecuación son 111
y -148.

5.14
Observamos que todo número natural se puede escribir de forma única de la forma
a  2k b , con b impar y 0  k .
Observamos también que si el número impar b es el mismo, digamos a1  2k1 b y a2  2k2 b ,
Entonces
k1  k2  a1  a2 , k1  k2  a1 | a2 , k1  k2  a2 | a1

y en todo caso uno divide al otro.

Entre 1 y 2n hay n números impares, por lo tanto, si asignamos a cada número xi  2ki bi de
nuestro conjunto su valor impar asociado bi , al haber n  1 elementos en nuestro conjunto,
aplicando el Principio del casillero, al menos dos elementos tendrán asociados el mismo impar,
y por tanto, como hemos visto al principio, uno será divisor del otro.

5.15
k será divisor de 1212=224·312, luego será de la forma k=2a3b, con 0  a  24 y 0  b  12 .
1212=224·312 es el menor múltiplo de 66=2636 y 88=224 y k=2a3b, luego 0  b  6 .
12=max{6,b} y de esto deducimos que b=12.
Hay 25 valores diferentes para a, luego hay un total de 25 valores diferentes para k.

5.16
[a,b]=1000=2353
[b,c]=2000=2453
[c,a]=2000=2453
Luego a  2i1 3 j1 , b  2i2 3 j 2 , c  2i3 3 j3 .
Vamos a estudiar, en primer lugar, los exponentes del 3:
Está claro que j1 , j2 , j3  3 , y que al menos dos elementos de j1, j2 y j3 tienen que ser iguales a
3. No hay más limitaciones.
Las opciones son (0,3,3) , (1,3,3) , (2,3,3) , (3,3,3) , (3,0,3) , (3,1,3) , (3,2,3) , (3,3,0) , (3,3,1) ,
(3,3,2) . Hay 10 en total.
Veamos ahora los exponentes del 2:
i1 , i2  3 y entre i1, i2 uno de los dos tiene que ser igual a 3
i2 , i3  4 y entre i2, i3 uno de los dos tiene que ser igual a 4.
i1 , i3  4 y entre i1, i3 uno de los dos tiene que ser igual a 4.
Si i1=3, entonces i3=4, y la única condición para i2 es i2  3 .
Si i2=3, entonces i3=4, y la única condición sobre i1 es i1<=3.
Los casos posibles son 16:
(i1,i2)=(3,0), (3,1), (3,2), (3,3), (0,3), (1,3), (2,3)
Hay 7 casos diferentes.
Luego el total es 7·10=70 triplas (a,b,c) ordenadas diferentes.

5.17
y 2  3x 2 y 2  30 x 2  517  y 2  3x 2 y 2  30 x 2  517 
y 2
    
 10 3x 2  1  10  517  y 2  10 3x 2  1  507  3 132

Casos:

 y  10  3  y  13
2 2

 2 imposible, pues 13 no es un cuadrado.



3 x  1  13 2

 y 2  10  13  y 2  23

 2 imposible, pues 23 no es un cuadrado.

3 x  1  3  13
 y 2  10  132  y 2  132  10  179

 2 imposible, pues 179 no es un cuadrado.

3 x  1  3  13

 y  10  3 13  y  39  10  49  y  7
2 2

 2 , en todo caso, 3x 2 y 2  12  49  588



3x  1  13  3x  12  x  4  x  2
2 2

 y 2  10  1  y 2  11

 2 , imposible, pues 11 no es un cuadrado.

3 x  1  517
 y 2  10  507  y 2  517

 2 , imposible, pues 517 no es un cuadrado.

3 x  1  517

La única solución posible es 588

5.18
Primera versión.
16 p  1 es impar. El cubo de un par es par, luego a es impar: a  2b  1
16 p  1  a 3  (2b  1)3  8b3  12b 2  6b  1 
  
16 p  2b 4b 2  6b  3  8 p  b 4b 2  6b  3 
Por un lado, p es primo, luego impar, y por otro, 4b2  6b  3 es impar.
Luego b  8 y p  4b2  6b  3  307 .

Segunda versión.
Observamos que el cubo de un par es par, luego a debe ser impar, pues 16 p  1 es impar.

16 p  1  a3  24 p  a3  1  (a  1) a 2  a  1 
a  1 es par, a 2  a  1  a(a  1)  1 es impar, luego a  1 debe ser múltiple de 16.
a3  1
Pero p  y por tanto, cualquier otro múltiplo que no sea 16 hará que p no sea primo.
16
Así pues, a  1  16  a  17  p  307 .

Fuente de estas soluciones: artofproblemsolving.com

5.19
m  ( a, b)
a  k1m
m | a, b  
b  k2 m
(a, b)a, b  ab  k1 m k2 m  k1 k2 m 2  ma, b  k1 k2 m 2  a, b  k1 k2 m

Y la ecuación queda
m  k1 k2 m  k1 m  k2 m  6  m  k1 k2 m  k1 m  k2 m  6 
m1  k1 k2  k1  k2   6  mk1  1k2  1  6  1  2  3

Las opciones son:


m  1, k1  1  2, k2  1  3  a  3, b  4  (a, b)  1 , a, b  12
m  2, k1  1  1, k2  1  3  a  4, b  8  (a, b)  4 , a, b  8 
m  3, k1  1  2, k2  1  1  a  9, b  6  (a, b)  3 , a, b  18
m  6, k1  1  1, k2  1  1  a  12, b  12  (a, b)  12 , a, b  12 
m  1, k1  1  6, k2  1  1  a  7, b  2  (a, b)  1 , a, b  14

Las opciones marcadas con  no cumplen la ecuación del enunciado, luego las soluciones son
a, b  3,4,2,7,6,9.
6.1
Si n  6 , está claro que los factores 3 y 6 están dentro de n ! , luego 6  3  9  2 | n!.
Por lo tanto, solo nos tenemos que ocupar de 1! 2! 3! 4! 5! 153  17  9
Así pues,
1! 2! 3! ...  5! 0 (mod 9)
  1! 2! 3! ...  n! 0  0  0 (mod 9)
6! 7! ...  n! 0 (mod 9) 
Y por tanto el residuo al dividirlo entre 9 es 0.

6.2
Por ejemplo: 62  42 (mod 5) , pero sin embargo, 6  4 (mod 5)

6.3
25  32  4  22 (mod 7)

2  2
50 5
   2 
10 2 10
  4
 
4
(mod 7)  25 (mod 7)  22 (mod 7) 
2  16  2 (mod 7)
4


  2
 24 (mod 7)  22 (mod 7)  4 (mod 7) , luego el residuo es 4.

41  6 (mod 7)  1 (mod 7)  4165 (mod 7)  (1)65 (mod 7)  1 (mod 7)  6 (mod 7) , luego el


residuo es 6.

6.4
  4
211  2048  1 (mod 89)  2 44  2 411  211  14 mod 89  1 mod 89 
2 44  1 (mod 89)  1  1 (mod 89)  0 (mod 89)  89 | 2 44  1

212  4096  22 (mod 97)  2 24  212212  22  22 (mod 97) 


 484 (mod 97)  96 (mod 97)  1 (mod 97) 
2 48  2 24  (mod 97)   1 (mod 97)  1 (mod 97) 
2 2

2 48  1 (mod 97)  1  1 (mod 97)  0 (mod 97)  97 | 2 48  1

6.5
9  1mod 10  91003  (1)1003 mod 10  1mod 10  9 mod 10
72  49  1 mod10  7902  72  
451
  1
451
mod10  1 mod10  9 mod10
32  9  1 mod10  3801  3  3800  3  3   2 400
 3 mod10  32  400
mod10 
 3 mod10   1 mod10  3 mod10  1 mod10  3 mod10
400

Luego 91003  7902  3801 mod10  9  9  3 mod10  3 mod10 , luego acaba en 3

6.6
11n  2  11n  112 
 n2
  11  12  11 mod 133
n

11  121  12 mod 133


2

12 2 n 1 2n
 
 12  12  12  12
2 n

  12  11 mod133
n

12  144  11 mod 133


2


n2 2 n 1
an  11  12  12 11  12 11n mod 133  0 mod 133  133 | 11n  2  122n 1
n

6.7
Está claro que 2137 2 acaba en 9, luego:
2137 2  9 mod10  1 mod10  2137 2  
376
  1
376
 1 mod10
Por otro lado, está claro también que 2137  7 mod10

Por lo tanto
2137753  2137752  2137  2137 2376  2137  2137 2  
376
 2137  1 7 mod10  7 mod10 ,
Luego acaba en 7.

Nota: Este problema también se puede resolver sin congruencias, observando en qué dígito van
acabando las potencias de 2137 n :
n  0 1
n 1 7
n29
n 33

Y para exponentes mayores entramos en un bucle.

6.8
Primera versión.
Estudiamos el dígito de las unidades de 31001:
30  1 , 31  3 , 32  9 , 33  27 , 34  81 , 35  253 , 36  759...

Vemos que se va repitiendo en grupos de 4: {1,3,9,7} , y puesto que 1001  1mod 4 ,


31001 acabará como 31 , es decir en 3.

El mismo análisis hacemos para estudiar el dígito de las unidades de 71002 :


70  1 , 71  7 , 72  49 , 73  343 , 74  2401 ,...
Vemos que se va repitiendo en grupos de 4: {1,7,9,3} , y puesto que 1002  2 mod 4 ,
71002 acabará como 7 2 , es decir en 9.

Finalmente, estudiemos el dígito de las unidades de 131003 :


130  1 , 131  13 , 132  169 , 133  2197 , 134  28561 ,...
Vemos que se va repitiendo en grupos de 4: {1,3,9,7} , y puesto que 1003  3 mod 4 ,
131003 acabará como 133 , es decir en 7.

El producto de un número acabado en 3, un número acabado en 9 y un número acabado en 7 es


un número acabado en 9.

Segunda versión.
Vamos a aprovechar el hecho de que toda potencia de un número acabado en 1 acaba siempre
en 1. En nuestro caso, cualquier potencia de 7  13  91 acaba en 1, y cualquier potencia de
34  81 acaba en 1. Luego:

3100171002131003  13  3100171002131002  13  310017  13  13  31001911002 


1002

 
 13  3  31000911002  13  3  34250911002  13  3  34
250
911002 
 13  3  81250911002  39  3  81250911002
Que claramente acabará en 9.

Fuente de esta versión: The Contest Problem Book V 198S-1988, pág. 62

6.9
  2
Para k  1 : 183  5832  833  7  1  186  183  12  1 (mod 7)
Para k  2 : 183  5832  12  49  1  18  18   1  1 (mod 49)
6 3 2 2

Para k  3 : 183  5832  2  2401  1  18  18   1  1 (mod 2401)


6 3 2 2

Observación: Sin embargo, para k  4 cambia la pauta: 183  1030 (mod 74 )

6.10
Si n es impar, entonces n  2k  1 , y por tanto n2  4k 2  4k  1  4k (k  1)  1
k o k  1 es par, luego k (k  1) es par, y por tanto 4k (k  1) es un múltiplo de 8. Luego
n2  4k (k  1)  1  0  1  1 (mod 8) .

6.11
0  n2  3n  2  (n  1)(n  2) (mod 6)
La tabla de congruencias módulo 6 es suficientemente pequeña para enunciar todas las
combinaciones posibles:
0  2  3  5  6  6  1 (mod 6) , y por tanto: (n  1)  2, 5, 6 (mod 6)  n  1,4, 5 (mod 6)
Y cumpliendo la condición 1  n  25 , tenemos:
n  1(mod 6)  n  1,7,13,19,25
n  4 (mod 6)  n  4,10,16,20
n  5 (mod 6)  n  5,11,17,23
En total, 5  4  4 posibilidades.

6.12
Primera versión.
Vemos el comportamiento de 2n  6  9n para los primeros valores de n:
Antes de nada, observamos que 6  1 (mod 7)
n  1  21  2 6  91  9  2 (mod 7)  2n  6  9n  2  2  0 (mod 7)
n  2  22  4 6  92  81  4 (mod 7)  2n  6  9n  4  4  0 (mod 7)
n  3  23  8  1 (mod 7) 6  93  729  1 (mod 7)  2n  6  9n  1  1  0 (mod 7)

Y, a partir de este valor, se van repitiendo la pauta:


n  4  24  232  1  2 (mod 7)  2 (mod 7)
6  94  93  9  1  9  9 (mod 7)  2n  6  9n  2  2  0 (mod 7)
n  5  25  2322  1  22 (mod 7)  22 (mod 7)
6  95  93  92  1  92  92 (mod 7)  2n  6  9n  4  4  0 (mod 7)
n  6  26  2323  1  23 (mod 7)  23 (mod 7)
6  96  93  93  1  93  93 (mod 7)  2n  6  9n  1  1  0 (mod 7)
Y siempre llegamos al mismo resultado: 2n  6  9n  0 (mod 7) .

Segunda versión.
Partimos de la igualdad 9  2 (mod 7) , luego 9n  2n (mod 7) , y por tanto:
2n  6  9n  9n  6  9n  7  9n  0  9n  0 (mod 7)

6.13
Primera versión.
Vamos a calcular 6  8 (mod 49) directamente, mediante el método de las potencias de
83 83

dos:
61  6 (mod 49)
62  36 (mod 49)
64  36  36  1296  22 (mod 49)
68  22  22  484  43 (mod 49)
616  43  43  1849  36 (mod 49)
632  36  36  1296  22 (mod 49)
664  22  22  484  43 (mod 49)
83  64  16  2  1  683  664616626  43  36  36  6
 43  22  6  15  6  41 (mod 49)

81  8 (mod 49)
82  64  15 (mod 49)
84  15  15  225  29 (mod 49)
88  29  29  841  8 (mod 49)
816  8  8  64  15 (mod 49)
832  15  15  225  29 (mod 49)
864  29  29  841  8 (mod 49)
83  64  16  2  1  883  864816828  8  15  15  8 
 15  29  435  43 (mod 49)
Finalmente, 6  8  41  43  84  35 (mod 49)
83 83

Segunda versión.
Aplicando el Binomio de Newton:
 n  n  n  1 n 1
(a  b)n  a n   a n 1b1   a n  2b2  ...   a b  b n
 
1  
2  n  1 

Aplicado a nuestro caso, y teniendo en cuenta que n  83 es impar, y por tanto (1)n  1 ,
n  n  n 
6n  (7  1)n  7 n    7 n 1    7 n  2  ...    7  1
1  2  n  1
 n  n  n 
8n  (7  1) n  7 n    7 n 1    7 n  2  ...    7  1
1  2  n  1

Y por tanto:
n n  n  3  n 
6n  8n  7 n    7 n  2    7 n  4  ...    7    7 
 2  4  n  3  n  1
 n n  n  3 
 2 7 n    7 n  2    7 n  4  ...    7  7n  
  2  4  n  3 
 n n  n  
 2  7 2  7 n  2    7 n  4    7 n  6  ...    7   14n  49k  14n
  2  4  n  3 

En particular, para n  83, a83  683  883  49k  14  83  14  83 (mod 49)


Luego solo nos queda calcular este último residuo:

14  83  1162  49  23  35  14  83  35 (mod 49)

Fuente de la segunda versión: The Contest Problem Book V 1983-1988 (George Berzsenyi, 1997).

Tercera versión.
Aplicando el Teorema de Euler (que se introducirá en el Tema 11)
 1
49  7 2   (49)  491    42 , y a 42  1 (mod 49) si (a, n)  1 .
 7
En nuestro caso (6,49)  (8,49)  1 , y por tanto:
  2
642  1 (mod 49)  683  62421  642 61  61 (mod 49)
842  1 (mod 49)  883  82421  8  8
42 2 1
 81 (mod 49)
1 1 8  6 14
683  883  61  81      14  35 (mod 49)
6 8 48 1
Fuente de esta versión: https://artofproblemsolving.com/wiki/index.php/1983_AIME_Problems/Problem_6

6.14
 
9  99  999  ...  99  00  1 
 9  9  99  (1000  1)  (10000  1)  ...  100
... 
 ... 
999 nueves  999 ceros 
 9  99  (10  1)  (10  1)  ...  10  1
3 4 999
 
Y observamos que si k  3  10k es múltiplo de 1000, y por tanto 10k  0 (mod1000) , luego
10k  1  0  1  1 (mod1000)
Y por tanto:
 
(103  1)  (104  1)  ...  10999  1  (1)99931  (1)997  1 (mod1000)
Y, finalmente,
 
9  99  (103  1)  (104  1)  ...  10999  1 mod(1000)  9  99(1) mod(1000) 
 891 mod(1000)  109 mod(1000)

6.15
Calculamos directamente los primeros valores de f (n) :
f (0)  0 , f (1)  2 , f (3)  6 , f (4)  14 , f (5)  30 , f (6)  62

Analizando cómo se obtiene f (n) llegamos a la conclusión de que


f (n)  2 f (n  1)  2(n  1)  2n  2 f (n  1)  2

Que es el típico comportamiento de una función exponencial. Mirando los primeros valores
vemos que un buen candidato puede ser f (n)  2n  2 .
Lo vamos a demostrar por inducción:
Para n  1,2,3 es cierto.
Suponiendo que f (n)  2n  2 , entonces
f (n  1)  2 f (n)  2  2(2n  2)  2  2n 1  4  2  2n 1  2 .
Luego es cierto para todo n.

f (100)  2100  2 , y queremos determinar 2100  2 (mod100) .


Calculamos 2100 (mod100) con el “Método de las potencias de 2”:
22  4 (mod100)
24  16 (mod100)
28  16  16  256  56 (mod100)
216  56  56  3136  36 (mod100)
232  36  36  1296  96  4 (mod100)
264  (4)  (4)  16 (mod100)
Luego
2100  264 32 4  26423224  16  (4)  16 (mod100)  56  (4) (mod100)
 224 (mod100)  24 (mod100)  24 (mod100)  76 (mod100)

Finalmente: 2100  2  76  2 (mod100)  74 (mod100) , y el residuo pedido es 74.

6.16
Queremos determinar k 2  2k (mod10) . Estudiemos los residuos 2n (mod10) :
21  2 (mod 10)
22  4 (mod 10)
23  8 (mod 10)
24  4  4  16  6 (mod 10)
25  6  2  12  2 (mod 10)
26  2  2  4 (mod 10)
27  4  2  8 (mod 10)
28  8  2  6 (mod 10)
...
En general: 2n (mod10) genera un ciclo 2,4,8,6 , y en particular, para todo n  4k múltiplo
de 4, 24k  6 (mod10) .

En particular, en nuestro caso:


4 | 2008  4 | 20082 

  4 | 2008  2 k
2 2008

2 2008
2  
2 4 251
 2 2 4 251
4 4 251
 4|2 
2008

k es un múltiplo de 4, y por tanto 2k  6 (mod10) .
También vemos que 2008 es múltiplo de 4, luego 22008  6 (mod10) .
Por otro lado, 2008  8 (mod10)  20082  82  64  4 (mod10)
Y por tanto k  20082  22008  4  6  10  0 (mod10)  k 2  02  0 (mod10)
Finalmente, k 2  2k (mod10)  0  6  6 (mod10) , y el dígito de las unidades es 6.

Fuente de la solución: https://artofproblemsolving.com/wiki/index.php/2008_AMC_12A_Problems/Problem_15

6.17
n y 107n tienen las dos últimas cifras iguales si y solo si
107n  n (mod 100)  7n  n (mod 100)  6n  0 (mod 100)  6n  100k
 3n  50k
De aquí deducimos que n es un múltiplo de 50. El múltiplo de 50 más pequeño es n  50 y ya
satisface la condición del enunciado, pues 107  50 acaba en 50.

6.18
Pasando a módulo 3 la ecuación se convierte en x 2  y 2  0
Hacemos la tabla x 2  y 2 (mod 3)

x0 x 1 x2
y0 0 1 1
y 1 1 2 2
y2 1 2 2

Vemos que la única posibilidad es x  0 , y  0 , es decir, x, y múltiplos de 3. Pero entonces:


2 2
   
3z 2  x 2  y 2  3x'  3 y'  9 x'2 9 y'2  9 x'2  y'2  z 2  3 x'2  y'2 , es decir, z 2 es
múltiplo de 3, y por tanto z es múltiplo de 3.
2 2 2
 
Pero entonces x 2  y 2  3z 2  3x'  3 y'  33z'  9 x'2  y'2  9  3z'2  x'2  y'2  3z'2

Este proceso lo puedo repetir una y otra vez, y esto solo puede ocurrir si x  y  z  0 .

6.19
n  182  n2  36n  324 .
Realizamos la división sintética n 2  36n  324 entre n  2 obtenemos un cociente igual a
n  34 y un residuo igual a 256, luego n  18  256 (mod n  2)
2

Así pues
n  2 | n  18  n  18  0 (mod n  2)  256  0 (mod n  2)
2 2

 n  2 | 256  28

Luego n  2 es una potencia 2 k , 0  k  8


1  n  1 , 21  2  n  0 , 22  4  n  2 , 23  8  n  6 , 24  16  n  14 ,
25  32  n  30 , 26  64  n  62 , 27  128  n  126 , 28  256  n  254
Se comprueba que todos estos valores satisfacen la condición del enunciado.

6.20
Sabemos que n  ck ck 1 ck 2 ... c1 c0 es equivalente a
n  ck 10k  ck 110k 1  ck 210k 2  ...  c110  c0
Es decir,
n  p(10) con p( x)  ck x k  ck 1 x k 1  ck 2 x k 2  ...  c1 x  c0 .

Y que las suma de sus cifras será S  p(1) .


Por otro lado, 9  10  1  9 | 10  1  9 | p(10)  p(1)  n  S  n  S (mod 9)

Luego 9 | n  n  0 (mod 9)  S  0 (mod 9)  9 | S

6.21
Sabemos que n  ck ck 1 ck 2 ... c1 c0 es equivalente a
n  ck 10k  ck 110k 1  ck 210k 2  ...  c110  c0
Es decir,
n  p(10) con p( x)  ck x k  ck 1 x k 1  ck 2 x k 2  ...  c1 x  c0 .

Y que la suma alternada de sus cifras es T  p(1) .

Por otro lado, 10  1 (mod 11)  p(10)  p(1) (mod 11) , es decir n  T (mod 11) .
Luego 11 | n  n  0 (mod 11)  T  0 (mod 11)  11 | T

7.1
Está claro que x  x  y  1984  x  1984  x  1984 .
1984  x  y  y  1984  x  y   1984  x   1984  x  2
2
1984 x

De aquí deducimos que 1984 x debe ser entero, luego 1984 x  26  31x tiene que ser un
cuadrado perfecto.

  2
26  31x  23 31x es un cuadrado perfecto  31x es un cuadrado perfecto.
31x  a 2  31 | a 2  31 | a  a  31b  a 2  312 b 2
31x  312 b 2  x  31b 2
x  31b2  1984  b  1984 / 31  8

Para cada valor 1  b  7 obtenemos un valor x  31b2  x  31b , y un valor


y  1984  x   8
2

31  31b  318  b  318  b
2 2 2

La condición x  y se convierte en
31b 2  31(8  b) 2  b 2  (8  b) 2  b 2  64  16b  b 2 
 0  164  b   0  4  b  b  4
Luego son 3 los posibles valores aceptables. Aunque no es necesario para la resolución del
problema, las soluciones son:
b  1  x  31  y  318  1  31  72  ( 31,1519 )
2

b  2  x  124  y  318  2  1116  (124 ,1116 )


2

b  3  x  279  y  318  3  775  (279 ,775 )


2

A partir de este valor se contradice la hipótesis x  y :


b  4  x  496  y  318  3  496  (496 ,496 )
2

7.2
Las condiciones del enunciado se corresponden con el siguiente sistema de ecuaciones
diofánticas:
n  14 2 a  14b  c

n  15 a  15c  b
2

n  6 3 a  6 2 c  6 a  c

cumpliendo, además: 0  a  6 , 0  c  6 , 0  b  14

Simplificamos la tercera ecuación:


n  63 a  62 c  6a  c  a(63  6)a  c(62  1)  222a  37c

y la sustituimos en las otras dos ecuaciones:


222a  37c  14 2 a  14b  c  0  26a  14b  36c  0  13a  7b  18c



222a  37c  15 a  15c  b  0  3a  b  22c
2

Multiplicamos la segunda ecuación por 7 y le restamos la primera:


0  13a  7b  18c
  0  34a  136c  34a  4c 
0  21a  7b  154c

Luego a  4c

Puesto que 0  a  6 , la única posibilidad es que c  1 , y por tanto a  4 , y


n  222  4  37  1  888  37  925

7.3
  
2009  x2  y 4  x  y 2 x  y 2  a  b

Supongamos en primer lugar, que a y b no son primos entre sí. Entonces mcd (a, b)2 es un
divisor de 2009  72  41 , y por tanto la única posibilidad es que mcd (a, b)  7 . Luego:
a  x  y 2  7a' 
  2 x  7(a'b' )  7 es divisor de x, y puesto que es también divisor de a,
b  x  y  7b' 
2

será también divisor de y. Luego:
x  7 x' 
y  7 y '
2
 2 2
 
2 2
2

2

( x  y )( x  y )  7 x'7 y ' 7 x'7 y '  7 x'7 y' x'7 y '  7  41 
2 2 2

x'7 y' x'7 y'   41


2 2

La única posibilidad es
x'7 y'2  41 , x'7 y'2  1  2 x'  41  1  42  x'  21  7 y'2  41  x'  41  21  20 absurdo.

Luego a y b son primos entre sí.


x  y  a
2

  x  a  y 2  a  y 2  y 2  b  2 y 2  b  a . Luego b  a , b  a es positivo, par



x  y  b
2

ba
y es un cuadrado perfecto.
2

Veamos todas las posibilidades:


a)
a7  ba
  b  a  287  7  280   140 no es un cuadrado.
b  7  41 2

b)
a  41  ba
  b  a  49  41  8   4  22 , sí es un cuadrado.
b  7  7 2

x  y 2  41

  2 x  41  49  90  x  45, y  49  45  4  y  2
2
En este caso:
x  y  49
2

  
En efecto, 45  22 45  22  41 49  2009
c)
a 1  ba
  b  a  2009  1  2008   1004 no es un cuadrado.
b  7  7  41  2009 2

Luego las soluciones son x  45 , y  2 .

Fuente de esta solución: Solución oficial de la OME.

7.4
Puesto que los únicos factores de 23 son 23 y 1, tenemos:
c  23
ac  bc  23  c(a  b)  23  
c  1

Supongamos que c  1 . Entonces el sistema queda:


ab  b  44 b(a  1)  44 a  1  b  23  1  22
   (23  a)(a  1)  44  
a  b  23 b  23  a a  21  b  23  21  2

Las soluciones son: a  1, b  22, c  1 y a  21, b  2, c  1 . Efectivamente, las comprobamos:


1  22  22  1  44
a  1, b  22, c  1  
1  1  22  1  23
21  2  2  1  44
a  21, b  2, c  1  
21  1  2  1  23

El caso c  23 no se puede dar puesto que a y b son positivos, y por tanto a  b  1 y


23(a  b)  23

Sin tener en cuenta este hecho, y intentando resolver el sistema, hubiéramos llegado a
soluciones negativas, y por tanto no válidas:
ab  23b  44 ab  23b  44  b(a  23)  44 a  1
   (1  a)(a  23)  44  
23a  23b  23 a  b  1  b  1  a a  21
El número de soluciones es 2.

7.5
abc  70  2  5  7

cde  71  71
efg  72  23  32

La clave está en la segunda ecuación: cde  71 que es un número primo, luego solo puede
ocurrir uno de los tres casos siguientes:
a) c  1, d  1, e  71
Pero entonces 71  fg  72 , lo cual es imposible.
b) c  71, d  1, e  1
Pero entonces ab  71  70 , lo cual es imposible.
c) c  1, d  71, e  1
Tenemos el sistema
ab  2  5  7

 fg  2  3
3 2

La primera ecuación permite los siguientes valores de a :


a  1,2,5,7,2  5,2  7,5  7,2  5  7 , ocho casos diferentes.
La segunda ecuación permite los siguientes valores de f :
f  1,2,22 ,23 ,3,32 ,2  3,2  32 ,22  3,22  32 ,23  3,23  32 , doce casos diferentes.
Luego el total de casos es 8  12  96 .

7.6
Primera versión.
Vamos a resolver la ecuación diofántica y 2  3x 2 y 2  30 x 2  517
Sean a  x 2 y b  y 2 . La ecuación anterior es equivalente a
b  3ab  30a  517  b  3ab  30a  517  b(1  3a)  30a  10  517  10 
b(1  3a)  10(3a  1)  507 (*)

Sea c  1 3a . Puesto que a y b son cuadrados, serán positivos, y también lo será c.

(*)  bc  10c  507  c(b  10)  507  3  132

De todas las soluciones posibles (tomando combinaciones de 3,13,132 ), la única que genera
como soluciones dos cuadrados es:
b  10  3  13  b  49  72  c  13  1  3a  a  4  22
Y por tanto 3x 2 y 2  3ab  3  4  49  588 .

Segunda versión.
 
La ecuación original se puede escribir como y 2  10 3x 2  1  3  132 .
Observamos que, puesto que y es un entero y 3x 2  1 es positivo, y 2  10 debe ser también
positivo. Luego y 2  10  1,3,13,39,169,507 , y por tanto
y 2  11,13,23,49,179,517. El único cuadrado perfecto de esta última lista es 49, luego
y 2  10  39 , y por tanto 3x 2  1  12  x 2  4  3x 2 y 2  12  49  588 .

Fuente de la segunda versión: The Contest Problem Book V (George Berzsenyi)

7.7
4 2 4n  2 m
 1  1  4 n  2 m  m n  4 n  2m  m n  0
m n mn

Completando cuadrados:
0  4n  2m  m n  (m  4)(n  2)  8  (m  4)(n  2)  8  23

Hay cuatro posibilidades:


m  4  1 , n  2  8  m  5 , n  10
m42,n24m6,n6
m4 4,n2 2 m 8,n  4
m  4  8 , n  2  1  m  12 , n  3

7.8
Realizamos la sustitución a  p  q  p  a  q  p  q  a  q  q  a  2q
La ecuación se ha transformado en a  2q  a3
 
a  2q  a3  2q  a3  a  a a 2  1  aa  1a  1

Forzosamente uno de los tres factores deben ser 2.


a  2  2q  2  1  3  6  2  3  q  3  p  a  q  2  3  5
Y en efecto, 8  5  3  (5  3)3

Las otras opciones no son aceptables:


a  1  2  a  3  2q  3  2  4  24  2  12  q  12 no es primo.
a  1  2  a  1  2q  1  0  2  0  q  0 no es primo.

La única solución es p  5 , q  3 .

7.9
1. Caso c  0  c  c
1.1 Si a  b  0 , la ecuación queda
a  b  ab  78  ab  a  b  78 , y completando cuadrados:
ab  a  b  1  78  1  (1  a)(1  b)  79 , y 79 es primo, luego:
1  a  79  a  78
 no cumplen las hipótesis.
1 b 1 b  0 

1.2 Si a  b  0 , la ecuación queda


 (a  b)  ab  78  78  ab  a  b , y completando cuadrados:
ab  a  b  1  78  1  (a  1)(b  1)  79
a  1  79  a  78
 no cumple a  b  0
b 1 1 b  0 
a  1  79  a  80
  c  19  a  b  19   80  2  19  82  0
b  1  1  b  2 

Luego no hay solución posible con c  0 .

2. Caso c  0  c  c :
 a  b  c  19
El sistema queda de la siguiente forma: 
 ab  c  97
2.1 Supongamos que a  b  0 . Entonces
a  b  c  19  c  19  a  b
  ab  (19  a  b)  97  ab  a  b  97  19  116
ab  c  97 
Completamos cuadrados:
ab  a  b  116  ab  a  b  1  116  1  (a  1)(b  1)  117  32  13
a 1  3  a  2 
  c  ab  97  2  38  97  21
b  1  3  13  39  b  38
a 1  9  a  8 
  c  ab  97  8  12  97  1
b  1  13  b  12
a 1 1  a  0 
  c  ab  97  0  97  97
b  1  117  b  116

Las soluciones que aparecen son:


a  2 , b  38 , c  21 , a  38, b  2 , c  21
a  8 , b  12 , c  1 , a  12, b  8 , c  1
a  0 , b  116 , c  97 , a  116, b  0 , c  97
2.2 Supongamos que a  b  0 . Entonces
 (a  b)  c  19  c  19  a  b
  ab  (19  a  b)  97  ab  a  b  97  19  116
ab  c  97 

Completando cuadrados:
ab  a  b  116  ab  a  b  1  116  1  (1  a)(1  b)  117

1 a 1  a  0 
  c  ab  97  0  97  97
1  b  117  b  116
1  a  3  a  2 
  c  ab  97  (2)(38)  97  21
1  b  3  13  39  b  38
1  a  9  a  8 
  c  ab  97  (8)(12)  97  1
1  b  13  b  12

Las soluciones que aparecen son:


a  0 , b  116 , c  97 , a  116, b  0 , c  97
a  2 , b  38 , c  21 , a  38 , b  2 , c  21
a  8 , b  12 , c  1 , a  12 , b  8 , c  1
En total 12 soluciones posibles.

7.10
a  1 (1)

a  1  a  1 , b par (2)
b

a  0, b  0 (3)

x  2
(1) x 2  x  1  1  0  x 2  x  2  ( x  2)( x  1)  
 x  1
Las dos soluciones son aceptables.

x  0  x  2  2
(2) x 2  x  1  1  0  x 2  x  x( x  1)  
x  1  x  2  3
La solución x  1 no es aceptable pues entonces el exponente no es par.

(3) x  2  0  x  2  (2)2  (2)  1  4  2  1  5  0 luego es aceptable.

Luego la ecuación tiene cuatro soluciones: x  2 ,  1 , 0 ,  2

7.11
Primera versión.
En primer lugar, vemos que al menos uno de estos tres números tiene que ser 1.
Supongamos, por el contrario, que a , b , c  1 . Entonces podemos escribir
a  x  1 , b  y  1 , c  z  1 con x, y, z  0 . Pero entonces:
abc  a  b  c  1  ( x  1)( y  1)( z  1)  x  1  y  1  z  1  1 
1  x  y  z  xy  yz  xz  xyz  x  y  z  4  xy  yz  xz  xyz  3
Lo cual es imposible pues x, y, z  0  xy  yz  xz  xyz  4

Así pues, podemos suponer que, por ejemplo, b  1 . Entonces la ecuación queda de la forma:
ac  a  c  2  ac  a  c  2  ac  a  c  1  3  (a  1)(c  1)  3
Y los casos posibles son:
a  1  1 a  1  3
  a  2, c  4   a  4, c  2
c  1  3 c 1  1 
Así pues, la única solución es la terna ( 1 , 2 , 4 ) y todas sus permutaciones, pues las incógnitas
a , b , c de la ecuación del enunciado son perfectamente intercambiables.

Segunda versión.
abc  a  b  c  1  abc  a  b  c  1  a(bc  1)  b  c  1 
bc  1 | b  c  1  bc  1  b  c  1  bc  b  c  2  bc  b  c  1  3
 (b  1)(c  1)  3
Veamos los casos posibles:
b  1  1
  b  2, c  4  a  1
c  1  3
b  1  2
  b  3, c  2  a  6 / 5 , y esta solución no es aceptable, pues no es entera.
c 1  1 
b  1  1
  b  2, c  2  a  5 / 3 , y esta solución no es aceptable, pues no es entera.
c  1  1

Y un último caso: b  1  0  b  1 , que ya se consideró en la primera versión.

Tercera versión.
Supongamos que los tres son iguales. Entonces la ecuación queda de la forma
a3  3a  1 , y entonces:
 
a3  3a  1  a3  3a  1  a a 2  3  1  a | 1  a  1
pero 13  3 1  1, por lo tanto los tres números son son iguales.
Los ordenamos: a  b  c , y al no ser iguales, a  c  a  1  c  a  b  1  b  c  c  c  2c .
Ahora volvemos a la ecuación del enunciado:
abc  a  b  c  1  abc  c  a  b  1  cab  1  a  b  1  c | a  b  1  2c

Solo hay dos posibilidades:


c  a  b  1  ab(a  b  1)  a  b  a  b  1  1 
ab(a  b  1)  2a  2b  2  2(a  b  1)  ab  2  a  1, b  2  c  4
2c  a  b  1  abc  2c  c  3c  ab  3  a  1, b  3  c  5 .
Pero esta solución no es aceptable pues no se satisface la ecuación original:
1 3  5  1  3  5  1
La única solución es a  1, b  2 , c  4 y todas sus permutaciones.

Fuente de las versiones 2 y 3: “Teoría de Números. Entrenamiento de Hidalgo para la Olimpiada Mexicana de
matemáticas” pág. 23.

7.12
Si m  0 , la ecuación 1  1  2  3n no tiene solución por el TFA.
Si m  1 , la ecuación 2  1  3n tiene solución n  1 .
Supongamos que m  2 .
2m  1  3n  2m  3n  1 y puesto que m  2 , 4 | 3n  1  3n  1  0 (mod 4)  3n  1 (mod 4)
Pero 3  1 (mod 4)  3n  (1) n  1 (mod 4)  n es par  n  2k
 2
 
2

Así pues, 2m  1  32k  3k  2m  3k  1  3k  1 3k  1  
  
Para que 3k  1 3k  1 sea una potencia de 2 es necesario que ambos factores sean una
potencia de dos:
3k  1  2 a
 k con a  b  m
3  1  2b
Pero entonces, sumando las dos igualdades llegamos a 2  3k  2a  2b
Si a, b  2 , entonces 4 | 2a  2b , pero está claro que 4 | 2  3k . Así pues, la menor potencia tiene
que ser 2, es decir, 3k  1  2  3k  3  k  1
La ecuación queda 2m  1  32  9  2m  8  m  3 .
Así pues, las dos únicas soluciones son m  1 , n  1 y m  3 , n  2 .

Fuente de la solución: “Teoría de números: Divisibilidad y Congruencias (Entrenamiento de Hidalgo para la


Olimpiada Mexicana de Matemáticas)”, pág. 25

7.13
Pasando a módulo 8: 8m  7  n 2  n2  7  1 (mod 8)
Las soluciones de la congruencia n 2  1 (mod 8) son: 1 , 3 , 5 , 7 (mod 8) , es decir:

n  8k  1 , n  8k  3 , n  8k  5 , n  8k  7
n2  7
Para cualquier entero k, y en todo caso n 2  7 será divisible entre 8 y por tanto: m 
8
Segunda parte:
n2  7
m  1959  n 2  7  1959  8  n 2  1959  8  7  15665
8
El primer cuadrado superior a 15665 es 1252  15625  15625  7 , pero no sirve pues es impar
y por tanto no es divisible entre 8.
El segundo cuadrado superior a 15665 es 1262  15876  15876  7 , pero no sirve pues es
impar y por tanto no es divisible entre 8.
El tercer cuadrado superior a 15665 es 1272  16129  16129  7  16136  8  2017 , luego
m  2017 .

7.14
En primer lugar, vemos que x  0, y  0 es solución trivial de la ecuación. También vemos que
la parte izquierda es positiva, y por tanto x, y deben tener el mismo signo. Si x, y  es
solución, también lo será  x, y  , así pues, podemos suponer que ambos son positivos.

Vamos a estudiar la presencia del factor 3 en la descomposición factorial de x, y .

Supongamos que x  3m a , y  3n b , con m, n  0 y a, b no divisibles entre 3.


Podemos suponer, además, que m  n .

    
34 23 x 2  y 2  x3 y 3  34 23 32 m a 2  32 n b 2  33m a 3 33n b3  23 32 m a 2  32 n b 2  33m  3n  4 a 3b3 
23 33 2n
 2m  2n
a b 3
2 2
 3m  3n  4
ab 2 3
3 3 3
 2m  2n
a b 3
2 2
 3m  n  4 3 3
ab
 2

 8 3m  n a  b 2  33m  n  4 a 3b3

Todo cuadrado es 0 o 1 módulo 3 (ver problema #2.16a), luego la suma de dos cuadrados será
0, 1 o 2 módulo 3, es decir, no será divisible entre 3, y por lo tanto, multiplicada por 8 tampoco
será múltiplo de 3. Así pues, la parte de la izquierda no es múltiplo de 3, y por tanto,
observando la parte de la derecha, necesariamente 3m  n  4  0 , y esto solo pasa cuando
m  n  1.

Por lo tanto, la ecuación queda 8 a 2  b2  a3b3 .  


Por simetría podemos suponer que a  b

   
a3b3  8 a 2  b2  8 a 2  a 2  16a 2  ab3  16

Las únicas opciones para b son b  1, 2 .

   
Si b  1  a  16 y 8 a 2  b2  a3b3  8 a 2  1  a3 ningún posible valor de a resuelve esta
ecuación.

Si b  2  a23  16  a 8  16  a  2 . La pareja b  2, a  1 no soluciona 8 a 2  b2  a3b3 .  


Finalmente, la pareja b  2, a  2 sí es solución de 8 a 2  b2  a3b3 :  
 
8 22  22  64  43  2323

Luego x  3m a  3  2  6 , y  3n b  3  2  6 , es la única solución posible (junto a x  0, y  0


y x  6, y  6 ).
Fuente de esta solución: Solución oficial.

7.15
Para resolver este problema vamos a utilizar que todo cuadrado perfecto es 0 o 1 módulo 3 (ver
Problema 2.16).

Entre el conjunto de todas las soluciones posibles, tomaremos aquella tal que a  b  c sea
mínimo.

Pasando a módulo 3, a 2  2b2  3c 2  2b2 (mod 3) .


Puesto que b 2 es 0 o 1 módulo 3, 2b 2 será 0 o 2 módulo 3. Puesto que es un cuadrado perfecto,
no podrá ser 2 módulo 3, por lo que la única posibilidad es que sea 0 módulo 3:

b2  0 (mod 3)  3 | b2  3 | b

3 | b  3 | 2b 2 

  3 | a  2b  3c  3 | a
2 2 2

3 | 3c 2

3 | b  b  3b'  b 2  9b'2 

2
 
 3c 2  2b 2  a 2  2  9b'2 9a'2  9 2b'2 a'2  9 | 3c 2  3 | c 2
3 | a  a  3a'  a  9a' 
2

3 | c 2  3 | c  c  3c'

2 2 2
b c b2 c2 1 1
 a
Luego 2b' 3c'  2   3   2  3  2b 2  3c 2  a 2     a'2
2 2

3  3 9 9 9 9 3

Es decir, la terna (a' , b' , c' ) también es solución de la ecuación del enunciado, pero
a b c
a'  b'  c'     a  b  c
3 3 3
Contradiciendo la hipótesis de que esta suma era mínima.
Así pues, no existe ninguna solución a esta ecuación que no sea la trivial a  b  c  0

Fuente de la solución: Solución oficial (Ver SE, página 247)

8.1
Sabemos que existirá solución pues (7,9)  1 .
7 x  9 y  3  7 x  3  9 y  31  3 y   3 | x  x  3k 
7  3k - 9y  3  7k - 3y  1  7k  1  3 y
Claramente una solución de esta última ecuación es y  2, k  1 , luego y  2, x  3, k  3 será
una solución particular de la ecuación del enunciado.
En general, las soluciones serán de la forma y  2  9k , x  3  7k , con k  Z

8.2
Sean N a y N b la cantidad de manzanas y naranjas compradas, respectivamente.
Sean Pa y Pb el precio de cada manzana y de cada naranja, respectivamente, en céntimos.

Tenemos las siguientes condiciones:


 N a  N b  12
P  P  3
 a b

N a  Nb
 Pa N a  Pb N b  132

Luego
Pb  3N a  Pb 12  N a   132  Pb N a  3N a  12Pb  Pb N a  132 
3N a  12 Pb  132  N a  4 Pb  44

Esta última ecuación diofántica tiene solución pues d  mcd (1,4)  1 , y 1 | 44 .

Una solución particular de esta ecuación es N a  4, Pb  10 , pero no satisface la condición


N a  N b . El resto de soluciones son de la forma
4
N a  4  k , Pb  10  k
1
k  0  N a  4, Pb  10  N b  8
k  1  N a  8, Pb  9  N b  4, Pa  12
k  2  N a  12, Pb  8  N b  0, Pa  11

Las soluciones posibles son 4 naranjas a 9 céntimos, (con 8 manzanas a 12 céntimos) o 12


manzanas a 11 céntimos (y sin ninguna naranja)

8.3
Aplicamos el algoritmo de Euclides:
858  3  253  99
253  2  99  55 

99  1  55  44   (858,253)  11 , y 11 | 33 , luego la ecuación tiene solución.
55  1  44  11 

44  4 11 

Deshacemos los pasos del algoritmo de Euclides:


11  55  44 
44  99  55 

  11  55  44  253  2  99  99  55  253  2  99  99  55 
55  253  2  99 
99  858  3  253
 253  3  99  253  2  99  2  253  5  99  2  253  5858  3  253 
 2  253  5  858  15  253  5  858  17  253

Y multiplicando ambos lados por 3 obtenemos la solución deseada:


11  5  858  17  253  33  15  858  51 253  x  15, y  51

El conjunto general de soluciones serán todas las parejas de la forma:


253 858
x  15  k  15  23k  8  23k , y  51  k  51  78k
11 11

8.4
Calculamos el (858,253) mediante el algoritmo de Euclides:
258  147  111
147  111  36 
  (858,253)  3 , y 3 | 369 , luego la ecuación tiene solución.
111  36  3  3 
36  3 12 

Deshaciendo los pasos del algoritmo de Euclides:


3  111  3  36 

36  147  111   3  111  3  36  258  147  3147  111  258  147  3  147  3  111 
111  258  147
 258  4  147  3  111  258  4  147  3258  147   258  4  147  3  258  3  147 
 4  258  7  147
Multiplicando por 123 tenemos 369  123  3  492  258  861 147
luego una solución concreta es x  492, y  861 , y el conjunto de soluciones es
x  492  49k , y  861  86k

8.5
x  15  11k , y  27  20k

8.6
La sucesión será de la forma a , a  k , a  2k , a  3k , a  4k , a  5k para un cierto ángulo inicial
a, y sabemos que los ángulos internos de un hexágono suman (6  2)  180º  720º , así pues:
a  a  k  a  2k  a  3k  a  4k  a  5k  720  6a  15k  720  2a  5k  240

Esta última ecuación diofántica tendrá solución pues mcm(2,5)  1 .


Una solución es a  100 , k  8 , con lo que el conjunto de soluciones será de la forma:
a  100  5n
 , con n  Z
b  8  2 n
Tenemos, además, la restricción 0  a  180 , luego
0  100  5n  180  100  5n  80  20  n  16

El valor máximo se toma con n  15  a  100  5  15  175º .

8.7
Primera versión.
Sea n dicho número.
n  10a  9
  10a  9  9b  8  10a  9b  1
n  9b  8 
Esta ecuación diofántica tiene por solución
a  1  9k 
  n  10(1  9k )  9  90k  1 , o equivalentemente, n  90c  1
b  1  10k 

Añadimos una condición más:


n  90c  1
  90c  1  8d  7  90c  8d  8  45c  4d  4
n  8d  7 
Esta ecuación diofántica tiene por solución:
c  4e 
  n  90(4e)  1  360e  1
d  45e  1

Añadimos una condición más:


n  360e  1
  360e  1  7 f  6  360e  7 f  7
n7f 6 
Esta ecuación diofántica tiene por solución:
e  7g 
  n  360(7 g )  1  2520 g  1
d  1  360 g 

Con este resultado ya podemos buscar candidatos, y vemos que con g  1  n  2519 ya
cumple todas las condiciones del enunciado.

Segunda versión.
Sea n el número buscado. Este número se puede escribir como
n  10a9  9  9a8  8  8a7  7  ...  2a1  1

Pero entonces:
n  1  10(a9  1)  9(a8  1)  8(a7  1)  ...  2(a1  1) , es decir:
2,3,4,5,6,7,8,9 | n  1  2520  (2,3,4,5,6,7,8,9) | n  1  n  1  2520k  n  2520k  1

De nuevo, comprobamos que, con k  1 , n  2519 cumple todas las condiciones exigidas.

9.1
d  (9,30)  3 , y 3 | 21 , luego la ecuación anterior tendrá 3 soluciones diferentes.
Encontramos la primera solución por tanteo:
x0  9 , pues 9  9  81  2  30  21.
Luego el resto de soluciones serán:
30
x1  9   1  9  10  19 , efectivamente: 9  19  171  5  30  21
3
30
x2  9   2  9  20  29 , efectivamente: 9  29  261  8  30  21
3

9.2
(3,10)  1 , luego la congruencia lineal anterior tendrá una única solución módulo 10.
Aunque no sea la más elegante, una manera de resolverla es ir probando números del 1 al 9:
3  1  3  7 mod 10 , 3  2  6  7 mod 10 , 3  3  9  7 mod 10 ,
3  4  12  2  7 mod 10 , 3  5  15  5  7 mod 10 , 3  6  18  8  7 mod 10 ,
3  7  21  1  7 mod 10 , 3  8  24  4  7 mod 10 , 3  9  27  7 mod 10 .

Luego la solución es x  9 .

9.3
Observamos que 9  9  81  1 (mod 82) , luego 9  (9)  81  1 (mod 82) . El inverso de 9
módulo 82 es  9  82  9  73 (mod 82) .

9.4
Primera versión: Mediante el método de las potencias de dos.
Vamos calculando 10322  (mod 100) :
n

10321  32 (mod 100)


10322  32  32  24 (mod 100)
10324  24  24  76 (mod 100)
10328  76  76  76 (mod 100)
...

Observamos que 10322   76 (mod 100) para todo n  2 .


n

Por otro lado,


1032  1024  4  2  10321032  103210248  10321024  10328 
10321032 (mod100)  10322  10322 (mod100)  76  76 (mod100)  76 (mod100)
10 3

Segunda versión: Mediante el Teorema Chino del Residuo.


1032  0 (mod 4)  10321032  0 (mod 4)
1032  7 (mod 25)  10322  49  1 (mod 25) 

 10321032  10322516  10322 
516
 (1)516  1 (mod 25)

Nos queda el siguiente sistema de congruencias que resolveremos mediante el Teorema Chino
del Residuo:

1032  0 (mod 4)
1032



1032  1 (mod 25)
1032

N  4  25  100 

N1  25  25 y1  1 (mod 4)   x  25  y1  0  4  19  1  76
N 2  4  4 y2  1 (mod 25)  y2  19

Y por tanto: 10321032  76 (mod 4  25) , tal como queríamos ver.

9.5
Primera versión.
2005  5 (mod 1000)
20052  25 (mod 1000)
20053  25  2005  25  5  125 (mod 1000)
20054  125  2005  125  5  625 (mod 1000)
20055  625  2005  625  5  125 (mod 1000)
20056  125  2005  125  5  625 (mod 1000)


2005  125 (mod 1000) si n es impar
n

Vemos la pauta: n  3  

2005  625 (mod 1000) si n es par
n

En N  200511  200512  ...  20052006 hay 1996 sumandos: 998 pares y 998 impares, luego,
tabajando módulo 1000, tenemos:
N  998  125  998  625  998(125  625)  998  750
Haciendo esta última multiplicación (no hace falta hacerla entera) vemos que acaba en 500.

Segunda versión.
Ante todo vemos que 2005  5 (mod1000)

Por un lado calculamos el sumatorio módulo 125:


Si k  3  5k  10(mod 125) , luego el sumatorio es cero.

Por otro lado, calculamos el sumatorio módulo 8:

Observamos que 52  1 (mod 8)


Luego 5k  1 (mod 8) si es par, y 5k  5 (mod 8) si k es impar, luego
1996
511  512  ...  52006  (1  5)  1996  3  5988  4 (mod 8)
2
Ahora aplicamos el Teorema Chino del Residuo:
 N  125  8
x  0 (mod 125)  N1  8

x  4 (mod 8)   N 2  125  125 y  1 (mod 8)  y  5
 x  8  0  y1  125  5  4  2500  500 (mod1000)

9.6
Si x no es divisible entre tres entonces x  1 mod 3 o x  2 mod 3 .
x  1 mod 3  x  3k  1  x 2  3k  1  9k 2  6k  1  3k (3k  2)  1  x 2  1 mod 3
2

x  2 mod 3  x  3k  2  x 2  3k  2  9k 2  12k  4  3(3k 2  4k  1  1  x2  1 mod 3


2

En todo caso, x 2  1 mod 3 .

Si x es impar, x  2k  1  x 2  2k  1  4k 2  4k  1  4k (k  1)  1 , aquí k o k+1 es par, y


2

por tanto 4k (k  1) es múltiplo de 8, y por lo tanto x 2  1 mod 8 .


Así pues,
 x 2  1 mod 3
 2
 x  1 mod 8
Y ahora aplicamos el Teorema chino del residuo:
N  8  3  24
N1  3  3 y1  1mod 8  y1  3
N 2  8  8 y2  1mod 38  y2  5
x  3  3  1  8  5  1  9  40  49
x  49  1mod 24

10.1
 
5
a) Aplicando PTF, 36  1 (mod 7)  331  356 1  36 3  15  3  3 (mod 7)
b) 4 , c) 9

10.2
Aplicando el PTF, 212  1 (mod13) , luego
 
21000  21283 4  212
83
 24  183  24  24 (mod13)  16 (mod13)  3 (mod13)
El residuo es 3.

10.3
Por el PTF, 1116  1 (mod17) .
 
104  16  6  8 , luego 11104  11166 8  1116 118  1 118  118 (mod17)
6 6

  4
112  121  2 (mod17)  118  112  24 (mod17)  16 (mod17)  1 (mod17)

Finalmente: 11104  1 (mod17)  11104  1  0 (mod17)  17 | 11104  1

10.4
El número a no puede ser múltiplo de 5, pues en ese caso mcd (a,35)  1 . Luego podemos
aplicar el PTF para garantizar que a 4  1 (mod 5) , y por tanto:
  3
a 4  1 (mod 5)  a12  a 4  13  1 (mod 5)
De la misma manera, a no puede ser múltiplo de 7, y de nuevo aplicamos el PTF para
garantizar que a 6  1 (mod 7) , y por tanto:
  2
a6  1 (mod 7)  a12  a6  12  1 (mod 7)
Luego:
a12  1 (mod 5) 

  a  1 mod5,7  a  1 mod 35
12 12

a  1 (mod 7)
12

pues 5,7  mcm(5,7)  5  7  35 ya que mcd (5,7)  1

10.5
Sabemos que 4k  4 (mod 6) para todo k (se demuestra fácilmente por inducción sobre k)
Observamos que la secuencia an parece dar siempre residuo 4 módulo 7:
a1  4  4 (mod 7)
a2  44  256  7  36  4  4 (mod 7)

Veamos que, en general, si k  4 (mod 6)  4k  4 (mod 7) . En efecto:


Por el PTF, 46  1 (mod 7) luego
  j
k  4 (mod 6)  k  6 j  4  4k  46 j  4  46 44  44  4 (mod 7)
Luego an  4 (mod 7) para todo n, en particular, para n  100 .

10.6
Queremos ver (a,42)  1  a6  1  0 mod(3  7  8)  a6  1 mod(3  7  8)

Aplicamos tres veces el PTF:


  3
(a,42)  1  3 | a  a 2  1mod(3)  a 6  a 2  13  1 mod(3)
(a,42)  1  7 | a  a 6  1mod(7)
(a,42)  1  2 | a  a  1mod( 2)  a 6  16  1mod( 2)

Y ahora:
a 6  1 mod(3) 

a 6  1 mod(7)  a 6  1 mod(3,7,2)  a 6  1 mod(3  7  8)
a 6  1 mod( 2)

10.7
Vemos que 7 | 2,3,4,5,6 y por tanto podemos aplicar el PTF:
  2
26  1 (mod 7)  220  236  2  26 22 (mod 7)  1222 (mod 7)  4 (mod 7)
36  1 (mod 7)  330  365  3  (mod 7)  1  1 (mod 7)
6 5 5

46  1 (mod 7)  440  466  4  4   4 (mod 7)  1  4 (mod 7)  4


6 6 4 6 4 4
(mod 7)  4 (mod 7)
  2
En donde hemos aplicado que 42  16  2 (mod 7)  44  42  22  4 (mod 7)
  8
56  1 (mod 7)  550  586  2  56  52 (mod 7)  18  52  52  25  4 (mod 7)
 5  (mod 7)  1 (mod 7)
10
66  1 (mod 7)  660  5106 6

Finalmente, 220  330  440  550  660  4  1  4  4  1  14  0 (mod 7)

10.8
Primera versión.
Aplicando el PTF a nuestro caso:
1335  133  3 (mod 5) 

1105  110  0 (mod 5)
  133  110  84  27  3  0  4  2  4 (mod 5)
5 5 5 5

84  84  4 (mod 5) 
5

275  27  2 (mod 5) 
n5  n (mod 5)  n  4 (mod 5)

Por otro lado:


133  1 (mod 3)  1335  15  1 (mod 3) 

110  1 (mod 3)  1105  (1)5  1 (mod 3)
  133  110  84  27  1  1  0 (mod 3)
5 5 5 5

84  0 (mod 3)  84  0 (mod 3)
5

27  0 (mod 3)  27  0 (mod 3)
5 

Por el PTF, n3  n (mod 3)  n5  n2n3  n2n  n3  n (mod 3) , luego


n  0 (mod 3)

Está claro que n5  1335  1105  845  275  1335  n  133

El primer número n  133 que cumple n  4 (mod 5) y n  0 (mod 3) es 144, y el siguiente es


176.

Enseguida (????) vemos que 176 es ya demasiado grande, por lo tanto la solución debe ser
n  144 .

Segunda versión.
133  3 (mod10)  1335  35  243  3 (mod10)

110  0 (mod10)  1105  0 (mod10) 
  n  2  0  4  7  4 (mod10)
5

84  4 (mod10)  84  4  4 (mod10)
5 5

27  7 (mod10)  27  7  7 (mod10)
5 5 

Pero por otro lado,


n2  n (mod 2)  n5  n2n2n  n3  n2n  n2  n(mod 2) , y por tanto:
n5  n (mod 2)

  n  n (mod10)
5

n  n (mod 5) 
5

Así pues, n5  n  4 (mod10) , y por lo tanto el número n buscado acaba en 4.

Observamos que 133 ,110 , 84 son números muy próximos a múltiplos de 27:
133  5  27  1335  55  275 


110  4  27  1105  45  275   n5  1335  1105  845  275  275 55  45  35  1 
84  3  27  845  35  275  
5
5
 5 5 5
 5 n5  n 
n  27 5  4  3  1  27  4393  4393  5   
5

27  27 

Como antes, sabemos que n  133 , y sabemos que acaba en 4, luego los candidatos son 134,
144, 154, 164…

Está claro que n  134 nos vamos a quedar cortos.

Probando (y utilizado mucho, pero mucho cálculo !!!!) con n  144 :


5
144 16  16  1048576
n  144       4315 , que se aproxima bastante a 4393.
27 3 3 243

Puesto que estamos trabajando con potencias quintas, que crecen de forma muy rápida, está
claro que el siguiente candidato n  154 ya no será muy próximo al valor buscado, por lo que
podemos asegurar que n  144 es la solución del problema.

Nota: El enunciado lleva implícito que este número existe. Ninguna de las dos soluciones
demuestra que 1335  1105  845  275  1445 , son buenos argumentos que justifican que
n  154 es un buen candidato para este resultado.

Fuente de esta solución: https://artofproblemsolving.com/wiki/index.php/1989_AIME_Problems/Problem_9

10.9
p | 29 p  1  29 p  1  0 (mod p)  29 p  1 (mod p) 
29    1
p 2 2
 
 1 (mod p)  292
p
 1 (mod p)

Pero, aplicando el PTF, sabemos que a p  a (mod p) para todo a , luego


  p
1  292  292 (mod p)  1  292 (mod p)  292  1  0 (mod p) 
p | 292  1  840  23  3  5  7  p  2 , 3 , 5 , 7
De los cuatro candidatos posibles, los tres primeros son satisfactorios:
p  2  292  29  1  1 (mod 2)
p  3  293  29  2  1 (mod 3)
p  5  295  29  4  1 (mod 5)
(en donde seguimos aplicando PTF : a p  a (mod p) )
Pero el cuarto primo no es satisfactorio: p  7  297  29  1  1 (mod 7)
Luego las soluciones son tres: p  2 , 3 , 5 .
11.1
i | 2 j  1  2 j  1  0 (mod i)  2 j  1 (mod i)
Si i  1 , 2  0 (mod1) , 1  0 (mod1) , y por lo tanto, trivialmente 2 j  1 (mod i) para j  1 .
Si i  2 , 2  0 (mod1) , 1  0 (mod1) , y por lo tanto, trivialmente 2 j  0  0 (mod i) para todo
j , luego no se cumple la condición del enunciado.
Supongamos que i  3 .
Aplicando el Teorema de Euler, sabemos que si (2, i)  1 , es decir, si i es impar, entonces
2 (i )  1 (mod i) , y por tanto basta tomar j   (i) , que sabemos cumplirá 1   (i)  i .

Si (2, i)  1, es decir, si i es par, no existirá ningún j tal que 2 j  1 (mod i) , pues entonces:
2 j  1 (mod i)  2  2 j 1  1 (mod i)
Y por tanto x  2 j 1 sería solución para la congruencia 2 x  1 (mod i) , pero ya vimos en en el
Tema 9 que no tiene solución.
Tomando j  1 , es imposible, pues 2  1 (mod i) .
Luego se cumple para todos los 1  i  1000 impares, es decir, para 500 números.

11.2
Estudiar los ocho últimos dígitos de la expansión binaria de 271986 es equivalente a determinar
(en binario) 271986 (mod 256)
 1
 (256)   (256)  2561    128 ,
 2
Puesto que 256 | 27 , podemos aplicar el Teorema de Euler para garantizar que
27128  1 (mod 256) .
 
15
Por otro lado, 271986  2715128 66  27128 2766  1152766  2766 (mod 256)

Calcularemos 2766 (mod 256) con el “método de las potencias de dos”:


272  729  729  217  39 (mod 256)  2764   39 (mod 256)
32

(39)2  1521  241  15 (mod 256)   39   39


32
 2 16
   15 (mod 256)
16

 
(15)2  225  (31) (mod 256)   15  (15)2  (31)8 (mod 256)
16 8

 
4
(31)2  961  193  (63) (mod 256)  (31)8  (31)2  (63)4 (mod 256)
 (63) 
2
(63)2  3969  129 (mod 256)  (63)4 2
 1292 (mod 256)
1292  1(mod 256)
Por lo anterior: 2766  2764272  1  217  217 (mod 256)

Y la expresión binaria de 217 es "11011001"

11.3
Queremos determinar a2008 (mod1000) , donde n se ha definido recursivamente:
a1  1 , a2  2a1 , a3  3a2 , ... , a2008  2008a2007
Por un lado, 2008  251  8  2008  0 (mod 8)  n  2008a2007  0 (mod 8) .

Por otro lado, 2008  16  125  8  2008  8 (mod 125)

 1 4
 (125)   (53 )  53 1    53 100 , luego, aplicando el Teorema de Euler, para cualquier
 5 5
número a , con (a,125)  1 , se cumple a100  1 (mod 125) .

Observamos que
2007  7 (mod 100)  20074k  74k  2401k  1k  1 (mod 100)

a2006  2006a2005  (2  1003)a2005  2a2005  1003a2005 es múltiplo de 4, pues seguro que a2005  2 ,
Y por tanto a2007  2007a2006  1 (mod 100)  a2007  100k  1 para cierto k, luego
a2008  2008a2007  2008100k 1  2008100k  2008  2008  8 (mod 125)

Finalmente aplicamos el Teorema Chino del Residuo:


n  0 (mod 8)

n  8 (mod 125)
N  8  125 

N1  125  125 y1  1 (mod 8)   n  125  y1  0  8  47  8  3008  8 (mod1000)
N 2  8  8 y2  1 (mod 125)  y2  47

Así pues, la solución al problema es "008".

Fuente de la solución: Olympiad Number Theory Through Challenging Problems (Justin Stevens, 3E) pág.49

11.4
Queremos calcular f (17)  f (18)  f (19)  f (20) (mod 100) , donde f (x) se ha definido
recursivamente: f1 ( x)  x , f 2 ( x)  x f1 ( x ) ,..., f ( x)  f 4 ( x)  x f3 ( x ) .
 1
 (25)   (52 )  251    20  a 20  1 (mod 25) si (a,25)  1
 5
Los calcularemos por separado.

Primera parte: f (20) (mod 100)


202  400  0 (mod100)  20k  0 (mod100) para todo k par, y puesto que
f 3 (20)  20 f2 ( 20) es par, está claro que f (20)  20 f3 ( 20)  0 (mod 100) .

Segunda parte: f (19) (mod 100)


f (19)  19 f3 (19)  (1) f3 (19) (mod 4)

  f (19)  1 (mod 4)
f 3 (19)  19 f 2 (19)
es impar 

f 2 (19)  19 f1 (19) es claramente impar, luego
f3 (19) (mod  (25))  f3 (19) (mod 20)  19 f2 (19) (mod 20)  1 (mod 20) , y por tanto:
f (19)  19 f3 (19)  19 f3 (19) (mod ( 25))  191  4 (mod 25) , pues 19  4  76  3  25  1
f (19)  1 (mod 4) 
Resolvemos el sistema  mediante el Teorema Chino del Residuo:
f (19)  4 (mod 25)
N  4  25  100 

N1  25  25 y1  1 (mod 4)  y1  1   f (19)  25 1  (1)  4 19  4  279  79 (mod 100)
N 2  4  4 y2  1 (mod 25)  y2  19 

Tercera parte: f (18) (mod 100)


f3 (18)  18 f2 (19)  2  9 2  2 f2 (19)  9 f2 (19)  2k
f (19)

Y por tanto f (18)  18 f3 (19)  2 f3 (18)  22k  4k  0 (mod 4)


184  1 (mod 25) (!!!!)
f 3 (18)  18 f2 (18)  (2  9) f2 (18)  2 f2 (18)  9 f2 (18) es múltiplo de 4
f 4 (18)  18 f3 (18)  1 f3 (18)  1 (mod 25)
f (18)  0 (mod 4) 
Resolvemos el sistema  mediante el Teorema Chino del Residuo:
f (18)  1 (mod 25)
N  4  25  100 

N1  25  25 y1  1 (mod 4)  y1  1   f (18)  25 1  0  4 19 1  279  76 (mod 100)
N 2  4  4 y2  1 (mod 25)  y2  19

Cuarta parte: f (17) (mod 100)


17  1 (mod 4)  f (17)  17 f3 (17)  1 (mod 4)
f (17)  17 f3 (17)  17 f3 (17) mod ( 25) (mod 25)  (**)
f3 (17) mod  (25)  f3 (17) mod 20  17 f2 (17) (mod 20)  17 f2 (17) mod ( 20) (mod 20)  (*)
 (20)  8 , luego: f 2 (17) mod  (20)  f 2 (17)  1717  117  1 mod 8
(*)  171  17 (mod 20)
(**)  1717 mod 25

Vamos a calcular esta última potencia por el "método de las potencias de dos":
17 2  289  14 (mod 25)
17 4  14 14  196  4 (mod 25)
178  (4)  (4)  16 (mod 25)
  2
1717  17 281  178 17  16 2 17  256 17  6 17  102  2 (mod 25)
Así pues, f (17)  2 (mod 25) .
Como antes, aplicamos el Teorema Chino del Residuo para determinar f (17) (mod 100)
f (17)  1 (mod 4) 

f (17)  2 (mod 25)
N  4  25  100 

N1  25  25 y1  1 (mod 4)  y1  1   f (17)  25 1 1  4 19  2  177  77 (mod 100)
N 2  4  4 y2  1 (mod 25)  y2  19

Y finalmente,
f (17)  f (18)  f (19)  f (20) (mod 100)  77  76  79  0  232 (mod 100)  32 (mod 100)

Fuente de la solución: Olympiad Number Theory Through Challenging Problems (Justin Stevens, 3E) pág.50

11.5
Queremos determinar 72014 (mod 1000) .

Primera versión.
Aplicamos el Teorema de Euler:
 (1000)  400 , y puesto que (7,1000)  1 , aplicando el Teorema de Euler:
1  7 (1000)  7400 mod (1000)

  5
Luego 72014  7400514  74005  714  7400  714  1  714  714 mod (1000)

72  49  73  343  74  2401  401  75  2807  807 


 76  5649  649  77  4543  543

  2
714  77  5432  * * *849 (no hace falta hacer toda la multiplicación)
72014  714 mod (1000)  849 mod (1000)

Fuente de esta versión: Solución oficial (Ver SE, página 333)

Nota: En la Solución oficial (SE, página 333) se presenta otra solución alternativa.

Segunda versión.
Como es habitual en este tipo de problemas (ver Problemas #9.5 y #11.3), calcularemos por
separado 72014 (mod 8) y 72014 (mod 125) para después determinar 72014 (mod 1000) mediante
el Teorema Chino del Residuo.

a) 72014 (mod 8)
72  49  1 (mod 8)  72014  72   1007
 11007  1 (mod 8)

b) 72014 (mod 125)


 1 4
 (125)   (53 )  53 1    53 100 , luego, aplicando el Teorema de Euler, para cualquier
 5 5
número a , con (a,125)  1 , se cumple a100  1 (mod 125) .

En nuestro caso se cumple (2014,125)  1, luego 7100  1 (mod 125)


 
Y por tanto 71000  7100
10
 110  1 (mod 125)
 
2
72000  71000  12  1 (mod 125)
7 2  49 (mod 125)
73  343  93 (mod 125)
7 4  651  151  26 (mod 125)
75  182  57 (mod 125)
7 6  219  24 (mod 125)
7 7  168  43 (mod 125)

Y por tanto 714  77  


2
 432  1849  99 (mod 125)

Y por último: 72014  72000  714  1  99  99 (mod 125)

Finalmente, aplicamos el Teorema chino del Residuo:

c)
7 2014  1 (mod 8)
7 2014  99 (mod 125)
N  8  125  1000
N1  125  125 y1  1(mod 8)  y1  5
N2  8  8 y2  1(mod125)  y2  47 (*)
x  125  5  1  8  47  99  37849  849 (mod1000)

Nota: Esta segunda versión no es operativa en un contexto de una competición en la que no se


pueden usar calculadoras, pues, por ejemplo, el paso (*) anterior exige demasiado cálculo.

13.1
Calculando a mano los primeros términos de esta suma observamos su pauta:
20 1 21 2
0 0 0 1  0  0
3 3 3 3
2 3
2 4 3 1 1 2 8 6 2 2
2    1 1 3     2 2
3 3 3 3 3 3 3 3 3 3
4 5
2 16 15 1 1 2 32 30 2 2
4    5 5 5     10   10
3 3 3 3 3 3 3 3 3 3
6 7
2 64 63 1 1 2 128 126 2 2
6     21   21 7     42   42
3 3 3 3 3 3 3 3 3 3
8 9
2 256 255 1 1 2 512 510 2 2
8     85   85 9     170   170
3 3 3 3 3 3 3 3 3 3

Si n  2 es par, 2n  1 es múltiple de 3.
En efecto: 2  1 (mod 3)  2n  (1)n  1 (mod 3) si n es par.
Por lo tanto:
 2n   2n  1 1  2n  1  1  2n  1
 3    3  3  3  3  3
     
 2n 1   2n 
Y también vemos que si n  2 es par,    2  3  , luego
 3   
2 n 1
2 n 1
 2 1 1 
n
2 1 2
n
2 
n 1
 2 n  1
2  2    2     2  3 
3 3  3 3 3 3  3   
es decir, van por parejas: un valor y su doble.

Añadimos el sumando correspondiente a 1001 para tener 501 parejas completas:


1001
 2n  1001 2n  1001 2n  1  2n  1   1001  2n  1  1001 n
      3     3  2 3     3 3    2  1 
n0  3  n2   n2     n2   n2
n par n par n par

 1001 
   500   500 
   2n   500    22 n   500    4n   500  (*)
 n2   n 1   n 1 
 n par 
k
1  r k 1
Aplicamos la fórmula de la serie geométrica:  r n 
n 0 1 r
500
 500 n  1  4501 1  4501 4501  1

n 1
4  4  1 
n

 n 0  1 4
1 
3
1 
3
1

Finalmente:
4501  1 4501  1
(*)   1  500   501
3 3

A este resultado le debemos restar el último valor que hemos contado de más:
 21001  2  21000 
 
   
 2 21000  1 2   2 21000  1 2  2 21000  1
   
 
 3   3  2 
     3 3  3 3 3

Por lo tanto, el resultado es


4501  1
 501 
  
2 21000  1 4501  1  2 21000  1

 501 
3 3 3
4501  1  21001  2 4501  21001  1 22501  21001  1
  501   501   501 
3 3 3
21002  21001  1 2  21001  21001  1 21001  1
  501   501   501
3 3 3
1 (mod 3) si n es par
Nota: 2n  
2 (mod 3) si n es impar

13.2
Primera versión.
Dibujando una cuadrícula 10x10 y tachando manualmente uno por uno todos los múltiplos de
3, vemos que hay 50 múltiplos de 2 de los cuales 16 son múltiplos de 3 luego
50  16 34 17
P  
100 100 50
Segunda versión.
100 
Hay 100 / 2  50 números pares, y hay   16 números divisibles entre 2 y 3, luego hay
 6 
50  16  34 números divisibles entre 2 y no entre 3.
50  16 34 17
P  
100 100 50

13.3
La potencia más alta de 7 que divide a 1000! viene dada por:
1000  1000  1000  1000 
 71   142 ,  7 2   20 ,  7 3   2 , y si k  4  7 k   0

1000 
Luego   k   142  20  2  164
k 1  7 

La potencia de 7 más alta que divide a 500! viene dada por:


 500   500   500   500 
 71   71 ,  7 2   10 ,  73   1 y si k  4  7 k   0

 500 
Luego   k   71  10  1  82
k 1  7 

1000  1000! 1000!


Finalmente, teniendo en cuenta que    
 500  500! 500! 500!
2

1000 
la potencia más alta de 7 que dividirá   será 164  2  82  0 , luego 7 no es un divisor.
 500 

13.4
 200  200! 200! 200 199 198  ...  2 1
n       
 100  100!100! 100! 100  99  98  ...  2 1
2 2 2 2 2 2

200 199 198  ... 102 101



100  99  98  ...  2 1

En el denominador aparecen todos los primos menores de 100 al menos una vez, luego
necesitamos el número primo p menor que 100 que aparezca al menos dos veces en el
200
numerador, es decir, de forma que 3 p  200  p   66
3
El mayor número primo menor que 66 es 61, y 100  2  61  122  200 , y
100  3  61  183  200 , y solo uno de estos dos factores 61 se anulará con el 61 del
denominador.
La solución es 61.

14.1
Por el TDB, (m, n)  am  bn para ciertos enteros a, b . Luego
(m, n)  n  am  bn  n   m  n  m n  n
       a  b    a    b 
n  m n  m  n  m  n  m  m
m n
Con lo que nuestro problema se reduce a demostrar que   es un entero.
n  m 

n  n 1
Ahora aplicamos la igualdad m   n  (Ver Tema 2 de PC)
 m  m  1

m  n  1  n  1  n 1  n 1
   m   n   que es un número combinatorio, y por tanto entero.
n  m  n  m  n  m  1  m  1

Fuente de la solución:
Number Theory: Concepts and problems (Andreescu, Dospinescu, Mushkarov, 2017) pág. 65

16.1
Basta aplicar n  p1a1 p2a2 ... prar   (n)  a1  1a2  1...ar  1
 (n) es impar  ai  1 es impar 1  i  r  ai es par 1  i  r  ai  2bi 1  i  r

 n  p1a1 p2a2 ... prar  p12b1 p22b2 ... pr2br  p1b1 p2b2 ... prbr 
2

16.2
Está claro que si n  p q1 con p, q primos, entonces  (n)  q  1  1  q es primo.
Sea n  p1a1 p2a2 ... prar y supongamos que  (n)  a1  1a2  1...ar  1 es primo. Entonces está
claro que r  1 . Luego
n  p a y  (n)  a  1 primo, y podemos escribir n  p ( n )1 con  (n) primo, como queríamos.

16.3
1099  2  5  299  599
99

El número total de divisores es (99  1)(99  1)  1002  10000


De estos divisores, los múltiplos de 1088 serán aquellos de la forma 2a 5b , con 88  a, b  99 , es
decir, 12 12  144 números.
144 6
Luego la probabilidad es 
10000 625

16.4
Puesto que 6 solo se factoriza como 2  3 o 6 1, el número n solo puede constar de dos factores
primos:
a  1  2
n  p a qb    a  1, b  2  n  p  q con p  q .
2

b 1  3
O bien:
n  p a ,  a  1  6  a  5  n  p5

16.5
1010  2  5  210 510 , y sus divisores son todas las combinaciones posibles de la forma
10

n  2a5b con 0  a, b  10 . Hay 112  121 en total.


157  3  5  37 57 , y sus divisores son todas las combinaciones posibles de la forma
7

n  3a5b con 0  a, b  7 . Hay 82  64 divisores posibles.


Pero los divisores de la forma n  305b  5b con 0  b  7 ya aparecen como divisores del
primer número, por lo tanto los restamos. En total hay 64  8  56 divisores.

 
11
1811  2  32  211 322 , y sus divisores son todas las combinaciones posibles de la forma
n  2a3b con 0  a  11 y 0  b  22 . Son 12  23  276

Pero de estos están repetidos aquellos de la forma


n  2a30 con 0  a  10 : 11 divisores, y aquellos de la forma
n  203b con 1  b  7 : 7 divisores. (¡Atención! No contemos el divisor 1 dos veces)
Luego hay 276  11  7  258 nuevos.

Así pues, hay un total de 121  56  258  435 divisores.

16.6
Para n  1 ,  (1)  1 ,  (2)  2 y está claro que no cumple la condición. Luego n  2 , y por tanto
 (n) ,  (n  1)  2 y en consecuencia  (n)   (n  1)  7   (n) ,  (n  1)  5 .

 (n)  1  n  1 y ya hemos visto que no cumple la igualdad del enunciado.

Luego las posibilidades que quedan son:


a)  (n)  2 , (n  1)  5
b)  (n)  3 , (n  1)  4
c)  (n)  4 , (n  1)  3
d)  (n)  5 , (n  1)  2

Teniendo en cuenta que:


 ( n)  2  n  p
 ( n)  3  n  p 2
 ( n)  5  n  p 4
n  p  q, p  q
 ( n)  4  2  2  
n  p
3

Tenemos
a) n  p , n  1  p 4
b) n  p 2 , n 1  p q
n  p 2 , n  1  p3
c) n  p 3 , n  1  q 2
n  p q , n  1  q2
d) n  p 4 primo y n 1  q primo.

Observamos que en todos los casos, uno de los dos números consecutivos es la potencia par de
un número primo, y con esto vamos probando casos:
n  3 , n  1  4   (3)  2 ,  (4)  3
22  4   ninguna cumple.
n  4 , n  1  5   (4)  3 ,  (5)  2
n  8 , n  1  9   (8)  4 ,  (9)  3
32  9   ambas cumplen la condición del enunciado.
n  9 , n  1  10   (9)  3 ,  (10)  4
n  15 , n  1  16   (15)  4 ,  (16)  5
24  16   cumple la segunda.
n  16 , n  1  17   (16)  5 ,  (17)  2
n  24 , n  1  25   (24)  8 ,  (25)  3
52  25   cumple la segunda.
n  25 , n  1  26   (25)  3 ,  (26)  4
n  48 , n  1  49   (48)  10 ,  (49)  3
7 2  49   ninguna cumple.
n  49 , n  1  50   (49)  3 ,  (50)  6
n  80 , n  1  81   (80)  10 ,  (81)  5
34  81   ninguna cumple.
n  81 , n  1  82   (81)  5 ,  (82)  4
n  120 , n  1  121   (120)  16 ,  (121)  3
112  121   cumple la segunda.
n  121 , n  1  122   (121)  3 ,  (122)  4
n  168 , n  1  169   (168)  16 ,  (169)  3
132  169   ninguna cumple.
n  169 , n  1  170   (169)  3 ,  (170)  8
n  288 , n  1  289   (288)  18 ,  (289)  3
17 2  289   ninguna cumple.
n  289 , n  1  290   (289)  3 ,  (290)  8
n  360 , n  1  361   (360)  24 ,  (361)  3
192  361   cumple la segunda.
n  361 , n  1  362   (361)  3 ,  (362)  4

Y paramos porque hemos conseguido las seis soluciones pedidas. El resultado es


8+9+16+25+121+361=540

Fuente de esta solución: www.artofproblemsolving.com/wiki/index.php/2019_AIME_I_Problems/Problem_9

16.7

2004  22  3 167  20042004  22  3 167 2004
 24008  32004 167 2004 .

Los divisores de 20042004 son todos los números de la forma

d  2a  3b 167c , con 0  a  4008 , 0  b  2004 , 0  c  2004 .

Luego  d   a  1b  1c  1 , y por tanto

 d   2004  a  1b  1c  1  22  3 167

O equivalentemente, todas las posibilidades x, y, z con x y z  22  3 167 y


1  x  4009 , 1  y  2005 , 1  z  2005 .

Colocamos primero los "2": Hay 6 formas diferentes.


2–2-/
2-/-2
/-2–2
22 - / - /
/ - 22 - /
/ - / - 22

Ahora tenemos que colocar el 3 y el 167, en cualquiera de las tres posiciones, luego el total será
6  3  3  54 formas diferentes.

17.1
1815  3  5 112
Sabemos que n  2a3b para ciertos enteros positivos a, b . Sabemos que entonces
2a1  1 3b1  1 1 a1
 ( n) 
2 1

3 1
 
 2  1 3b1  1
2

A falta de algo mejor, vamos viendo los diferentes valores que se obtienen:
k 2k 1  1 3k 1  1
1 3 8
2 7 26=2·13
3 15=3·5 80=24·5
4 31 242=2·112
5 63=3·31 728=23·7·13

Vemos que la única combinación que se adapta a nuestro problema es a  3 y b  4 :

2

1 31
 1
2  1 341  1  3  5  2 112  1815 . Así pues, n  2334  8  81  648 .
2

18.1
Queremos ver que 7n  3 (mod 10)  7n  43 (mod 100)
Vamos viendo el comportamiento de las potencias de 7 módulo 100:
70  1
73  63  3 (mod 10)
71  7
7 4  21  1 (mod 10)
7 2  49  9 (mod 10)
Vemos que hay un bucle de longitud 4, y que 7n  3 (mod 10)  n  3 (mod 4) .
Veamos ahora como son las potencias de 7 módulo 100:
70  1
73  343  43 (mod 100)
7 7
1

7 4  301  1 (mod 10)


7  49
2

Vemos que también hay un bucle de longitud 4, y que 7n  43 (mod 100)  n  3 (mod 4) ,
luego 7n  3 (mod 10)  n  3 (mod 4)  7n  43 (mod 100) .

18.2
2222  3 (mod 7)
32  9  2 (mod 7)
33  6  1 (mod 7)

5555  3  1851  2
 
22225555  35555  331851 2  33 3   1
1851 2 1851
2  2  5 (mod 7)

5555  4 (mod 7)
42  16  2 (mod 7)
43  8  1 (mod 7)

2222  3  740  2
 
55552222  42222  43740 2  43 4  1
740 2 1851
2  2 (mod 7)

22225555  55552222  5  2  0 (mod 7)


Fuentes.
# Título Página
1.6 Elementary Number Theory Notes (David A. Santos, 2004) 23
1.7 Elementary Number Theory Notes (David A. Santos, 2004) 3
1.8 Elementary Number Theory Notes (David A. Santos, 2004) 23
1.9 Elementary Number Theory Notes (David A. Santos, 2004) 24
2.5 Problem-Solving and Selected Topics in Number (Theory Michael Th.Rassias) 242
2.8 Elementary Number Theory, Revised Printing (David M.Burton, 1980) 23
2.17 Problem-Solving and Selected Topics in Number Theory (Michael Th. Rassias) 153
3.2 Elementary Number Theory Notes (David A. Santos, January 2004) 66
5.2 Problem-Solving and Selected Topics in Number (Theory Michael Th.Rassias) 244
5.13 Teoría de Números. Entrenamiento de Hidalgo para la OMM. 24
5.19 Mathematical Excalibur Volume 5, Number 4, September 2000 – November 2000 3
6.1 Elementary Number Theory, Revised Printing (David M.Burton, 1980) 75
6.2 Elementary Number Theory, Revised Printing (David M.Burton, 1980) 76
6.3 Elementary Number Theory, Revised Printing (David M.Burton, 1980) 76
6.4 Number Theory: Concepts and problems (Andreescu, Dospinescu, Mushkarov, 2017) 6
6.5 Number Theory: Concepts and problems (Andreescu, Dospinescu, Mushkarov, 2017) 6
6.8 Elementary Number Theory, Revised Printing (David M.Burton, 1980) 100
8.2 Elementary Number Theory (David M.Burton, 6E) pág.35
9.2 Olympiad Number Theory Through Challenging Problems (Justin Stevens, 3E) 48
7.11 Teoría de Números. Entrenamiento de Hidalgo para la OMM. 23
7.12 Teoría de Números. Entrenamiento de Hidalgo para la OMM. 25
10.5 Elementary Number Theory Notes (David A. Santos, 2004) 142
11.1 Olympiad Number Theory Through Challenging Problems (Justin Stevens, 3E) 46
11.3 Olympiad Number Theory Through Challenging Problems (Justin Stevens, 3E) 49
11.4 Olympiad Number Theory Through Challenging Problems (Justin Stevens, 3E) 49
11.2 Problem-Solving Strategies With 223 Figures (Arthur Engel, 1998) 134
13.1 Olympiad Number Theory Through Challenging Problems (Justin Stevens, 3E) 46
14.1 Number Theory: Concepts and problems (Andreescu, Dospinescu, Mushkarov, 2017) 65
Apéndice
El "problem-solving", tal y como yo lo entiendo.
La resolución de poblemas, el llamado "problem-solving" es la experiencia más apasionante de
las matemáticas. Los "ejercicios", tan repetitivos propios de los libros de texto, pasan ahora a
ser "problemas", y cada problema es una aventura única, es un enemigo desconocido al que el
estudiante es llamado a enfrentarse con valentía. La resolución de problemas no es muy
importante ni poco importante, es lo único importante en matemáticas. Pero el problem-
solving no es para pusilánimes. Dejemos algunas cosas claras:

1. El problem-solving requiere tiempo: Cada problema exige tiempo para pensarlo, tiempo
para resolverlo, y en la mayoría de las veces, tiempo, mucho tiempo para estudiar
detenidamente la solución propuesta cuando hemos fracasado en su resolución. Solo así se
aprende, día tras día, semana tras semana, año tras año. Solo después de muchos fracasos llegan
los primeros éxitos.
Todos los problemas de los libros de "Toomates Cool·lección" se ofrecen siempre con las
soluciones totalmente desarrolladas, pero no mires nunca la solución, no te rindas, hasta haber
dedicado al problema todo el tiempo necesario... y un poco más.

2. La frustración es inevitable, pero la impotencia que uno siente al fracasar intentando resolver
problemas demasiado difíciles puede llegar a quemar al estudiante de matemáticas, por ello es
fundamental seleccionar problemas de dificultad adecuada.

Todos los problemas de los libros de "Toomates Cool·lección" se presentan siempre indicando
su dificultad: MF: Muy fácil, F: Fácil, M: Dificultad media, D: Difícil, MD: Muy difícil.

Aunque hay que dejar claro que el grado de dificultad de un problema es algo muy subjetivo:
Aquello que alguien puede considerar difícil puede ser muy fácil para otro.

3. Todo juego exige unas reglas, reglas que deben estar claras. Es muy frustrante (aunque muy
enriquecedor) enfrentarse durante horas a un problema para finalmente descubrir que se están
utilizando técnicas o conceptos que uno desconoce. La sensación de haber perdido el tiempo
miserablemente puede ser muy desoladora. Las técnicas y conceptos teóricos que se utilizan
en la resolución de los problemas deben estar claros.

Los libros de problemas de "Toomates Cool·lección" se acompañan con los "libros de teoría"
en donde se recopilan de una forma ordenada todos los contenidos teóricos utilizados en las
resolución de los problemas.

4. La resolución de problemas supone en el estudiante un nivel importante de iniciativa y


autonomía. Los libros de "Toomates Cool·lección" son un recurso más que el estudiante tiene
a su disposición en su biblioteca personal, biblioteca que deberá enriquecer con la adquisición
de infinidad de otros recursos encontrados en Internet, y muchos libros, gratuitos y comprados,
digitales y en papel.

5. El problem-solving requiere la máxima concentración. Cuando nos enfrentamos a un


problema, ¡el cerebro encendido y el móvil apagado!
Las competiciones AMC, un excelente sendero hacia las IMO detrás de un
mar de siglas.
AMC "American Mathematics Competitions"
Es el programa de competiciones matemáticas organizado por la MAA (Mathematical
Association of America) para la selección del equipo que representará a USA en la IMO.
Organiza el sistema de pruebas selectivas AMC10/12, AIME y USAMO.

El sistema escolar USA consta de 12 cursos ("grades") divididos en 3 niveles, que corresponden a las siguientes
edades: Elementary school (Preschool: 4-5, Kindergarten: 5-6, 1st Grade: 6-7, 2nd Grade: 7-8, 3rd Grade: 8-9,
4th Grade: 9-10, 5th Grade: 10-11), Middle school (6th Grade: 11-12, 7th Grade: 12-13, 8th Grade: 13-14), High
school (9th Grade "Freshman":14-15, 10th Grade "Sophomore": 15-16, 11th Grade "Junior": 16-17, 12th Grade
"Senior": 17-18)

AHSME "American High School Mathematics Examination" (1949-2000)


Es la antigua competición matemática para los grados 9 a 12. A partir del año 2000 desaparece
al bifurcarse en AMC10 (Grado 10) y AMC12 (Grado 12).
Consta de 30 preguntas "tipo test" con 5 posibles respuestas, para resolver en 90 minutos.
Los estudiantes que alcanzan los 100 puntos o más de los 150 posibles obtienen el "AHSME
Honor Roll", y son invitados a participar en la AIME (American Invitational Mathematics
Examination). Se suelen clasificar unos 4000 estudiantes anualmente.
Para alcanzar estos 100 puntos, los estudiantes deben contestar correctamente
aproximadamente la mitad de las 30 preguntas y dejar en blanco el resto, pues las respuestas
equivocadas conllevan severas penalizaciones.
Las calculadoras se permiten a partir de 1994, aunque no son necesarias.

AMC8 "American Mathematics Competition Grade 8"


Prueba de 25 preguntas "tipo test" en 40 minutos, para estudiantes de Grado 8 (13-14 años, el
2º ESO en España).
Cubre (aunque no está limitado a ellos) los temas propios del currículum de la "Middle
School": Combinatoria, probabilidad, estimación, razonamiento de proporcionalidad, geometría
elemental incluyendo teorema de Pitágoras, visión espacial, aplicaciones en la vida cotidiana,
lectura e interpretación de gráficos y tablas.
Además, en las últimas preguntas pueden aparecer funciones y ecuaciones lineales y
cuadráticas, geometria cartesiana y algunos elementos de álgebra básica.

AMC10/12 "American Mathematics Competition Grades 10 & 12"


Prueba de 25 preguntas "tipo test" en 75 minutos.
La AMC10 está pensada para estudiantes hasta el grado 10 (el 4º de ESO en España), y 17.5
años de edad como máximo, y cubre el currículum hasta dicho grado.
La AMC12 está pensada para estudiantes hasta el grado 12 (el 2º de Bachillerato en España), y
cubre todo el currículum de la "high school", incluyendo trigonometría, álgebra avanzada,
geometría avanzada, pero excluyendo el calculus.

Existen dos versiones de dichas pruebas: A y B, con la misma estructura y el mismo nivel de
dificultad. Las preguntas son diferentes porque se presentan en fechas diferentes. Los
estudiantes se pueden presentar a ambas pruebas.
AIME "American Invitational Mathematics Examination"
Prueba de 15 preguntas en 3 horas. Las respuestas son siempre números positivos de tres
dígitos. Son convocados los mejores estudiantes en AMC10 y/o AMC12. Su primera edición
fue en el año 1983.

USAMO y USAJMO "USA Mathematical Olympiad y USA Junior Mathematical Olympiad"


Prueba de 6 preguntas en dos días, 9 horas de duración.
A la USAMO son convocados los mejores estudiantes en AMC12 y AIME (alrededor del 5%
superior). A la USAJMO son convocados los mejores estudiantes en AMC10 y AIME
(alrededor del 2.5% superior). Solo pueden presentarse estudiantes americanos y estudiantes en
escuelas americanas o en Canadá.
Los 6 estudiantes con mejores puntuaciones en el combinado AMC10/12, AIME y USAMO
forman el equipo que representa a USA en la Annual International Mathematical Olympiad
(IMO)

IMO "International Mathematical Olympiad"


Es una competición anual para estudiantes preuniversitarios y es la más antigua de las
Olimpiadas Internacionales de Ciencias.1 La primera IMO se celebró en Rumania en 1959.
Desde entonces se ha celebrado cada año. Cerca de cien países de todo el mundo envían
equipos de un máximo de seis estudiantes junto con un líder de equipo, un tutor - o colíder - y
observadores. La competición consta de dos cuestionarios con tres problemas cada uno. Cada
pregunta da una puntuación máxima de 7 puntos, con una puntuación máxima total de 42
puntos. La prueba se desarrolla en dos días, en cada uno de los cuales el concursante dispone de
cuatro horas y media para resolver tres problemas. Estos se escogen entre varias áreas de la
matemática vista en secundaria, los cuales pueden clasificarse grosso modo en geometría, teoría
de números, álgebra y combinatoria. No se requieren conocimientos de matemáticas superiores
y de las soluciones se espera que sean cortas y elegantes. Encontrarlas requiere, sin embargo,
ingenio excepcional y habilidad matemática.

Otras siglas de interés:

MOP "Mathematical Olympiad Summer Program"


Programa de entrenamiento para el equipo IMO de los Estados Unidos.

También podría gustarte